Vous êtes sur la page 1sur 364

050 – METEOROLOGY

050-01 THE ATMOSPHERE


050-01-01 Composition, extent, vertical division
8814. The troposphere is the: A – part of the atmosphere above the stratosphere B –
part of the atmosphere below the tropopause C – boundary between the mesosphere and
thermosphere D – boundary between the stratosphere and the mesosphere Ref: all Ans:
B 8817. What is the boundary layer between troposphere and stratosphere called: A –
Tropopause B – Ionosphere C – Stratosphere D – Atmosphere Ref: all Ans: A 8824. The
amount of water vapour which air can hold largely depends on: A – relative humidity
B – air temperature C – stability of air D – dew point Ref: all Ans: B
8876. The tropopause is a level at which: A – vertical currents are strongest B –
water vapour content is greatest C – pressure remains constant D – temperature
ceases to fall with increasing height Ref: all Ans: D 8889. The tropopause is
lower; A – south of the equator than north of it B – in summer than winter in
moderate latitudes C – over the North pole than over the equator D – over the
equator than over the South Pole Ref: all Ans: C 10050. What is the approximate
composition of the dry air by volume in the troposphere? A – 21% oxygen, 78%
nitrogen, and the rest other gases B – 10% oxygen, 89% nitrogen, and the rest other
gases C – 88% oxygen, 9% nitrogen, and the rest other gases D – 50% oxygen, 40$
nitrogen and the rest other gases Ref: all Ans: A 10055. In which layer is most of
the atmospheric humidity concentrated? A – Troposphere B – Tropopause C –
Stratosphere D – Stratopause Ref: all Ans: A
10061. The thickness of the troposphere varies with: A – latitude B – longitude C –
rotation of the earth D – the wind Ref: all Ans: A 10102. Going from the equator to
the north pole, the altitude of the tropopause: A – increases and its temperature
increases B – decreases and its temperature increases C – increases and its
temperature decreases D – decreases and its temperature decreases Ref: all Ans: B
10753. In the mid-latitudes the stratosphere extends on an average from: A – 85 to
more than 200 km B – 0 to 11 km C – 50 to 85 km D – 11 to 50 km Ref: all Ans: D
10804. Which layer of the atmosphere contains more than 90 per cent of all water
vapour? A – Troposphere B – Lower stratosphere C – Upper stratosphere D –
Ionosphere Ref: all Ans: A
10825. Which one of the following statement applies to the tropopause? A – It is,
by definition, a temperature inversion B – It is, by definition, an isothermal
layer C – It indicates a strong temperature lapse rate D – It separates the
troposphere from the stratosphere Ref: all Ans: D 10846. The average height of the
tropopause at 50oN is about: A – 14 km B – 8 km C – 11 km D – 16 km Ref: all Ans: C
10851. The troposphere: A – has a greater vertical extent above the equator than
above the poles B – contains all oxygen of the stratosphere C – is the separation
layer between the stratosphere and atmosphere D – reaches the same height at all
latitudes Ref: all Ans: A 10855. How does the height of the tropopause normally
vary with latitude in the northern hemisphere? A – It remains constant throughout
the year B – It remains constant from north to south C – It increases from south to
north D – It decreases from south to north Ref: all Ans: D
10896. What, approximately, is the average height of the tropopause over the
equator? A – 40 km B – 8 km C – 11km D – 16 km Ref: all Ans: D 15783. Why are
indications about the height of the tropopause not essential for flight
documentation in the tropics? A – The tropopause is generally well above the flight
level actually flown B – The meteorological services are unable to provide such a
chart C – The temperatures of the tropical tropopause are always very cold and
therefore not important D – Tropopause information are of no value Ref: all Ans: A
15790. What is the most likely temperature at the tropical tropopause? A - -25oC B
- -75oC C - -55oC D - -35oC Ref: all Ans: B 15820. Half the mass of the atmosphere
is found in the first: A – 5 km B – 3 km C – 8 km D – 11 km Ref: all Ans: A
15849. What of the following is the most important constituent in the atmosphere
from a weather stand point? A – Hydrogen B – Water vapour C – Nitrogen D – Oxygen
Ref: all Ans: B 15850. The height and the temperature of the tropopause are
respectively in the order of: A – 16 km and -40oC over the poles B – 16 km and
-75oC over the equator C – 8 km and -40oC over the equator D – 8 km and -75oC over
the poles Ref: all Ans: B 16572. The troposphere is: A – deepest over the equator B
– deepest over the poles C – the same depth all over the earth D – shallowest over
the poles in summer Ref: all Ans: A 24289. In relation to the total weight of the
atmosphere, the weight of the atmosphere between mean sea level and a height of
5500m is: A – 25% B – 1% C – 50% D – 99% Ref: all Ans: C
24425. Which of the following statements concerning the tropopause is correct? A –
The temperature of the tropopause at the equator and at the poles is equal B – The
temperature remains constant above and below the tropopause C – The temperature of
the tropopause at the equator is higher than at the poles D – The temperature lapse
rate changes abruptly at the tropopause Ref: all Ans: D 24440. Which statement
concerning the tropopause is correct? A – The temperature at the tropopause is
approximately -80oC over the Poles and approximately -40oC over the equator B –
Above the tropopause no clear air turbulence occurs C – in the ICAO standard
atmosphere the tropopause lies higher over the Poles than over the equator D – The
layer just above the tropopause is absolutely stable Ref: all Ans: D

050-01-02 Temperature
8808. Which is true of the temperature at the tropopause? A – It is higher in polar
regions than in equatorial regions B – It is higher in equatorial regions than in
polar regions C – It is highest in middle latitudes D – There is no significant
difference with change of latitude Ref: all Ans: A 8810. Several physical processes
contribute to atmospheric warming. Which of the following contribute the most? A –
Absorption and evaporation B – Solar radiation and conduction C – Absorption and
vaporisation D – Convection and condensation Ref: all Ans: D
8833. An outside air temperature of -35oC is measured while cruising at FL 200.
What is the temperature deviation from the ISA at this level? A – 5oC colder than
ISA B – 10oC warmer than ISA C – 5oC warmer than ISA D – 10oC colder than ISA Ref:
all Ans: D 8856. A layer can be: A – unstable for unsaturated air and conditionally
unstable B – stable for saturated air and unstable for unsaturated air C – unstable
for unsaturated air and neutral for saturated air D – stable for unsaturated air
and unstable for saturated air Ref: all Ans: D 8858. At a certain position, the
temperature on the 300 hPa chart is -48oC; according to the tropopause chart, the
tropopause is at FL 330. What is the most likely temperature at FL 350? A - -56.5oC
B - -50oC C - -54oC D - -58oC Ref: all Ans: C 8864. Absolute instability exists
whenever the environmental lapse rate: A – exceeds the saturated adiabatic lapse
rate B – exceeds the dry adiabatic lapse rate C – is less than the saturated
adiabatic lapse rate D – is between the dry and saturated adiabatic lapse rate Ref:
all Ans: B
8887. A significant inversion at low height is a characteristic of: A – the passage
of cold front B – nocturnal radiation C – advection fog D – cumulus clouds Ref: all
Ans: B 8890. An inversion is a layer of air which is: A – absolutely unstable B –
absolutely stable C – conditionally unstable D – conditionally stable Ref: all Ans:
B 8898. An inversion is: A – an increase of temperature with height B – an increase
of pressure with height C – a decrease of pressure with height D – a decrease of
temperature with height Ref: all Ans: A 10024. When in the upper part of a layer
warm air is advected the A – stability increases in the layer B – stability
decreases in the layer C – wind will back with increasing height in the northern
hemisphere D – wind speed will always decrease with increasing height in the
northern hemisphere Ref: all Ans: A
10028. Advection is: A – the same as convection B – vertical motion of air C – the
same as subsidence D – horizontal motion of air Ref: all Ans: D 10038. The
environmental lapse rate in an actual atmosphere: A – has a fixed value of 2oC/1000
ft B – has a fixed value of 1oC/100m C – has a fixed value of 0.65oC/100m D –
varies with time Ref: all Ans: D 10066. In the lower part of the stratosphere the
temperature: A – decreases with altitude B – is almost constant C – increases with
altitude D – increases at first and decreases afterward Ref: all Ans: B 10068. An
isothermal layer is a layer of air in which the temperature: A – increases with
height at a constant rate B – increases with height C – decreases with height at a
constant rate D – remains constant with height Ref: all Ans: D
10074. Which of the following is a common result of subsidence? A – Clear air
turbulence at higher altitudes B – CB-clouds and thunderstorms over a large sea C –
Wide spread NS and AS clouds and intense precipitation D – An inversion over a
large area with haze, mist Ref: all Ans: D 10083. An air mass is called stable
when: A – the vertical motion of rising air tends to become weaker and disappears B
– the temperature in a given air mass decreases rapidly with height C – the
pressure in a given area is constant D – the environmental lapse rate is high, with
little vertical motion of air currents Ref: all Ans: A 10095. What is the technical
term for an increase in temperature with altitude? A – Subsidence B – Inversion C –
Adiabatic D – Advection Ref: all Ans: B 10111. The dry adiabatic lapse rate has a
value of: A – 0.5oC/100m B – 2oC/1000 ft C – 0.65oC/100m D – 1oC/100m Ref: all Ans:
D
10122. How would you characterise an air temperature of 15oC at the 700 hPa level
over western Europe? A – Within +/-5oC of ISA B – High C – Low D – 20oC below
standard Ref: all Ans: C 10128. In an air mass with no clouds the surface
temperature is 15oC and the temperature at 1000m/AGL is 13oC. This layer of air is:
A – unstable B – stable C – a layer of heavy turbulence D – conditionally unstable
Ref: all Ans: B 10130. The rate of decrease of temperature with height per 100m in
the International Standard Atmosphere is: A – 0.65oC B – 1oC C – 0.5oC D – variable
Ref: all Ans: A 10145. The radiation of the sun heats: A – the air in the
troposphere only directly if no clouds are present B – the air in the troposphere
directly C – the water vapour in the air of the troposphere D – the surface of the
earth, which heats the air in the troposphere Ref: all Ans: D
10742. From which of the following pieces of information can the stability of the
atmosphere be derived? A – Surface temperature B – Environmental lapse rate C – Dry
adiabatic lapse rate D – Pressure at the surface Ref: all Ans: B 10754. What
characteristic is associated with a temperature inversion? A – Stability B –
Instability C – Clear ice D – Area of active storms Ref: all Ans: A 10760.
Convective activity over land in mid-latitudes is greatest in: A – winter in the
afternoon B – winter during the night and early morning C – summer during the night
and early morning D – summer in the afternoon Ref: all Ans: D 10761. The dry
adiabatic lapse rate: A – has a constant fixed value B – is greater in summer than
in winter C – is greater during the night than during the day D – has a variable
value Ref: all Ans: A
10802. The temperature at FL 160 is -22oC. What will the temperature be at FL 90 if
the ICAO standard lapse rate is applied? A - -4oC B - -8oC C – 0oC D - +4oC Ref:
all Ans: B 10812. The temperature at FL 140 is -12oC. What will the temperature be
at FL 110 if the ICAO standard lapse rate is applied: A - -9oC B - -18oC C - -6oC D
- -15oC Ref: all Ans: C 10820. The temperature at FL 80 is +6oC. What will the
temperature be at FL 130 if the ICAO standard lapse rate is applied? A - +2oC B -
-6oC C – 0oC D - -4oC Ref: all Ans: D 10839. The diurnal variation in temperature
is largest when the sky is: A – clear and winds are strong B – clear and winds are
weak C – overcast and winds are weak D – overcast and winds are strong Ref: all
Ans: B
10861. On a clear sky, continental ground surface, wind calm, the minimum
temperature is reached approximately: A – at the moment the sun rises B – half an
hour before sunrise C – half an hour after sunrise D – one hour before sunrise Ref:
all Ans: C 10862. The temperature at FL 110 is -5oC. What will the temperature be
at FL 50 if the ICAO standard lapse rate is applied? A - -3oC B - +3oC C – 0oC D -
+7oC Ref: all Ans: D 10877. The 0o isotherm is forecast to be at FL 50. At what FL
would you expect a temperature of -6oC? A – FL 110 B – FL 20 C – FL 100 D – FL 80
Ref: all Ans: D 10879. A temperature of 15oC is recorded at an altitude of 500
metres above sea level. If the vertical temperature gradient is that of a standard
atmosphere, what will the temperature be at the summit of a mountain, 2500 metres
above sea level? A - +4oC B - +2oC C – 0oC D - -2oC Ref: all Ans: B
10887. The value of the saturated adiabatic lapse rate is closest to that of the
dry adiabatic lapse rate in: A – cumulus B – freezing fog C – stratus D – cirrus
Ref: all Ans: D 10894. Around Paris on January 3rd at 1800 UTC, the surface
temperature under shelter is 3oC. The sky is covered by 8 oktas of stratus. QNH is
1033 hPa. If the sky is covered all night, the minimum temperature of the night of
January 3rd to January 4th should be: A – slightly above +3oC B – significantly
below 0oC C – slightly below +3oC D – significantly above +3oC Ref: all Ans: C
15782. Which of the following is a common cause of ground or surface temperature
inversion? A – Terrestrial radiation on a clear night with no or very light winds B
– Warm air being lifted rapidly aloft, in the vicinity of mountainous terrain C –
The movement of colder air under warm air, or the movement of warm air over cold
air D – Heating of the air by subsidence Ref: all Ans: A 15810. How does
temperature vary with increasing altitude in the ICAO standard atmosphere below the
tropopause? A – increases B – At first it increases and higher up it decreases C –
Remains constant D – Decreases Ref: all Ans: D
15822. How would you characterise an air temperature of 30oC at the 300 hPa level
over western Europe? A – High B – Within +/- 5oC of ISA C – Low D – Very low Ref:
all Ans: A 15823. How would you characterise an air temperature of 55oC at the 200
hPa level over western Europe? A – High B – Low C – Very high D – Within +/- 5oC of
ISA Ref: all Ans: D 15851. An inversion is a layer of air in which the temperature:
A – increases with height more than 1oC/100m, B – decreases with height more than
1oC/100m C – increases with height D – remains constant with height Ref: all Ans: C
15858. A wide body takes off on a clear night in Dhahran, Saudi Arabia. Shortly
after take off the aircraft’s rate of climb drops to zero. This can be due to: A –
low relative humidity B – a very strong temperature inversion C – sand/dust in the
engines D – very pronounced downdrafts Ref: all Ans: B
15876. In still air the temperature decreases at an average of 1.2oC per 100m
increase in altitude. This temperature change is called: A – environmental lapse
rate B – saturated adiabatic lapse rate C – dry adiabatic lapse rate D – normal
lapse rate Ref: all Ans: A 16340. In the disturbed temperate regions: A – winters
are generally mild B – the weather is mainly governed by travelling frontal
depressions C – the wet season is normally from May to September D – the surface
winds are moderate westerlies Ref: all Ans: B 16348. Horizontal differences in the
mean temperature of a layer are caused by: A – insulation B – advection C –
differential heating of the earth’s surface D – change of air mass Ref: all Ans: C
16351. Which of the following statements concerning the thermal wind component
(TWC) is true? A – TWC decreases as the horizontal mean temperature gradient
increases B – TWC increases as the horizontal mean temperature gradient increases C
– the greater the TWC, the greater the reduction in the upper wind D – the greater
the TWC, the greater the surface wind Ref: all Ans: B
16555. Which one of the following describes normal conditions? A – Temperature
decreases with height in the troposphere B – Temperature increases with height in
the troposphere C – Temperature decreases with height in the stratosphere D –
Temperature decreases at a similar rate in the troposphere as in the stratosphere
Ref: all Ans: A 16556. For international aviation meteorological purposes,
temperature is measured in degrees: A – Fahrenheit B – Celsius C – Absolute D –
Kelvin Ref: all Ans: B 16571. If the depth of the troposphere increases, the
temperature at the tropopause must: A – decrease B – stay the same C – increase D –
impossible to say Ref: all Ans: A 16594. Air at T = +16oC and DP = +4oC is forced
from sea level over a 10,000 ft mountain range and descends back to sea level on
the other side. If the leeward condensation level is observed to be 8,000 ft what
will be the final temperature? A – 18oC B – 20oC C – 22oC D – 24oC Ref: all Ans: C
24216. A parcel of unsaturated air is forced to rise through an isothermal layer.
So long as it remains unsaturated, the temperature of the parcel: A – decreases
0.65oC per 100m B – remains constant C – decreases 1oC per 100m D – becomes equal
to the temperature of the isothermal layer Ref: all Ans: C 24221. Absolute
instability in the atmosphere will occur when the environmental lapse rate is A –
greater than both saturated adiabatic lapse rate and dry adiabatic lapse rate B –
less than saturated adiabatic lapse rate C – less than both saturated adiabatic
lapse rate and dry adiabatic lapse rate D – greater than saturated adiabatic lapse
rate but less than dry adiabatic lapse rate Ref: all Ans: A 24222. According to ISA
the temperature in the lower part of the stratosphere: A – decreases with altitude
B – is almost constant C – increases with altitude D – increases at first and
decreases afterward Ref: all Ans: B 24269. For both saturated and unsaturated air
instability will occur when the A – environmental lapse rate is greater than both
dry adiabatic lapse rate and saturated adiabatic lapse rate B – environmental lapse
rate is greater than saturated adiabatic lapse rate but less than dry adiabatic
lapse rate C – environmental lapse rate is less than both dry adiabatic lapse rate
and saturated adiabatic lapse rate D – dry adiabatic lapse rate is less than
saturated adiabatic lapse rate but greater than environmental lapse rate Ref: all
Ans: A
24291. In the lower levels of the atmosphere when the environmental lapse rate is
greater than saturated adiabatic lapse rate but les than dry adiabatic lapse rate –
the air mass is described as being A – conditionally unstable B – stable C –
unstable D – absolutely unstable Ref: all Ans: A 24353. The temperature lapse rate
of the standard atmosphere in the troposphere is: A – 2.5oC/1000 ft B – 3oC/1000 ft
C – 6.5oC/1000 ft D – 2oC/1000 ft Ref: all Ans: D 24386. What is, approximately,
the temperature at 20000 ft in the ICAO Standard Atmosphere? A - -20oC B - -15oC C
- -25oC D - -30oC Ref: all Ans: C

050-01-03 Atmospheric pressure


10035. What positions are connected by isobars on the surface weather chart? A –
Positions with the same air pressure at a given level B – Positions with the same
temperature at a given level C – Positions with the same wind velocity at a given
level D – Positions with the same relative pressure heights Ref: all Ans: A
10059. In the troposphere the decrease of pressure per 100m increase in height: A –
is greater at higher levels than at lower levels B – remains constant at all levels
C – is smaller at higher levels than at lower levels D – is in the order of 27 hPa
near MSL Ref: all Ans: C 10085. An isohypse (contour): A – indicates the altitude
of the zero degree isotherm B – is the longest slope line of a frontal surface C –
is the limit between two air masses of different temperature D – indicates the true
altitude of a pressure level Ref: all Ans: D 10137. The station pressure used in
surface weather charts is: A – QNE B – QFE C – QNH D – QFF Ref: all Ans: D 10807.
Which of the following is true concerning atmospheric pressure? A – It is higher in
winter than in summer B – It decreases with height C – It is higher at night than
during the day D – It always decreases with height at a rate of 1 hPa per 8m Ref:
all Ans: B
10822. Isobars on a surface chart are lines of equal: A – QFE B – QFF C – QNE D –
QNH Ref: all Ans: B 15780. What is approximate vertical interval which is equal to
a pressure change of 1 hPa at an altitude of 5,500m? A – 15m (50 ft) B – 8m (27 ft)
C – 32m (105 ft) D – 64m (210 ft) Ref: all Ans: A 15809. The isobars drawn on a
surface weather chart represent lines of equal pressure: A – at height of
observatory B – at a determined density altitude C – reduced to sea level D – at
flight level Ref: all Ans: C 15837. Between which latitudes are you most likely to
find the region of travelling low pressure systems? A – 25o – 35o B – 10o – 15o C –
55o – 75o D – 35o – 55o Ref: all Ans: C
15877. Assume that an aircraft is flying in the northern hemisphere at the 500 hPa
pressure surface on a heading of 270 degrees. Which of the following statements is
correct? A – If in this pressure surface the wind comes from the direction 360
degrees, then true altitude is increasing B – If in this pressure surface the wind
comes from the direction 180 degrees, then true altitude is increasing C – If in
this pressure surface the wind comes from the direction 20 degrees, then true
altitude is increasing D – If in this pressure surface the wind comes from the
direction 090 degrees, then true altitude is increasing Ref: all Ans: A 16349. At
altitude, the atmospheric pressure in a column of warm air is likely to be: A –
lower than at the same height in a column of cold air B – higher than at the same
height in a column of cold air C – the same irrespective of the temperature D –
depends on the relative humidity Ref: all Ans: B 16357. Which of the following
statements is true? A – High contour values are equivalent to high pressure B – Low
contour values are equivalent to high pressures C – High contour values are
equivalent to low pressure D – There is no direct relationship between contour
values and pressure Ref: all Ans: A 16358. When flying towards high contour values
an aircraft will experience: A – headwind B – tailwind C – port drift D – starboard
drift Ref: all Ans: C
16363. Contour heights are: A – true heights AGL B – true heights AMSL C –
indicated heights above 1013.25 mb D – do not indicate heights at all Ref: all Ans:
B 16364. When flying from high to low contour values, which of the following is
incorrect? A – the true height of the aircraft will be falling B – the pressure
altimeter will indicate a constant value C – the indicated height of the aircraft
will be constant D – the indicated height of the aircraft will only be true if
1013.25 mb is set Ref: all Ans: D 16436. You are making a long distance flight and
have chosen a suitable cruising altitude for the whole flight. Towards the end of
your flight, you have descended. What may be the reason for this? A – you are
approaching a region of high pressure B – you are approaching a region of low
pressure C – standard pressure has dropped D – temperature has increased Ref: all
Ans: B 16438. Select the correct statement regarding the wind direction in
connection with the high and low pressure systems in the Northern Hemisphere: A –
the winds blow counter clockwise around a high and clockwise in a low B – the winds
blow clockwise in both highs and lows C – the winds blow clockwise in a high and
counter clockwise in a low D – the winds blow counter clockwise in both highs and
lows Ref: all Ans: C
16439. If you fly across the isobars towards a region of high pressure in the
Northern Hemisphere you will: A – drift to the right B – drift to the left C –
experience no drift but experience a headwind D – experience no drift but
experience a tailwind Ref: all Ans: B 16511. According to definition, flight levels
are surfaces with constant air pressure determined from a certain pressure value.
Which is this value? A – 1013.25 hPa B – 1025.13 hPa C – Actual QFE D – Actual QNH
Ref: all Ans: A 16517. If you have a column of air limited by two isobaric surfaces
at a pressure difference of 100 hPa, the distance between the pressure surfaces
will change if mean temperature and mean pressure of the column of air change. In
which of the following alternatives will the change of temperature and pressure
interact to shorten the distance as much as possible? A – The temperature increases
and pressure increases B – The temperature decreases and pressure increases C – The
temperature increases and pressure decreases D – The temperature decreases and
pressure decreases Ref: all Ans: B 16534. Lines joining points of equal pressure
are known as: A – Isotherms B – Isopleths C – Isobars D – Isotachs Ref: all Ans: C
16544. A pressure difference of 10 hPa close to the ground corresponds to a height
difference of: A – about 50m B – about 150m C – about 300ft D – about 30ft Ref: all
Ans: C 16587. A rising parcel of air which has no heat entering or leaving it,
will: A – reduce in pressure, rise in temperature, decrease in density B – maintain
volume, decrease in density, reduce in pressure C – maintain pressure, reduce in
density, increase in volume D – reduce in pressure, decrease in density, increase
in volume Ref: all Ans: D 24345. The QFF at an airfield located 400 metres above
sea level is 1016 hPa. The air temperature is 10oC higher than a standard
atmosphere. What is the QNH? A – 1016 hPa B – More than 1016 hPa C – Less than 1016
hPa D – It is not possible to give a definitive answer Ref: all Ans: B 24431. Which
of the following statements is correct? A – Cumulus clouds and a good viability are
normally observed in a warm sector in winter B – Cumulus clouds and a good
visibility are normally observed in a warm sector in autumn C – Normally
atmospheric pressure stops falling rapidly behind a warm front, the air temperature
rises D – At warm fronts thunderstorms are often observed Ref: all Ans: C
050-01-04 Atmospheric density
10146. Under what condition does pressure altitude have the same value as density
altitude? A – When the altimeter has no position error B – At sea level when the
temperature is 0oC C – At standard temperature D – When the altimeter setting is
1013.2 hPa Ref: all Ans: C 15779. At FL 180, the air temperature is -35oC. The air
density at this level is: A – unable to be determined without knowing the QNH B –
greater than the density of the ISA atmosphere at FL 180 C – less than the density
of the ISA atmosphere at FL 180 D – equal to the density of the ISA atmosphere at
FL 180 Ref: all Ans: B 16525. What happens if density altitude is 3000 ft at an
airport whose elevation is 1000 ft? A – Take off and landing performance will be
unaffected B – The altimeter will indicate 3000 ft when the aircraft is on the
ground C – Take off and landing performance will be about the same as for an
airport with an elevation of 3000 ft D – Indicated speed at 50 kt on take off and
landing will be higher than in a standard atmosphere Ref: all Ans: C 24454. With
all other quantities being constant, the density of the atmosphere increases with
increasing: A – relative humidity B – air pressure C – stability D – temperature
Ref: all
Ans: B

050-01-05 International Standard Atmosphere (ISA)


8840. The lowest assumed temperature in the International Standard Atmosphere (ISA)
is: A - -44.7oC B - -273oC C - -58.5oC D - -100oC Ref: all Ans: C 8850. A 500 hPa
pressure level can vary in height. In temperate regions which of the following
average heights is applicable? A – FL 180 B – FL 160 C – FL 100 D – FL 390 Ref: all
Ans: A 8869. A 700 hPa pressure level can vary in height. In temperate regions
which of the following average heights is applicable? A – FL 100 B – FL 180 C – FL
300 D – FL 390 Ref: all Ans: A 8882. The temperature at 10000 ft in the
International Standard Atmosphere is: A - -20oC B – 0oC C - -5oC D - -35oC Ref: all
Ans: C 8884. If you are flying at FL 120 and the outside temperature is -2oC, at
what altitude will the freezing level be? A – FL 110 B – FL 130 C – FL 150 D – FL
90 Ref: all Ans: A 10073. A 850 hPa pressure level can vary in height. In temperate
regions which of the following average heights is applicable? A – FL 300 B – FL 100
C – FL 50 D – FL 390 Ref: all Ans: C 10120. If you are flying at FL 300 in an air
mass that is 15oC warmer than a standard atmosphere, what is the outside
temperature likely to be? A - -30oC B - -45oC C - -60oC D - -15oC Ref: all Ans: A
10125. In the International Standard Atmosphere the decrease in temperature with
height below 11 km is: A – 1oC per 100m B – 0.65oC per 100m C – 0.5oC per 100m D –
0.6oC per 100m Ref: all Ans: B
10127. What is the vertical temperature lapse rate, up to 11 km, in the standard
ICAO atmosphere? A – 2oC per 1000m B – 4.5oC per 1000m C – 3oC per 1000m D – 6.5oC
per 1000m Ref: all Ans: D 10136. A 200 hPa pressure altitude level can vary in
height. In temperate regions which of the following average heights is applicable?
A – FL 50 B – FL 300 C – FL 100 D – FL 390 Ref: all Ans: D 10736. A 300 hPa
pressure level can vary in height. In temperate regions which of the following
average heights is applicable? A – FL 100 B – FL 390 C – FL 300 D – FL 50 Ref: all
Ans: C 19763. If you are flying at FL 100 in an air mass that is 10oC warmer than a
standard atmosphere, what is the outside temperature likely to be? A - +15oC B -
+5oC C - -10oC D - -15oC Ref: all Ans: B
10888. Which statement is correct regarding the International Standard Atmosphere?
A – At MSL temperature is 15oC and pressure is 1013.25 hPa B – At MSL temperature
is 15oC and the decrease in temperature with height is 1oC per 100m C – At MSL
temperature is 10oC and the decrease in temperature with height is 1oC Ref: all
Ref: All Ans: A 24253. Between mean sea level and a height of 20 km, the lowest
temperature in the ICAO Standard Atmosphere (ISA) is: A - -44.7oC B - -273oC C -
-56.5oC D - -100oC Ref: all Ans: C 24336. The ICAO Standard Atmosphere (ISA)
assumes that temperature will reduce at the rate of: A - 2oC per 1000 ft up to
65617 ft after which it will remain constant to 104987 ft B – 1.98oC per 1000 ft up
to 36090 ft and will then rise at 0.3oC per 1000 ft up to 65617 ft when it will
remain constant C – 1,98oC per 1000 ft up to 36090 ft after which it remains
constant to 65617 ft D – 2oC per 1000 ft up to 36090 ft and will then increase at
0.3oC per 1000 ft up to 65617 ft Ref: all Ans: C
050-01-06 Altimetry
8807. In Geneva, the local QNH is 994 hPa. The elevation of Geneva is 1411 ft. The
QFE adjustment in Geneva is: A – 942 hPa B – 967 hPa C – 961 hPa D – 948 hPa Ref:
all Ans: A 8811. An aircraft is descending to land under IFR. If the local QNH is
1009 hPa, what will happen to the altitude reading when the altimeter is reset at
the transition level? A – It will increase B – It will decrease C – It will remain
the same D – It will not be affected Ref: all Ans: B 8815. The QNH level at an
airfield located 200 metres above sea level is 1009 hPa. The air temperature is
10oC lower than a standard atmosphere. What is the QFF? A – Less than 1009 hPa B –
1009 hPa C – More than 1009 hPa D – It is not possible to give a definitive answer
Ref: all Ans: C
8818. An aircraft is flying at FL 80. The local QNH is 1000 hPa. After the second
altimeter has been adjusted to the local QNH, the reading will be approximately: A
– 7650 ft B – 8600 ft C – 8350 ft D – 8000 ft Ref: all Ans: A 8829. The QFF at an
airfield in California located 69 metres below sea level is 1030 hPa. The air
temperature is 10oC lower than a standard atmosphere. What is the QNH? A – It is
not possible to give a definitive answer B – Less than 1030 hPa C – 1030 hPa D –
More than 1030 hPa Ref: all Ans: D 8834. An aircraft lands at an airport (airport
elevation 1240 ft, QNH 1008 hPa). The altimeter is set to 1013 hPa. The altimeter
will indicate: A – 1200 ft B – 1375 ft C – 1105 ft D – 1280 ft Ref: all Ans: B
8836. The following temperatures have been observed over a station at 1200 UTC.
Assume the station is at MSL. Height in feet. Temperature in degrees C. 20000 (-12)
18000 (-11), 16000 (-10), 14000 (-10), 12000 (-6), 10000 (-2), 8000 (+2), 6000
(+6), 4000 (+12), 2000 (+15), surface (+15). A – The layer between 16000 and 18000
ft is absolutely unstable B – The height of the freezing level over the station is
approximately 12000 ft C – The temperature at 10000 ft is in agreement with the
temperature in the International Standard Atmosphere D – Assuming that the MSL
pressure is 1013.25 hPa the true altitude of an aircraft would actually be higher
than the indicated altitude Ref: all Ans: D 8842. An aircraft lands at an airport
(airport elevation 540 ft, QNH 993 hPa) with the altimeter set to 1013 hPa. What
will it indicate? A – 1080 ft B – 700 ft C – 380 ft D – 0 ft Ref: all Ans: A 8843.
You are flying at FL 130, and your true altitude is 12000 ft. What is the
temperature deviation from that of the standard atmosphere at FL 130 (QNH 1013.2
hPa)? A – ISA + 12oC B – ISA +/- 0oC C – ISA +20oC D – ISA -20oC Ref: all Ans: D
8845. The pressure altitude is equal to the true altitude if: A – the outside air
temperature is standard for that height B – standard atmospheric conditions occur C
– the air pressure is 1013.25 hPa at the surface D – the indicated altitude is
equal to the pressure altitude Ref: all Ans: B 8861. Which of the following
conditions would cause the altimeter to indicate a lower altitude than that
actually flown? A – Pressure altitude the same as indicated altitude B –
Atmospheric pressure lower than standard C – Air temperature higher than standard D
– Air temperature lower than standard Ref: all Ans: C 8863. What pressure is
defined as QFE? A – The pressure reduced to sea level using actual temperatures B –
The pressure of the altimeter C – The pressure at field elevation D – The pressure
reduced to sea level using ISA temperatures Ref: all Ans: C 8867. During the climb
after takeoff, the altimeter setting is adjusted at the transiktion altitude. If
the local QNH is 1023 hPa, what will happen to the altimeter reading during the
resetting procedure? A – It is not possible to give a definitive answer B – It will
increase C – It will remain the same D – It will decrease Ref: all Ans: D
8870. An aircraft is flying at FL 180 on the northern hemisphere with a crosswind
from the left. Which of the following is correct concerning its true altitude?= A –
It remains constant B – It increases C – It decreases D – Without knowing
temperatures at FL 180 this question cannot be answered Ref: all Ans: C 8875. In
order to calculate QFE from QNH, which of the following must be known? A –
Elevation and the temperature at the airfield B – Temperature at the airfield C –
Elevation of the airfield and the temperature at MSL D – Elevation of the airfield
Ref: all Ans: D 8879. If the QNH at Locarno (200 metres above sea level) is 1025
hPa, what is the approximate QFE? A – 1005 hPa B – 995 hPa C – 1000 hPa D – 1025
hPa Ref: all Ans: C 10030. You are flying at FL 200. Outside air temperature is
-40oC and the pressure at sea level is 1033 hPa. What is the true altitude? A –
20660 feet B – 19310 feet C – 21740 feet D – 18260 feet Ref: all
Ans: B

10046. The QNH at an airfield located 6 metres above sea level is 1022 hPa. The air
temperature is not available. What is the QFF? A – It is not possible to give a
definitive answer B – Less than 1022 hPa C – More than 1022 hPa D – 1022 hPa Ref:
all Ans: D 10049. Which of the following conditions gives the highest value of the
QNH? A – QFE = 1003 hPa, elevation = 1200 ft (366m) B – QFE = 1000 hPa, elevation =
1200 ft (366m) C – QFE = 995 hPa, elevation = 1600 ft (488m) D – QFE = 995 hPa,
elevation = 1200 ft (366m) Ref: all Ans: C 10052. Which statement is true? A – QNH
can be 1013.25 only for a station at MSL B – QNH cannot be 1013.25 hPa C – QNH is
lower than 1013.25 at any time D – QNH can be lower as well as higher than 1013.5
hPa Ref: all Ans: D 10079. When the subscale is set to the QNH of an airfield the
pressure altimeter indicates: A – zero while landing B – elevation while landing C
– elevation while landing only if conditions are as in the International Standard
Atmosphere D – zero while landing only if conditions are as in the International
Standard Atmosphere Ref: all
Ans: B

10087. A vertical spacing of 1000 ft, is the standard required separation between
two FL. Under conditions of cold air advection (ISA -15oC), what would the true
vertical separation be? A – More than 1000 ft B – It remains 1000 ft C – Less than
1000 ft D – Without QNH information, it cannot be determined Ref: AIR: atpl, ir;
HELI: atpl, ir Ans: C 10090. The QNH at an airfield in California located 69 metres
below sea level is 1018 hPa. The air temperature is 10oC higher than a standard
atmosphere. What is the QFF? A – More than 1018 hPa B – Less than 1018 hPa C – 1018
hPa D – It is not possible to give a definitive answer Ref: all Ans: A 10091. You
are planning to fly across a mountain range. The chart recommends a minimum
altitude of 12000 feet above mean sea level. The air mass you will be flying
through is an average 10oC warmer than ISA. Your altimeter is set to 1023 hPa (QNH
of a nearby airport at nearly sea level). What altitude will the altimeter show
when you have reached the recommended minimum altitude? A – 12210 feet B – 11520
feet C – 11250 feet D – 11790 feet Ref: AIR: atpl, ir; HELI: atpl, ir Ans: B
10101. You intend to overfly a mountain range. The recommended minimum flight
altitude is, according to the aviation chart, 15000 ft/AMSL. The air mass that you
will fly through is on average 15oC warmer than the standard atmosphere. The
altimeter is set to QNH (1023 hPa). At what altimeter reading will you effectively
be at the recommended minimum flight altitude? A – 15900 ft B – 13830 ft C – 14370
ft D – 14100 ft Ref: AIR: atpl, ir; HELI: atpl, ir Ans: D 10113. QNH is defined as:
A – The pressure at MSL obtained using the standard atmosphere B – The pressure at
MSL, obtained using the actual conditions C – QFE reduced to MSL using the actual
conditions D – QFE reduced to MSL using the standard atmosphere Ref: all Ans: D
10124. At which pressure and temperature conditions may you safely assume that the
minimum usable flight level at least lies at the same height, as the minimum safe
altitude? A – In a cold low pressure region B – At a temperature greater than or
equal to that of the ISA and where the QNH is greater than or equal to 1013 hPa C –
At a temperature less than or equal to that of the ISA and where the QNH is less
than 1013 hPa D – In a warm high pressure region Ref: AIR: atpl, ir; HELI: atpl, ir
Ans: B
10140. The QNH is equal to the QFE if: A – T actual < T standard B – T actual = T
standard C – T actual > T standard D – the elevation = 0 Ref: all Ans: D 10147. The
QNH of an airport at sea level is 983 hPa and the temperature deviation from ISA is
-15oC below FL 100. What is the true altitude of FL 100? A – 9740 ft B – 10160 ft C
– 8640 ft D – 11460 ft Ref: all Ans: C 10162. The barometric compensator of an
altimeter is locked on reference 1013.2 hPa. The aircraft has to land on a point
with an elevation of 290 feet where the QNH is 1023 hPa. Assuming that 1 hPa
corresponds to 27 ft, the reading on the altimeter on the ground will be: A – 20 ft
B – 11 ft C - -10 ft D – 560 ft Ref: AIR: atpl, ir; HELI: atpl, ir Ans: A 10165. An
aircraft is flying over the sea at FL 90; the true altitude is 9100 feet; local QNH
is unknown. What assumption, if any, can be made about the air mass in which the
aircraft is flying? A – It is colder than ISA B – There is insufficient information
to make any assumption C – It is warmer than ISA D – Its average temperature is the
same as ISA
Ref: all Ans: B 10735. An aircraft is flying through the Alps on a very cold
winters day. The regional QNH is 1013 hPa. During the flight, you circle around a
mountain at an altitude of its summit. What reading will the aneroid altimeter
give, compared to the elevation of the summit? A – The same altitude as the
elevation of the summit B – A lower altitude than the elevation of the summit C – A
higher altitude than the elevation of the summit D – There is insufficient
information to come to a conclusion Ref: AIR: atpl, ir; HELI: atpl, ir Ans: C
10740. An aircraft is flying through the Alps on a warm summers day. The weather is
fine, and there is a high pressure system in the area. During the flight, a
mountain is passed at an altitude of its summit. What reading will the aneroid
altimeter give, compared to the summit’s elevation? A – There is insufficient
information to come to a conclusion B – A higher altitude than the elevation of the
summit C – The same altitude as the elevation of the summit D – A lower altitude
than the elevation of the summit Ref: AIR: atpl, ir; HELI: atpl, ir Ans: D 10798.
The QFF at an airfield located 400 metres above sea level is 1016 hPa. The air
temperature is 10oC lower than a standard atmosphere. What is the QNH? A – It is
not possible to give a definitive answer B – More than 1016 hPa C – 1016 hPa D –
Less than 1016 hPa Ref: all Ans: D
10799. In order to reduce QFE to QNH, which of the following item(s) must be known?
A – Elevation of the airfield and the temperature at the airfield B – Temperature
at the airfield C – Elevation of the airfield and the temperature at MSL D –
Elevation of the airfield Ref: all Ans: D 10803. After landing at an aerodrome
(aerodrome elevation 1715 ft), the altimeter indicates an altitude of 1310 ft. The
altimeter is set to the pressure value of 1013 hPa. What is the QNH at this
aerodrome? A – 1015 hPa B – 1028 hPa C – 1013 hPa D – 998 hPa Ref: all Ans: B
10808. What information is required to convert a minimum safe altitude into a
lowest usable flight level? A – Lowest value of QNH and the highest negative
temperature deviation from ISA B – Highest value of QNH and the highest negative
temperature deviation from ISA C – Highest value of QNH and the highest positive
temperature deviation from ISA D – Lowest value of QNH and the lowest negative
temperature deviation from ISA Ref: AIR: atpl, ir; HELI: atpl, ir Ans: A
10813. An aircraft is flying over the sea at FL 100, with a true altitude of 10000
feet; local QNH is 1003 hPa. What assumption, if any, can be made about the air
mass in which the aircraft is flying? A – There is insufficient information to come
to any conclusion B – Its average temperature is about ISA C – It is colder than
ISA D – It is warmer than ISA Ref: all Ans: D 10816. Which weather condition lowers
true altitude as compared to pressure altitude to a position where flight over
mountains could be dangerous? A – Cold high B – Warm depression C – Cold low D –
Warm high Ref: all Ans: C 10830. An aircraft flying at FL 100 from Marseille (QNH
1012 hPa) to Palma de Mallorca (QNH 1006 hPa) experiences no change to true
altitude. The reason for this is that: A – the altimeters are erroneous, and need
to be tested B – the air at Palma de Mallorca is colder than that at Marseille C –
the air at Palma de Mallorca is warmer than that at Marseille D – one of the two
QNH values may be incorrect Ref: all Ans: C 10835. During a flight over the sea at
FL 135, the true altitude is 13500 feet; local QNH is 1019 hPa. What information,
if any, can be gained about the air mass in which the aircraft is flying? A – Its
average temperature is the same as ISA B – It is colder than ISA C – It is warmer
than ISA D – There is insufficient information to make any assumption
Ref: all Ans: B 10836. If atmospheric conditions exist such that the temperature
deviation is ISA 10oC in the lower troposphere up to 18000 ft, what is the actual
layer thickness between FL 60 and FL 120? A – 6240 ft B – 6000 ft C – 5900 ft D –
5760 ft Ref: AIR: atpl, ir; HELI: atpl, ir Ans: A 10853. For a given airfield the
QFE is 980 hPa and the QNH is 1000 hPa. The approximate elevation of the airfield
is: A – 120 metres B – 600 metres C – 540 metres D – 160 metres Ref: all Ans: D
10858. If the QFE at Locarnho (200 metres above sea level) is 980 hPa, what is the
approximate QNH? A – 1015 hPa B – 1000 hPa C – 1005 hPa D – 1010 hPa Ref: all Ans:
C 10859. If the QFE at Locarno (200 metres above sea level) is 1000 hPa, what is
the approximate QNH? A – 985 hPa B – 1025 hPa C – 990 hPa D – 1035 hPa Ref: all
Ans: B 10864. An altimeter adjusted to 1013 hPa indicates an altitude of 3600 ft.
Should this altimeter be adjusted to the local QNH value of 991 hPa, the altitude
indicated would be: A – 2922 ft B – 3006 ft C – 4278 ft D – 4194 ft Ref: all Ans: B
10865. You are flying at FL 160. Outside air temperature is -27oC and the pressure
at sea level is 1003 hPa. What is the true altitude? A – 15630 feet B – 15100 feet
C – 16370 feet D – 16910 feet Ref: all Ans: B 10868. During a flight over the sea
at FL 100 from Marseille (QNH 1012 hPa) to Palma de Mallorca (QNH 1012 hPa), the
true altitude is constantly increasing. What action, if any, should be taken? A –
None, the reason for the change is that the air around Palma is warmer than the air
around Marseille B – Have your altimeter checked, because its readings are
obviously wrong C – Re-check the QNH because one of the QNH values must be wrong D
– Compensate by heading further to the left Ref: AIR: atpl, ir; HELI: atpl, ir Ans:
A
10873. During a flight at FL 100 from Marseille (QNH 1012 hPa) to Palma de Mallorca
(QNH 1015 hPa), an aircraft remains at a constant true altitude. The reason for
this is that: A – the air at Marseille is colder than that at Palma de Mallorca B –
the altimeters are erroneous, and need to be tested C – the air at Marseille is
warmer than that at Palma de Mallorca D – one of the two QNH values may be
incorrect Ref: AIR: atpl, ir; HELI: atpl, ir Ans: C 10876. During a flight over the
sea at FL 100 from Marseille (QNH 1016 hPa) to Palma de Mallorca (QNH 1016 hPa),
the true altitude is constantly decreasing. What is the probable reason for this? A
– One of the QNH values must be wrong B – The air at Marseille is warmer than that
at Palma de Mallorca C – The altimeter is faulty D – The aircraft is being blown
off track to the left Ref: AIR: atpl, ir; HELI: atpl, ir Ans: B 10881. You plan a
flight over a mountain range at a true altitude of 15000 ft/AMSL. The air is on an
average 15oC colder than ISA, the pressure at sea level is 1003 hPa. What
indication must the altimeter (setting 1013.2 hPa) read? A – 15690 ft B – 16230 ft
C – 14370 ft D – 13830 ft Ref: AIR: atpl, ir; HELI: atpl, ir Ans: B 15812. What is
the relationship, if any, between QFE and QNH at an airport situated 50 ft below
sea level? A – QFE equals QNH B – No clear relationship exists C – QFE is greater
than QNH D – QFE is smaller than QNH
Ref: all Ans: C 15824. The QNH at an airfield located 200 metres above sea level is
1022 hPa. The air temperature is not available. What is the QFF? A – Less than 1022
hPa B – It is not possible to give a definitive answer C – More than 1022 hPa D –
1022 hPa Ref: all Ans: B 15825. If the QNH at Locarno (200 metres above sea level)
is 1016 hPa, what is the approximate QFE? (Assume 1 hPa = 8m) A – 1005 hPa B – 990
hPa C – 995 hPa D – 1000 hPa Ref: all Ans: B 15826. An aircraft is flying over the
sea at FL 120, with a true altitude of 12000 feet, local QNHY is 1013 hPa. What
assumption, if any, can be made about the air mass in which the aircraft is flying?
A – It is warmer than ISA B – There is insufficient information to come to any
conclusion C – Its average temperature is the same as ISA D – It is colder than ISA
Ref: all Ans: C 15827. During the climb after takeoff, the altimeter setting is
adjusted at the transition altitude. If the local QNH is 966 hPa, what will happen
to the altimeter reading during the resetting procedure? A – It will decrease B –
It will remain the same C – It is not possible to give a definitive answer D – It
will increase
Ref: all Ans: D 15828. (Refer to figure 050-14) An aircraft is flying from Point A
to Point B on the upper level contour chart. The altimeter setting is 1013.2 hPa.
Which of these statements is correct? A – Wind speed at A and at B is the same B –
The true altitude will be higher at A than at B C – The true altitude will be
higher at B than at A D – Wind speed at A is higher than at B Ref: all Ans: B
15836. After landing at an aerodrome (QNH 993 hPa) it is noticed that the altimeter
is still set to 1013.2 hPa and that it reads 1200 feet. What is the elevation of
the aerodrome above mean sea level? A – 2280 feet B – 660 feet C – 1200 feet D –
1740 feet Ref: all Ans: B 15852. Which of the following statements is true? A – QNH
is always lower than QFE B – QNH is always higher than QFE C – QNH is always equal
to QFE D – QNH can be equal to QFE Ref: all Ans: D 15857. You must make an
emergency landing at sea. The QNH of a field on a nearby island with an elevation
of 4000 ft is 1025 hPa and the temperature is -20C. What is your pressure altimeter
reading when landing if 1025 hPa is set in the subscale? A – 4000 ft B – Less than
0 ft C – 0 ft D – More than 0 ft, but less than 4000 ft
Ref: all Ans: B 15889. Before landing, an altimeter set to QFE indicates: A – the
height of the aircraft’s wheels above the runway B – in standard atmosphere the
height of the aircraft above the official airport elevation C – the flight level D
– the aircraft’s altitude above the mean sea level Ref: all Ans: B 16504. Suppose
that you are flying at FL 40 and that you reset the altimeter to reference pressure
993 hPa. The altimeter then indicates: A – about 3,400 ft B – about 3,700 ft C –
about 4,300 ft D – about 4,600 ft Ref: all Ans: A 16505. In which of the following
cases will the altimeter always indicate the airport elevation when landing? A – If
QFF of the airport is set as reference B – If QFE of the airport is set as
reference C – If QNH of the airport is set as reference D – At standard setting
Ref: all Ans: C 16506. A pressure altimeter can indicate different altitudes
depending on the setting. Regardless of setting it can generally be said that it
indicates: A – altitude above mean sea level at +15oC B – altitude in the standard
atmosphere at +15oC C – altitude corresponding to difference between reference
pressure and the pressure where the instrument is D – height above terrain
Ref: all Ans: C 16509. If an aircraft, without changing altimeter reference, flies
so that the altimeter all the time indicates the same altitude, this always means
that : A – the actual height above the mean ground level is unchanged B – the air
pressure around the aircraft is unchanged C – the actual altitude above sea level
is unchanged D – the air pressure at sea level is unchanged Ref: all Ans: B 16510.
At an airport 1700 ft above sea level the temperature reading is +10oC from the
barometer readings made at the same time QFE, QFF and QNH are computed. Which of
the following statements concerning air pressure is correct? A – QFF is higher than
QNH B – QFF is equal to QNH C – QFF is equal to QFE D – QNH is equal to QFE Ref:
all Ans: A 16512. When landing at an airport you have correctly set QNH = 1023 hPa
as reference pressure on your altimeter. The altimeter indicates 1200 ft after
landing. Suppose that you change the pressure reference to standard setting (1013
hPA) what will your altimeter indicate? A – 0 ft B – 900 ft C – 1200 ft D – 1500 ft
Ref: all Ans: B
16514. What will the altimeter indicate if the actual altitude is to be 10000 ft
when the pressure altitude is 10000 ft at OAT = -30oC? A – 9000 ft B – 10,000 ft C
– 11,000 ft D – 12,000 ft Ref: all Ans: C 16515. A barometric altimeter always
indicates: A – correct altitude to the terrain below B – altitude to the set
altitude reference C – standard altitude D – the dynamic pressure Ref: all Ans: B
16516. Suppose that you want your altimeter on landing to indicate airport
elevation which of the following pressures will you use as reference? A – Standard
(1013.25) B – QFE C – QFF D – QNH Ref: all Ans: D 16520. A pressure altimeter
indicates: A – actual altitude above MSL B – actual altitude above the terrain
below C – the distance between two isobaric surfaces in the actual atmosphere D –
the distance between two isobaric surfaces in the standard atmosphere Ref: all Ans:
D
16521. You are flying at a constant altitude according to your altimeter. Your
altimeter setting is unchanged. Which of the following interact to give a minimum a
true altitude? 1) 2) 3) 4) Flying from an area of low pressure to that of an area
of high pressure Flying from an area of high pressure to that of an area of low
pressure Flying from a warm air mass to a cold air mass Flying from a cold air mass
to a warm air mass

A – 1, 3 B – 2, 4 C – 1, 4 D – 2, 3 Ref: all Ans: D 16522. You are flying over an


airport at an indicated altitude of 5600 ft. Airport elevation = 2785 ft. The
altimeter is set at the correct aerodrome QNH value of 993 hPa and the instrument
error is zero, you are crossing the airport at a height above ground of: (standard
pressure setting = 1013 hPa, 1 hPa = 30 ft) A – 5000 ft B – 5600 ft C – 2815 ft D –
2215 ft Ref: AIR: atpl, ir; HELI: atpl, ir Ans: C 16523. Your pressure altimeter is
set at 1000 hPa. You have been cleared to join the traffic circuit at 2600 ft and
received current QNH = 1010 hPa. You join the circuit at 2600 ft but forget to
reset your altimeter. Other aircraft with correct altimeter settings indicating
2600 ft in the circuit will, compared to you, be flying: A – 300 ft above B – 300
ft below C – 100 ft above D – 100 ft below Ref: AIR: atpl, ir; HELI: atpl, ir
Ans: B

16524. At the official measuring level for a specific airport, an aircraft


altimeter, set at QNH for the airport, should read: A – the elevation of the
airport, but only at standard ISA temperature B – the elevation of the airport,
regardless of temperature C – zero, regardless of temperature D – zero, only at
standard ISA temperature Ref: all Ans: A 16526. Which factors below increase
density altitude for a given aerodrome: 1) 2) 3) 4) Decreasing air pressure
Increasing air pressure Decreasing temperature Increasing temperature

A – 1, 2 B – 1, 4 C – 2, 3 D – 2, 4 Ref: all Ans: B 16527. Which one of the


following conditions gives the shortest take off run, if the airports have the same
QNH? A – High temperature and low airport elevation B – Low temperature and low
airport elevation C – High temperature and high airport elevation D – Low
temperature and high airport elevation Ref: all Ans: B 16535. On the ground, an
altimeter will read ___ if QFE is set and ___ if QNH is set A – airfield elevation;
airfield altitude B – zero ft; airfield altitude C – zero ft; airfield height D –
zero ft; airfield elevation
Ref: all Ans: D 16536. If flying North with easterly drift, an aircraft’s altimeter
will progressively: A – under-read B – over-read C – remain correct D – impossible
to say – it depends on the atmospheric pressure Ref: all Ans: B 16541. An aircraft
is flying at FL 75 over point A where the QNH is 1013 hPa, enroute to B where the
QNH is 979 hPa. Assuming that 1 hPa equals 30 ft and that point B is 823 metres
AMSL the terrain clearance over B is: A – 6480 ft B – 4280 ft C – 3780 ft D – 5680
ft Ref: AIR: atpl, ir; HELI: atpl, ir Ans: C 16543. An aircraft is to fly at an
indicated altitude of 5000 ft from X (elevation 850 ft, QNH 984 hPA) to Y
(elevation 2300 ft QNH 1024 hPa). Assuming that the altimeter sub-scale is set to
984 mbs and 1 hPa = 30 ft, the height of the aircraft over X and Y will be: A –
5000 ft; 3900 ft B – 4150 ft; 3900 ft C – 4150 ft; 1500 ft D – 5000 ft; 1500 ft
Ref: AIR: atpl, ir; HELI: atpl, ir Ans: B 16568. The barometric reading of pressure
must be corrected for the following errors: A – temperature, index, instrument B –
index, temperature, pressure C – instrument, gravity, temperature D – instrument,
temperature, pressure Ref: all
Ans: D 24188. (Refer to figure 050-14) An aircraft is flying from Point A to Point
B on the upper level contour chart. The altimeter setting is 1013.2 hPa. Which of
these statements is correct? A – The true altitude will be higher at B than at A B
– The true altitude will be higher at A than at B C – Wind speed at A is higher
than at B D – Wind speed at A and at B is the same Ref: all Ans: B 24189. (Refer to
figure 050-15) An aircraft is flying from Point A to Point B on the upper level
contour chart. The altimeter setting is 1013.2 hPa. Which of these statements is
correct? A – Wind speed at B is higher than at A B – The true altitude will be
higher at A than at B C – Wind speed at Madrid is higher than at A D – The true
altitude will be higher at B than at A Ref: all Ans: D 24190. (Refer to figure 050-
15) An aircraft is flying from Point A to Point B on the upper level contour chart.
The altimeter setting is 1013.2 hPa. Which of these statements is correct? A – Wind
speed at B is higher than at A B – The true altitude will be higher at A than at B
C – Wind speed at A and at B is the same D – The true altitude will be higher at B
than at A Ref: all Ans: D
24229. An aeroplane flies at flight level 40. Elevation of the aerodrome: 990 ft
QNH 976 hPa The tower clears the pilot to fly at 3000 ft QNH Which of the following
statements is correct? A – Only a small change of altitude is necessary B – The
aeroplane has to climb about 1000 ft C – The aeroplane has to descend about 1000 ft
D – The aeroplane has to descend about 2000 ft Ref: AIR: atpl, ir; HELI: atpl, ir
Ans: A 24236. An aircraft flying at FL 45 (OAT 6oC) obtains a reading of 1860 ft on
its radio altimeter (ground elevation 3090 ft). What is the value of the QNH, to
the nearest hPa, at that point? A – 1042 B – 996 C – 1013 D – 1030 Ref: all Ans: D
24237. An aircraft maintains a constant indicated altitude of 4500 ft from A (360
ft/AMSL – QNH 986 hPa) to B (690 ft/AMSL –QNH 1011 hPa). Assuming that the
altimeter sub-scale setting remains unchanged at 986 hPa, the height of the
aircraft above the surface is: A – 4815 ft B – 3135 ft C – 4485 ft D – 5175 ft Ref:
AIR: atpl, ir; HELI: atpl, ir Ans: C
24238. An aircraft maintains a constant indicated altitude of 5500 ft from A (1050
ft/AMSL – QNH 968 hPa) to B (650 ft/AMSL – QNH 1016 hPa). Assuming that the
altimeter sub-scale setting remains unchanged at 968 hPa, the height of the
aircraft above the surface is: A – 4854 ft B – 6796 ft C – 6146 ft D – 7446 ft Ref:
AIR: atpl, ir; HELI: atpl, ir Ans: C 24239. An aircraft maintains a constant
indicated altitude of 6500 ft from A (600 ft/AMSL – QNH 1012 hPa) to B (930
ft/ALMSL – QNH 977 hPa). Assuming that the altimeter sub-scale setting remains
unchanged at 1012 hPa, the height of the aircraft above the surface is: A – 4625 ft
B – 6515 ft C – 5555 ft D – 5225 ft Ref: AIR: atpl, ir; HELI: atpl, ir Ans: A
24240. An aircraft maintains a constant indicated altitude of 7500 ft from A (270
ft/AMSL – QNH 1021 hPa) to B (1650 ft/AMSL – QNH 983 hPa). Assuming that the
altimeter sub-scale setting remains unchanged at 1021 hPa, the height of the
aircraft above the surface: A – 6204 ft B – 4824 ft C – 6876 ft D – 6474 ft Ref:
AIR: atpl, ir; HELI: atpl, ir Ans: B
24273. Given: Altimeter setting: 1013.2 hPa Altimeter reading: 5000 ft Outside air
temperature at 5000 ft: +5oC QFE: 958 hPa QNH: 983 hPa What is the true height of
the aeroplane above the aerodrome? A – 4325 ft B – 4190 ft C – 3515 ft D – 4865 ft
Ref: AIR: atpl, ir; HELI: atpl, ir Ans: C 24280. If the QFE, QNH and QFF of an
airport have the same value, A – the 1013.25 hPa level must be at MSL B – the
conditions must be as in the ISA C – the airport must be at MSL and the conditions
must be as in the ISA D – the airport must be at MSL Ref: all Ans: D 24313.
Pressure altitude is obtained by: A – setting the altimeter to QFF pressure B –
correcting the altimeter for temperature deviation from ISA C – setting the
altimeter to a station pressure which has been corrected to sea level D – setting
the altimeter to standard sea level pressure Ref: all Ans: D
24408. Which FL corresponds with the 200 hPa pressure level? A – FL 300 B – FL 390
C – FL 100 D – FL 50 Ref: all Ans: B 24409. Which FL corresponds with the 300 hPa
pressure level? A – FL 390 B – FL 300 C – FL 100 D – FL 50 Ref: all Ans: B 24410.
Which FL corresponds with the 500 hPa pressure level? A – FL 100 B – FL 160 C – FL
180 D – FL 390 Ref: all Ans: C 24411. Which FL corresponds with the 700 hPa
pressure level? A – FL 300 B – FL 180 C – FL 100 D – FL 390 Ref: all Ans: C
24412. Which FL corresponds with the 850 hPa pressure level? A – FL 100 B – FL 50 C
– FL 300 D – FL 390 Ref: all Ans: B 24456. You are flying at FL 340 (250 hPa) on
the northern hemisphere. The wind is geostrophic and there is a cross wind from the
right all the time. Your true altitude will: A – increase, only if the temperature
at your flight level is rising on your route B – decrease C – decrease, only if the
pressure at the surface is decreasing on your route D – increase Ref: all Ans: D
24458. You are flying over the sea at FL 250 and measure an outside temperature of
– 50oC. The pressure at sea level is 10213 hPa. What is your approximate true
altitude calculated using normal vertical change in temperature with increase in
height? A – 23770 ft/AMSL B – 26230 ft/AMSL C – 26770 ft/AMSL D – 23230 ft/AMSL
Ref: AIR: atpl, ir; HELI: atpl, ir Ans: A
050-02

WIND

050-02-01 Definition and measurement


8940. What is the approximate speed of a 40 knot wind, expressed in m/sec? A – 25
m/sec B – 15 m/sec C – 20 m/sec D – 30 m/sec Ref: all Ans: C 8944. (Refer to figure
050-98) What does zone A depict? A – A trough B – A ridge C – The warm sector D –
The cold front Ref: all Ans: A 8947. What values are used for the forecasted wind
at higher levels? A – Direction relative to grid north and speed in kmh B –
Direction relative to magnetic north and speed in knots C – Direction relative to
magnetic north and speed in kmh D – Direction relative to true north and speed in
knots Ref: all Ans: D
8952. The wind tends to follow the contour lines (isohypses) above the friction
layer because: A – the coriolis force tends to balance with the horizontal pressure
gradient force B – contour lines are lines that connect points with the same wind
speed in the upper air C – the coriolis force acts perpendicular on a line that
connects high and low pressure system D – the friction of the air with the earth’s
surface gives the airflow a diversion perpendicular to the gradient force Ref: all
Ans: A 8953. During periods of prolonged clear skies associated with anti-cyclonic
conditions, the: A – surface wind speed tends to be highest during the early
afternoon B – surface wind speed tends to be highest at night C – angle between
isobars and surface wind direction tends to be greatest in the early afternoon D –
wind tends to back from early morning until early afternoon Ref: all Ans: A 8963.
(Refer to figure 050-10) Which air mass and cloud depiction matches the routing A-
B? A–2 B–3 C–4 D–1 Ref: all Ans: B
8988. (Refer to figure 050-101) On which route do you expect moderate to severe CAT
at FL 300? A – Zurich-Rome B – London-Zurich C – Zurich-Copenhagen D – Paris-
Bordeaux Ref: all Ans: A 8994. Wind is caused by? A – Mixing of fronts B –
Horizontal pressure difference C – Earth rotation D – Surface friction Ref: all
Ans: B 9000. (Refer to figure 050-98) What does zone C depict? A – A trough B – A
ridge C – The warm sector D – The cold front Ref: all Ans: C 10561. (Refer to
figure 050-108) What wind speed do you expect over Rome at FL 340? A – 145 kt B –
340 kt C – 95 kt D – 140 km/h Ref: all Ans: A
10562. What causes surface winds to flow across the isobars at an angle rather than
parallel to the isobars? A – Coriolis force B – Surface friction C – The greater
density of the air at the surface D – The greater atmospheric pressure at the
surface Ref: all Ans: B 10600. (Refer to figure 050-71) What are the weather
conditions for a flight Zurich-Stockholm at FL 240? A – Your flight will be mainly
in clouds, outside of CAT areas B – Your flight will be mainly clear of clouds,
intermittently in icing conditions C – You may encounter thunderstorms
intermittently D – Your flight will be permanently clear of clouds Ref: all Ans: B
10600. (Refer to figure 050-71) What are the weather conditions for a flight
Zurich-Stockholm at FL 240? A – Your flight will be mainly in clouds, outside of
CAT areas B – Your flight will be mainly clear of clouds, intermittently in icing
conditions C – You may encounter thunderstorms intermittently D – Your flight will
be permanently clear of clouds Ref: all Ans: A 10632. What is the approximate speed
of a 25 knot wind, expressed in kilometres per hour? A – 60 km/h B – 35 km/h C – 55
km/h D – 45 km/h Ref: all Ans: D
10648. What prevents air from flowing directly from high pressure areas to low
pressure areas? A – The pressure gradient force B – Surface friction C – Katabatic
force D – Coriolis force Ref: all Ans: D 10687. What is the approximate speed of a
90 km/h wind, expressed in knots: A – 55 kt B – 50 kt C – 60 kt D – 70 kt Ref: all
Ans: B 10702. The difference between geostrophic wind and gradient wind is caused
by: A – curvature of isobars B – friction C – horizontal temperature gradients D –
slope of pressure surfaces Ref: all Ans: A 10783. (Refer to figure 050-98) What
does zone B depict? A – A trough B – A ridge C – The warm sector D – The cold front
Ref: all Ans: D
15860. In an area of Converging air in low level: A – convective clouds can be
dissolved B – stratified clouds can be dissolved C – clouds cannot be formed D –
clouds can be formed Ref: all Ans: D 15862. What relationship exists between the
wind at 3,000 feet and the surface wind? A – The wind at 3,000 feet is parallel to
the isohypses and the surface wind direction is across the isobars toward the low
pressure and the surface wind is weaker B – They have the same direction, but the
surface wind is weaker, caused by friction C – The geostrophic force deflects the
wind to the left in the Southern hemisphere D – the wind blows anti-clockwise round
anti-cyclones in the Northern hemisphere Ref: all Ans: A 16597. Which of the
following statements is NOT-TRUE? The Coriolis force deflects the wind to the right
in the Northern hemisphere only B – The geostrophic force is the same as the
Coriolis force C – The geostrophic force deflects the wind to the left in the
Southern hemisphere D – The Coriolis force deflects the wind to the right in both
hemispheres Ref: all Ans: D
16600. Buys Ballots Law implies that: A – the wind blows anti-clockwise round
depressions in both hemispheres B – the wind blows clockwise round an anti-cyclone
in the southern hemispheres C – the wind blows clockwise round a depression in the
southern hemisphere only D – the wind blows anti-clockwise round anti-cyclones in
the Northern Hemisphere Ref: all Ans: C 16609. Which of the following statements is
true? A – A gale has an average speed of 33 kts or more B – A gust is a squall
which lasts for several minutes C – A gust is a squall which lasts for several
minutes D – The more stable the atmosphere, the less turbulence Ref: all Ans: D
24219. A wind of 20 knots corresponds to an approximate speed of A – 10 m/sec B –
40 m/sec C – 10 km/h D – 50 km/h Ref: all Ans: A 24294. In the mid-latitudes of the
northern hemisphere the wind blows: A – direct from high to low pressure areas B –
clockwise around anti-cyclones and cyclones C – clockwise around cyclones and anti-
clockwise around anti-cyclones D – clockwise around anti-cyclones and anti-
clockwise around cyclones Ref: all Ans: D
24297. In this question the wind speed with straight isobars (vst), the wind speed
around a high pressure system (vhigh) and the wind speed around a low pressure
system (vlow) in the southern hemisphere are being compared while pressure gradient
and latitude are: A – vst < vhigh and vst < vlow B – vst > vhigh and vst < vlow C –
vst < vhigh and vst > vlow D – vst > vhigh and vst > vlow Ref: all Ans: C

050-02-02 Primary cause of wind


2849. For a similar pressure gradient, the geostrophic wind speed will be? A –
greater at 60oN than at 30oN B – greater at 30oN than at 60oN C – equivalent to
gradient wind ± thermal component D – the same at all latitudes north or south of
15o Ref: all Ans: B 8918. The wind speed in a system with curved isobars compared
to a system with straight isobars is (other conditions being the same): A – always
higher B – higher if curvature is anti-cyclonic C – always lower D – higher if
curvature is cyclonic Ref: all Ans: B 8965. What characteristics will the surface
winds have in an area where the isobars on the weather map are very close together?
A – Strong and parallel to the isobars B – Very weak but gusty and flowing across
the isobars C – Strong the flowing across the isobars D – Moderate and parallel to
the isobars Ref: all
Ans: C 8993. In the southern hemisphere what wind effect would you expect when
flying from a high pressure area towards a low pressure area at FL 100? A – Wind
from the left B – Wind from the right C – Tailwind with no drift D – Headwind with
no drift Ref: all Ans: B 10606. Geostrophic wind is the wind when isobars are: A –
straight lines and no friction is involved B – curved lines and no friction is
involved C – straight lines and friction is involved D – curved lines and friction
is involved Ref: all Ans: A 10631. The geostrophic wind is greater than the
gradient wind around a low pressure system because the: A – centrifugal force
opposes the pressure gradient B – centrifugal force is added to the pressure
gradient C – coriolis force is added to the pressure gradient D – coriolis force
opposes to the centrifugal force Ref: all Ans: A 10644. For the same pressure
gradient at 60oN, 50oN and 40oN the speed of the geostrophic wind will be: A
greatest at 40oN B – the same at all latitudes C – greatest at 60oN D – least at
50oN Ref: all Ans: A
10645. Divergence in the upper air results, near the surface, in: A – falling
pressure and likely dissipation of clouds B – rising pressure and likely formation
of clouds C – rising pressure and likely dissipation of clouds D – falling pressure
and likely formation of clouds Ref: all Ans: D 10649. Under anti-cyclone conditions
in the northern hemisphere, with curved isobars the speed of the gradient wind is:
A – less than the geostrophic wind B – greater than the geostrophic wind C – the
same as the thermal component D – proportional only to the coriolis force Ref: all
Ans: B 10656. The greater the pressure gradient the: A – further the isobars will
be apart and the weaker the wind B – closer the isobars and the stronger the wind C
– closer the isobars and the lower the temperatures D – further the isobars will be
apart and the higher the temperature Ref: all Ans: B 10692. Whilst flying at FL 180
on the northern hemisphere an aircraft experiences right drift. What effect, if
any, will this have on the aircraft’s true altitude? A – Without knowing the
pressure change this question cannot be answered B – It increases C – It remains
constant D – It decreases Ref: all Ans: D
10697. When isobars, for an area in the mid-latitudes on a weather map, are close
together, the wind is most likely to be: A – strong B – blowing perpendicular to
the isobars C – changing direction rapidly D – light Ref: all Ans: A 10698. Select
the true statement concerning isobars and wind flow patterns around high and low-
pressure systems that are shown on a surface weather chart? A – When the isobars
are close together, the pressure gradient force is greater and wind velocities are
stronger B – Surface winds flow perpendicular to the isobars C – Isobars connect
contour lines of equal temperature D – When the isobars are far apart, crest of
standing waves may be marked by stationary lenticular clouds Ref: all Ans: A 10701.
Which forces are balanced with geostrophic winds? A – Pressure gradient force,
coriolis force B – Friction force, pressure gradient force, coriolis force C –
Pressure gradient force, coriolis force, centrifugal force D – Pressure gradient
force, centrifugal force, friction force Ref: all Ans: A 10719. An aircraft flying
in the southern hemisphere at 2000 feet, has to turn to the right in order to allow
for drift. In which direction, relative to the aircraft, is the centre of low
pressure? A – To the left B – Behind C – In front D – To the right Ref: all Ans: C
10777. Where are you likely to find the strongest winds close to the ground? A – At
the centre of a high pressure system B – At the centre of a low pressure system C –
In the transition zone between two air masses D – Where there is little variation
in pressure over a large area during the winter months Ref: all Ans: C 15863. The
geostrophic wind depends on: A – density, earth’s rotation, geographic latitude B –
earth’s rotation, geographic latitude, centripetal force C – geographic latitude,
centripetal force, height D – centripetal force, height, pressure gradient Ref: all
Ans: A 15865. The geostrophic wind speed is directly proportional to the: A –
density of the air B – horizontal pressure gradient C – curvature of isobars D –
sine of latitude Ref: all Ans: B 15866. Geostrophic wind: A – always increases with
increasing height B – veers with height if cold air is advected in the northern
hemisphere C – is perpendicular to the horizontal pressure gradient force D – is
directly proportional to the density of the air Ref: all Ans: C
16595. A pressure gradient is said to exist when: A – two columns of air have
different temperatures B – surface pressure is compared at two different points on
the earth’s surface C – two points at the same level have a different atmospheric
pressure D – two points on the earth’s surface have the same isobar passing through
them Ref: all Ans: C 16596. The pressure gradient force acts: A – parallel to the
isobars with low pressure on its left (in the Northern hemisphere) B –
perpendicular to the isobars with low pressure behind it C – perpendicular to the
isobars and away from the high pressure D – parallel to the isobars and towards the
low pressure Ref: all Ans: C 16598. The wind, which blows when the gradient and
geostrophic forces are in balance, is: A – the gradient wind B – the coriolis wind
C – the geostrophic wind D – the surface Ref: all Ans: C 16599. A geostrophic wind
is a true wind only under the following conditions: A – straight and parallel
isobars, unchanging pressure field, no friction B – uniformly curved isobars,
constant pressure field, 2000 ft wind C – circular isobars, slack pressure
gradient, no friction D – parallel isobars, constant pressure field, no friction
Ref: all Ans: A
16601. For a given latitude, which of the following statements is true: A –
Geostrophic force acts at right angles to the wind and affects its speed and
direction B – Geostrophic force acts at right angles to the wind and affects speed
but not direction C – Geostrophic force acts at right angles to the wind and
affects direction but not speed D – none of the above since the geostrophic force
is only an apparent force Ref: all Ans: C 16602. With balanced flow, which of the
following statements is untrue? A – The geostrophic force decreases near the poles
B – The geostrophic force is non-existent at the equator C – The geostrophic force
varies in direct proportion to the wind speed D – The pressure gradient force is a
maximum at the poles Ref: all Ans: A 16603. The gradient wind: A – blows across the
isobars when there is a surface pressure gradient B – is the 2000 ft geostrophic
wind C – is the surface wind affected by friction D – blows parallel to curved
isobars due to a combination of the pressure gradient force, geostrophic force and
cyclostrophic force Ref: all Ans: D 16604. The effect of curved isobars on
geostrophic wind speed correction is: A – greater at high latitudes B – greater at
low altitudes C – less at high latitudes D – no effect Ref: all Ans: C
16605. The gradient wind is: A – less that the geostrophic wind around a low B –
more than the geostrophic wind around a low C – less that the geostrophic wind
around a high D – none of the above Ref: all Ans: A 16606. In the Northern
hemisphere, surface friction causes the geostrophic wind to: A – back and decrease
B – veer and decrease C – back and increase D – veer and increase Ref: all Ans: A
16607. Which of the following statements is untrue? A – At night the surface wind
is lighter in speed and more inclined towards low pressure B – Over land, the
surface wind is backed by 15o to the isobars and is 2/3 geostrophic speed C – The
steeper the pressure gradient, the less marked the nocturnal effect D – At night,
the wind can be geostrophic down to 1000 ft or less Ref: all Ans: B 24270. For the
same horizontal distance between adjacent isobars at latitudes 50oN and 30oN the
gradient wind speed will be least at latitude: A – 30oN in an anti-cyclone B – 50oN
in an anti-cyclone C – 30oN with a cyclonic circulation D – 50oN with a cyclonic
circulation Ref: all Ans: D
24398. When compared to the geostrophic wind in the northern hemisphere, surface
friction will cause the surface wind to: A – back and increase B – back and
decrease C – veer and decrease D – veer and increase Ref: all Ans: B

050-02-03 General circulation


8941. Considering Melbourne (C) in July, the weather is predominantly influenced by
the zone of: A – equatorial low pressure due to the proximity of the inter-tropical
convergence zone over central Australia B – Antarctic high pressure due to the
absence of any protective land mass between south Australia and Antarctica C –
disturbed temperate low pressure, bringing an almost continuous succession of
fronts resulting in strong winds, low cloud and rain D – sub-tropical high
pressure, with the occasional passage of fronts originating in the adjacent zone of
disturbed temperate low pressure Ref: all Ans: D 8968. (Refer to figure 050-48)
Assuming a generalised zonal system of world climatic and wind circulation, zone U
is in area of: A – SW trade winds B – travelling depressions C – NE trade winds D –
sub-tropical high pressure Ref: all Ans: C
8976. (Refer to figure 050-100) Select from the map the average wind for the route
Frankfurt-Rome at FL 170: A – 200/50 kt B – 230/40 kt C – 060/50 kt D – 030/35 kt
Ref: all Ans: B 8998. Between which latitudes are you most likely to find the sub-
tropical high pressure belt? A – 55o – 75o B – 10o – 15o C – 35o – 55o D – 25o –
35o Ref: all Ans: D 10566. In the central part of the Atlantic Ocean between 10oN
and 20oN the prevailing winds are: A – NE monsoon in winter and SW monsoon in
summer B – NE trade winds C – SE trade winds D – SW winds throughout the whole year
Ref: all Ans: B 10576. The average slope of a cold front is in the order of: A –
1:150 B – 1:250 C – 1:500 D – 1:80 Ref: all Ans: D
10650. (Refer to figure 050-48) Assuming a generalised zonal system of world
climatic and wind circulation, zone Y is an area of: A – SE trade winds B – NE
trade winds C – travelling low pressure systems D – sub-tropical high pressure
systems Ref: all Ans: C 10670. (Refer to figure 050-107) What is the average wind
for the route Shannon-Lisbon, FL 290: A – 360/80 kt B – 030/70 kt C – 190/75 kt D –
340/90 kt Ref: all Ans: A 10781. (Refer to figure 050-107) What is the average
temperature for the route Geneva-Stockholm, FL 260? ABCD-55oC -51oC -63oC -47oC

Ref: all Ans: D 10790. (Refer to figure 050-99) Assuming a generalised zonal system
of world climatic and wind circulation, zone T is an area of: A – NE trade winds B
– SE trade winds C – travelling low pressure systems D – sub-tropical high pressure
systems Ref: all Ans: C
15861. You are flying from east to west in the northern hemisphere at the 500 hPa
pressure surface. Which of the following statements is correct? A – If the wind is
from the north you are gaining altitude B – If the wind is from the south you are
gaining altitude C – If you have a head wind you are gaining altitude D – If you
have a tail wind you are losing altitude Ref: all Ans: A 16437. Which of the
following alternatives is the correct one, regarding the surface wind in relation
to the air pressure in the Northern hemisphere? A – The wind over land blows
parallel to the isobars B – The wind around a high pressure blows clockwise and
slants across the isobars towards higher pressure C – The wind blows counter-
clockwise around a low and slants across the isobars towards lower pressure D – The
wind blows counter-clockwise around a high and clockwise around a low Ref: all Ans:
C 24262. During periods of undisturbed radiation weather, overland, the: A – wind
tends to back from early morning until early afternoon B – surface wind speed tends
to be highest at night C – angle between isobars and surface wind direction tends
to be greatest in the mid-afternoon D – surface wind speed tends to be highest
during the mid-afternoon Ref: all Ans: D
050-02-04 Turbulence
8933. Which cloud type may indicate the presence of severe turbulence? A –
Altocumulus lenticularis B – Stratocumulus C – Cirrocumulus D – Nimbostratus Ref:
all Ans: A 8981. Which degree of aircraft turbulence is determined by the following
ICAO description? “There may be moderate changes in aircraft attitude and/or
altitude but the aircraft remains in positive control at all times. Usually, small
variations in air speed. Changes in accelerometer readings of 0.5 to 1.0g at the
aircraft’s centre of gravity. Occupants feel strain against seat belts. Loose
objects move about. Food service and walking are difficult.” A – Severe B – Light C
– Moderate D – Violent Ref: all Ans: C 8983. What degree of turbulence, if any, is
likely to be encountered while flying through a cold front in the summer over
Central Europe at FL 100? A – Moderate turbulence in NS cloud B – Severe turbulence
in CB cloud C – Light turbulence in CB cloud D – Light turbulence in ST cloud Ref:
all Ans: B
10676. On a clear summer day, turbulence caused by solar heating is most
pronounced: A – immediately after sunset B – during the early afternoon C – during
early morning hours before sunshine D – about mid-morning Ref: all Ans: B 10728.
Fair weather cumulus often is an indication of: A – a high risk of thunderstorms B
– poor visibility at surface C – smooth flying conditions below the cloud level D –
turbulence at and below the cloud level Ref: all Ans: D 16329. If a strong wind
perpendicular to a ridge decreases or reverses in direction at medium and high
levels the likely result is: A – travelling rotors with very severe turbulence B –
stationary rotors with very severe turbulence C – stationary rotors with light
turbulence D – travelling rotors with light turbulence Ref: all Ans: A

050-02-05 Variation of wind with height


8913. If Paris reports a wind of 08010 kt on the METAR, what wind velocity would
you expect to encounter at a height of 2000 feet above the ground? A – 11020 kt B –
08015 kt C – 05020 kt D – 08005 kt Ref: all Ans: A
8955. You are flying at 2500 ft/AGL, with a southerly wind, and intend to land at
an airport, at sea level directly below. From approximately which direction would
you expect the surface wind (mid-latitude, northern hemisphere)? A – South B –
South-southwest C – Southwest D – South-southeast Ref: all Ans: D 8962. In the
lower layers of the atmosphere due to friction the wind changes direction towards
the low pressure area because: A – turbulence is formed and pressure decreases B –
the pressure gradient increases C – turbulence is formed and pressure increases D –
wind speed decreases and therefore coriolis force decreases Ref: all Ans: D 8996.
In the northern hemisphere the wind at the surface blows: A – from a low pressure
area to a high pressure area B – counter-clockwise around, and towards the centre
of, a low pressure area C – clockwise around, and away from the centre of a low
pressure area D – counter-clockwise around, and away from the centre of, a high
pressure area Ref: all Ans: B 10647. In a low pressure system the convergence at
the surface is caused by: A – the imbalance of the horizontal gradient force and
the coriolis force B – centripetal forces C – frictional forces D – the curvature
of the isobars Ref: all Ans: C
10660. If Paris reports a wind of 19015 kt on the METAR, what wind velocity would
you expect to encounter at a height of 2000 feet above the ground? A – 16020 kt B –
22030 kt C – 25025 kt D – 22010 kt Ref: all Ans: B 10671. At the approach of a warm
front (northern hemisphere) the wind direction changes from the surface up to the
tropopause. The effect of this change is that the wind: A – veers in the friction
layer and backs above the friction layer B – backs in the friction layer and veers
above the friction layer C – veers in the friction layer and veers above the
friction layer D – backs in the friction layer and backs above the friction layer
Ref: all Ans: C 10693. In the northern hemisphere the gradient wind of a cyclonic
pressure distribution is 350/24 over the sea the surface wind would approximate: A
– 340/20 B – 030/20 C – 240/28 D – 030/28 Ref: all Ans: A 10725. The geostrophic
wind is less than the gradient wind around an anti-cyclone because the: A –
centrifugal force opposes the pressure gradient B – centrifugal force is added to
the pressure gradient C – effect of coriolis is added to friction D – coriolis
effect opposes the centrifugal force Ref: all Ans: B
10794. If Paris reports a wind of 30012 kt on the METAR, what wind velocity would
you expect to encounter at a height of 2000 feet above the ground? A – 30025 kt B –
23030 kt C – 33025 kt D – 27020 kt Ref: all Ans: C 10797. If Paris reports a wind
of 16020 kt on the METAR, what wind velocity would you expect to encounter at a
height of 2000 feet above the ground: A – 14020 kt B – 16030 kt C – 19040 kt D –
17015 kt Ref: all Ans: C 15789. Generally northern hemisphere winds at 5000 ft AGL
are south westerly while most of the surface winds are southerly. What is the
primary reason of difference between these two wind directions? A – A strong
pressure gradient at higher altitudes B – Stronger coriolis force at the surface C
– The influence of warm air at the lower altitude D – Friction between the wind and
the surface Ref: all Ans: D 15838. An aircraft is approaching under visual flight
rules an airfield whose runway is parallel to the coast. When down wind over the
sea, the airfield is on the left. What wind effect should be anticipated on final
approach and landing during a sunny afternoon? A – Tail wind B – Head wind C –
Crosswind from the left D – Crosswind from the right Ref: all
Ans: C 15859. During a descent from 2000 ft above the surface to the surface (no
frontal passage) the wind normally: A – veers and decreases B – backs and decreases
C – veers and increases D – backs and increases Ref: all Ans: B 15864. The vertical
extent of the friction layer depends primarily on: A – wind speed, roughness of
surface, temperature B – roughness of surface, temperature, local time C –
temperature, local time environmental lapse rate D – stability, wind speed,
roughness of surface Ref: all Ans: D

050-02-06 Local winds


8905. Which of the following is true concerning an aircraft that is flying at FL
180 in the northern hemisphere where wind is geostrophic and the true altitude
remains constant? A – There is a cross wind from the right B – There is a cross
wind from the left C – There is no cross wind D – Without knowing temperature at FL
180 this question cannot be answered Ref: all Ans: C
8927. An aircraft is flying in the southern hemisphere at low altitude (less than
2000 feet) and going directly away from a centre of low pressure. What direction,
relative to the aircraft, does the wind come from? A – From the left and slightly
on the tail B – From the right and slightly on the nose C – From the right and
slightly on the tail D – From the left and slightly on the nose Ref: all Ans: D
8942. In a mountain-valley wind circulation, the mountain wind blows: A – during
the day up from the valley B – at night up from the valley C – during the day down
from the mountains D – at night down from the mountains Ref: all Ans: D 10577. In a
land and sea breeze circulation the land breeze blows: A – during the day and is
stronger than the sea breeze B – during the night and is weaker than the sea breeze
C – during the day and is weaker than the sea breeze D – during the night and is
stronger than the sea breeze Ref: all Ans: B 10590. Which of the following is true
of a land breeze? A – It blows from land to water B – It blows from water to land C
– It blows by day D – It blows only at noon Ref: all Ans: A
10599. Friction between the air and the ground results in the northern hemisphere
in: A – backing of the wind and increase of wind speed at the surface B – veering
of the wind and decrease of wind speed at the surface C – backing of the wind and
decrease of wind speed at the surface D – veering of the wind and increase of wind
speed at the surface Ref: all Ans: C 10621. A high pressure area (shallow pressure
gradient) covers an area of the Mediterranean Sea and its nearby airport. What
surface wind direction is likely at the airport on a sunny afternoon? A – Parallel
to the coastline B – Land to sea C – Variable D – Sea to land Ref: all Ans: D
10641. In the northern hemisphere a pilot flying at 1000 ft/AGL directly towards
the centre of a low pressure area, will find the wind blowing from: A – about 45
degrees to the right of directly ahead B – right and behind C – left and behind D –
directly ahead Ref: all Ans: C 10658. The most frequent wind direction in a valley
caused by thermal effects is toward the: A – valley during daylight hours B –
mountain at night C – mountain during daylight hours D – valley during daylight as
much as at night Ref: all Ans: C
10662. The sea breeze is a wind from the sea: A – blowing at night in mid latitudes
B – that reaches up to the tropopause in daytime C – occurring only in the lower
layers of the atmosphere in daytime D – occurring only in mid latitudes and in day
time Ref: all Ans: C 10681. When otherwise calm and clear conditions exist a
station on the shore of a large body of water will experience wind: A – continually
from land to water B – from the water in daytime and from the land at night C –
continually from water to the land D – from the land in day time and from the water
at night Ref: all Ans: B 10684. An aircraft is approaching under visual flight
rules an airfield whose runway is parallel to the coast. When down wind over the
sea, the airfield is on the right. What wind effect should be anticipated on final
approach and landing during a sunny afternoon? A – Cross wind from the left B –
Cross wind from the right C – Tailwind D – Headwind Ref: all Ans: B 10726. A
mountain breeze (katabatic wind) blows: A – down the slope during the night B – up
the slope during the day C – down the slope during the day D – up the slope during
the night Ref: all Ans: A
15883. In the northern hemisphere with an anti-cyclonic pressure system the
geostrophic wind at 2000 ft over the sea is 060/15. At the same position the
surface wind is most likely to be: A – 075/12 B – 060/18 C – 060/12 D – 045/12 Ref:
all Ans: D 16611. The normal maximum height of the sea breeze in mid altitudes is
approximately: A – 5000 ft B – 200 ft C – 500 – 1000 ft D – 50 ft Ref: all Ans: C
24211. A high pressure area (slack pressure gradient) covers part of the
Mediterranean Sea and coastal region during the summer. What surface wind direction
is likely at an airport at the coast on a sunny afternoon? A – Land to sea B – Sea
to land C – Variable D – Parallel to the coastline Ref: all Ans: B
24317. Sea breezes are most likely to occur when: A – slack pressure gradient and
clear skies result in relatively high land temperatures B – a strong pressure
gradient, relatively high sea temperatures and overcast conditions persist C – a
strong pressure gradient, relatively high sea temperatures and clear skies at night
exist D – a slack pressure gradient, relatively high sea temperatures and overcast
conditions persist Ref: all Ans: A

050-02-07 Jet streams


8906. What jet streams are likely to be crossed during a flight from Stockholm to
Rio de Janeiro (23oS) at FL 350 in July? A – A polar front jet stream followed by a
sub-tropical jet stream and later, a second polar front jet stream B – A sub-
tropical jet stream followed by a polar front jet stream C – A polar front jet
stream followed by one or two sub-tropical jet streams D – One sub-tropical jet
stream Ref: AIR: atpl; HELI: atpl Ans: C 8930. What is the minimum speed for a wind
to be classified as a jet stream? A – 70 kt B – 50 kt C – 60 kt D – 100 kt Ref:
AIR: atpl; HELI: atpl Ans: C
8931. In which zone of a jet stream is the strongest CAT to be expected? A – About
12000 ft above the core B – The warm air side of the core C – Exactly in the centre
of the core D – The cold air side of the core Ref: AIR: atpl; HELI: atpl Ans: D
8956. In the month of August you prepare a flight (cruising level FL 370) from
Bombay (19oN – 73oE) to Bangkok (13oN – 100oE). What wind conditions can you
expect? A – Light winds diagonal to the route B – Head winds C – Tail winds D –
Strong northerly winds Ref: AIR: atpl; HELI: atpl Ans: B 8987. While crossing a jet
stream at right angles in Western Europe (3000 ft below its core) and OAT is
decreasing, what would be the prevailing wind? A – A head wind B – Cross wind from
the right C – Cross wind from the left D – A tail wind Ref: AIR: atpl; HELI: atpl
Ans: B 10565. Where, as a general rule, is the core of the polar front jet stream
to be found? A – Just below the cold air tropopause B – In the cold air mass C –
Just above the warm air tropopause D – In the warm air mass Ref: AIR: atpl; HELI:
atpl Ans: D
10568. Which of the following types of jet streams can be observed all year round?
A – Equatorial jet stream/arctic jet stream B – Equatorial jet stream/polar front
jet stream C – Arctic jet stream/sub-tropical jet stream D – Sub-tropical jet
stream/polar front jet stream Ref: AIR: atpl; HELI: atpl Ans: D 10569. Under which
of the following conditions is the most severe CAT likely to be experienced: A – A
jet stream, with great spacing between the isotherms B – A curved jet stream near a
deep trough C – A westerly jet stream at low latitudes in the summer D – A straight
jet stream near a low pressure area Ref: AIR: atpl; HELI: atpl Ans: B 10592. A wind
sounding in the region of a polar front jet stream gives the following wind profile
(Northern hemisphere) 900 hPa 700 hPa 400 hPa 250 hPa 220/20 kt 230/35 kt 280/85 kt
310/120 kt 800 hPa 500 hPa 300 hPa 200 hPa 220/25 kt 260/60 kt 300/100 kt 310/80 kt

Which system is the jet stream associated with: A – With an easterly wave B – With
a cold front C – With a ITCZ D – With a warm front Ref: AIR: atpl; HELI: atpl Ans:
D
10605. At approximately what altitude is the sub-tropical jet stream found over
Europe? A – FL 500 B – FL 200 C – FL 300 D – FL 400 Ref: AIR: atpl; HELI: atpl Ans:
D 10609. What is the main cause for the formation of a polar front jet stream? A –
The varied elevations of the tropopause in the polar front region B – The pressure
difference, close to the ground, between a high over the Azores and a low over
Iceland C – The north-south horizontal temperature gradient at the polar front D –
Strong winds in the upper atmosphere Ref: AIR: atpl; HELI: atpl Ans: C 10619. What
is the average height of the jet core within a polar front jet stream? A – 50000 ft
B – 20000 ft C – 40000 ft D – 30000 ft Ref: AIR: atpl; HELI: atpl Ans: D 10626.
Which area of a polar front jet steam in the northern hemisphere has the highest
probability of turbulence? A – Looking downstream, the area to the left of the core
B – Looking downstream, the area to the right of the core C – In the core of the
jet stream D – Above the core in the boundary between warm and cold air Ref: AIR:
atpl; HELI: atpl Ans: A
10633. Which of the following statements concerning the core of a polar front jet
stream is correct? A – It lies in the warm air; its pressure surfaces are
horizontal at the height of the core B – It and its surface projection lie in the
warm air C – It lies at a height where there is no horizontal temperature gradient,
the slope of the pressure surfaces at the height of the core is at its maximum D –
It lies in the cold air; the thermal wind reverses direction at the height of the
core Ref: AIR: atpl; HELI: atpl Ans: C 10640. What is the most significant
difference between an equatorial jet stream and all the other jet streams? A –
Horizontal dimension B – Vertical dimensions C – Wind direction D – Wind speed Ref:
AIR: atpl; HELI: atpl Ans: C 10643. What is the average height of the arctic jet
stream core? A – 30,000 ft B – 20,000 ft C – 40,000 ft D – 50,000 ft Ref: AIR:
atpl; HELI: atpl Ans: B
10667. You cross a jet stream in horizontal flight at approximately right angles.
While crossing, in spite of a strong wind of 120 kt, you notice the temperature
barely changes. A – You assume the front associated with the jet stream to be very
weak with practically no temperature difference between the two air masses B – This
phenomenon is absolutely normal as you are crossing the jet core C – Since the
result of such readings seems impossible, you will after landing have the
instruments tested D – This phenomenon does not surprise you at all, since normally
no large temperature differences are possible at these heights Ref: AIR: atpl;
HELI: atpl Ans: B 10672. Which jet stream blows all year round, over the northern
hemisphere? A – The arctic jet stream B – The polar night jet stream C – The
equatorial jet stream D – The sub-tropical jet stream Ref: AIR: atpl; HELI: atpl
Ans: D 10690. Most strong air currents at higher levels (jet streams) have a
westerly direction. There is, however, an important easterly jet stream. When and
where is it likely to be encountered? A – In winter along the Russian coast facing
the Arctic ocean B – In summer from south east Asia extending over southern India
to central Africa C – In summer from the Middle East extending over the southern
part of the Mediterranean to southern Spain D – Throughout the year to the south of
the Azorian high Ref: AIR: atpl; HELI: atpl Ans: B
10709. On a particular day part of a polar front jet stream runs from north to
south in the northern hemisphere. This means that: A – above the core of the jet
the horizontal temperature gradient runs from north to south B – the polar air is
on the eastern side and above the core of the jet C – below the core of the jet the
horizontal temperature gradient runs from north to south D – the polar air is below
and to the east of the core of the jet Ref: AIR: atpl; HELI: atpl Ans: D 10717.
(Refer to figure 050-39) What name is given to the jet stream lying over North
Africa (B)? A – Polar front jet stream B – Equatorial jet stream C – Sub-tropical
jet stream D – Arctic jet stream Ref: AIR: atpl; HELI: atpl Ans: C 10720. Where, in
central Europe, are the highest wind speeds to be found? A – At about 5500 metres
altitude B – Just below the tropopause C – Close to the ground D – In the
stratosphere Ref: AIR: atpl; HELI: atpl Ans: B 10727. Which jet stream is connected
with a surface front system? A – The arctic jet stream B – The polar front jet
stream C – The sub-tropical jet stream D – The equatorial jet stream Ref: AIR:
atpl; HELI: atpl Ans: B
10786. An aircraft is flying through the polar front jet stream from south to
north, beneath the core. How would the OAT change, in the northern hemisphere,
during this portion of the flight? A – It first increases, then decreases B – It
increases C – It decreases D –It remains constant Ref: AIR: atpl; HELI: atpl Ans: C
10792. During the winter months in mid-latitudes in the northern hemisphere, the
polar front jet stream moves towards the: A – south and speed decreases B – north
and speed decreases C – south and speed increases D – north and speed increases
Ref: AIR: atpl; HELI: atpl Ans: C 10795. An aircraft over Western Europe is
crossing a jet stream 2500 ft below its core at right angles. While crossing, the
outside temperature is increasing. The prevailing wind is: A – head wind B – from
the left C – tail wind D – from the right Ref: AIR: atpl; HELI: atpl Ans: D 15813.
What is the approximate ratio between height and width for a jet stream cross
section? A – 1/1 B – 1/10 C – 1/1000 D – 1/100 Ref: AIR: atpl; HELI: atpl Ans: D
15815. A wind speed of 350 kt within a jet stream core should be worldwide regarded
as: A – possible but a very rare phenomenon B – not possible C – a common
occurrence D – not unusual in polar regions Ref: AIR: atpl; HELI: atpl Ans: A
15878. Which of the following statements concerning jet streams is correct? A – In
the southern hemisphere only easterly jet streams occur B – In the northern
hemisphere both westerly and easterly jet streams occur C – In the northern
hemisphere only westerly jet streams occur D – In the southern hemisphere no jet
streams occur Ref: AIR: atpl; HELI: atpl Ans: B 16335. In January, a mean sub-
tropical jet appears at: A – 30oS B – 30oN C – 60oS D – 50oS Ref: AIR: atpl; HELI:
atpl Ans: B 16355. A jet stream is: A – a band of strong winds only found near
thunderstorms B – a broad band of strong winds flattened and tubular in cross
section C – a narrow band of strong winds only found near fronts D – a narrow band
of strong winds flattened and tubular in cross section Ref: AIR: atpl; HELI: atpl
Ans: D
16365. The normal maximum speeds of the Polar Front Jet are in the region of: A –
150 kt in January/135 kt in July B – 150 kt in July/135 kt in January C – 150 kt in
July/50 kt in January D – 135 kt in July/135 kt in January Ref: AIR: atpl; HELI:
atpl Ans: A 24251. At which time, if any, are polar front jet streams over the
South Pacific usually strongest? A – October B – January C – There is no annual
variation D – July Ref: AIR: atpl; HELI: atpl Ans: D 24302. Isotechs are lines
joining equal: A – wind speed lapse rates B – sea level pressures C – horizontal
wind speed gradients D – wind speeds Ref: AIR: atpl; HELI: atpl Ans: D 24304.
Maximum wind speeds associated with sub-tropical jet streams are usually located in
the: A – polar air above the tropopause B – tropical air above the tropopause C –
tropical air below the tropopause D – polar air below the tropopause Ref: AIR:
atpl; HELI: atpl Ans: C
24328. The core of the polar front jet stream is usually located in the: A –
tropical air above the tropical tropopause B – polar air above the tropopause C –
polar air below the tropopause D – tropical air below the tropopause Ref: AIR:
atpl; HELI: atpl Ans: D 24371. What is necessary for the development of a polar
front jet stream? A – An unstable atmosphere up to great heights B – Strong
vertical temperature gradients C – A uniform pressure pattern D – Strong horizontal
temperature gradients Ref: AIR: atpl; HELI: atpl Ans: D 25626. (Refer to figure
050-39) What name is given to the jet stream lying across India (A)? A – Tropical
jet stream B – Polar front jet stream C – Arctic jet stream D – Sub-tropical jet
stream Ref: AIR: atpl; HELI: atpl Ans: A 25630. An aircraft is flying from south to
north, above the polar front jet stream, at FL 400 in the southern hemisphere. What
change, if any, in temperature will be experienced? A – It decreases B – It
increases C – It stays the same D – It decreases and then increases Ref: AIR: atpl;
HELI: atpl Ans: A
25636. Where, as a general rule, is the core of the polar front jet stream to be
found: A – Just above the warm air tropopause B – In the polar air mass C – In the
tropical air mass D – Just below the cold air tropopause Ref: AIR: atpl; HELI: atpl
Ans: C 8909. (Refer to figure 050-11) In which direction does the polar front move
in this picture? A–1 B–2 C–3 D–3 Ref: all Ans: C

050-02-08 Standing waves


10675. At the top of orographic waves, in mountainous regions, the cloud most
likely to be encountered is: A – altocumulus lenticularis B – cirrostratus C –
cirrus D – cumulus mediocris Ref: all Ans: A 16613. Standing waves are likely when:
A – wind speeds are uniform with height B – the atmosphere is uniformly stable C –
the wind direction is at 45o to the ridge of the hills D – none of the above Ref:
all Ans: D
050-03

THERMODYNAMICS 050-03-01 Humidity

9004. Relative humidity: A – is not affected by temperature changes of the air B –


is not affected when air is ascending or descending C – changes when water vapour
is added, even though the temperature remains constant D – does not change when
water vapour is added provided the temperature of the air remains constant Ref: all
Ans: C 9012. The dew point temperature: A – cannot be equal to the air temperature
B – is always lower than the air temperature C – is always higher than the air
temperature D – can be equal to the air temperature Ref: all Ans: D 9018. Relative
humidity: A – is higher in cool air than in warm air B – is higher in warm air than
in cool air C – increases if the air is cooled whilst maintaining the vapour
pressure constant D – decreases if the air is cooled whilst maintaining the vapour
pressure constant Ref: all Ans: C
9019. Relative humidity depends on: A – temperature of the air only B – moisture
content and pressure of the air C – moisture content of the air only D – moisture
content and temperature of the air Ref: all Ans: D 9022. Which of the following
statements is true of the dew point of an air mass? A – It can be used to estimate
the air mass’ relative humidity even if the air temperature is unknown B – It can
be higher than the temperature of the air mass C – It can be used together with the
air pressure to estimate the air mass’ relative humidity D – It can only be equal
to, or lower, than the temperature of the air mass Ref: all Ans: D 9027. During the
late afternoon an air temperature of +12oC and a dew point of +5oC were measured.
What temperature change must occur during the night in order to induce saturation?
A – It must decrease to +6oC B – It must decrease by 5oC C – It must decrease to
+5oC D – It must decrease to +7oC Ref: all Ans: C 9040. (Refer to figure 050-42)
Where do the westerly waves occur in this picture? A – In Central Europe B – Over
the North Sea C – In the Mediterranean Sea D – Over the Baltics Ref: all Ans: A
9047. Dew point is defined as: A – the temperature below which the change of state
in a given volume of air will result in the absorption of latent heat B – the
lowest temperature at which evaporation will occur for a given pressure C – the
lowest temperature to which air must be cooled in order to reduce the relative
humidity D – the temperature to which moist air must be cooled to become saturated
at a given pressure Ref: all Ans: D 9049. Relative humidity at a given temperature
is the relation between: A – dew point and air temperature B – water vapour weight
and dry air weight C – water vapour weight and humid air volume D – actual water
vapour content and saturated water vapour content Ref: all Ans: D 9051. When a
given mass of air descends, what effect will it have on relative humidity? A – It
increases up to 100%, then remains stable B – It increases C – It remains constant
D – It decreases Ref: all Ans: D 9064. The relative humidity of a sample air mass
is 50%. How is the relative humidity of this air mass influenced by changes of the
amount of water vapour in it? A – It is not influenced by changing water vapour B –
It increases with increasing water vapour C – It decreases with increasing water
vapour D – It is only influenced by temperature Ref: all Ans: B
9066. Which of the following is the definition of relative humidity? A – Ratio
between the actual mixing ratio and the saturation mixing ratio X 100 B – Ratio
between air temperature and dew point temperature X 100 C – Ratio between water
vapour pressure and atmospheric pressure X 100 D – Ratio between water vapour (g)
and air (kg) X 100 Ref: all Ans: A 9074. How does relative humidity and the dew
point in an unsaturated air mass change with varying temperature? A – When
temperature decreases, the relative humidity and the dew point remains constant B –
When temperature increases, the relative humidity increases, and the dew point
decreases C – When temperature decreases, the relative humidity decreases, and the
dew point increases D – When temperature increases, the relative humidity
decreases, and the dew point remains constant Ref: all Ans: D 9077. The dew point
temperature: A – can be reached by lowering the pressure whilst keeping temperature
constant B – can be reached by cooling the air whilst keeping pressure constant C –
cannot be equal to the air temperature D – cannot be lower than the air temperature
Ref: all Ans: B 9083. The maximum amount of water vapour that the air can contain
depends on the: A – dew point B – relative humidity C – stability of the air D –
air temperature Ref: all Ans: D
9093. The difference between temperature and dew point is greater in: A – air with
low temperature B – moist air C – air with high temperature D – dry air Ref: all
Ans: D 9094. (Refer to figue 050-42) What does this picture depict? A – A westerly
wave over Central Europe B – A high pressure area over Central Europe C – South
foehn D – North foehn Ref: all Ans: A 9100. What does dew point mean? A – The
temperature at which ice melts B – The temperature to which a mass of air must be
cooled in order to reach saturation C – The freezing level (danger of icing) D –
The temperature at which the relative humidity and saturation vapour pressure are
the same Ref: all Ans: B
10174. (Refer to figure 050-44) Which weather situation can be expected during the
day at Zurich airport? A – TAF LSZH 1601 05020G35KT 8000 BKN015 TEMPO 1720 05018KT
+SHSN W002= B – TAF LSZH 1601 23012KT 6000 RA BKN012 OVC030 TEMPO 2023 22025G40KT
1600 +SNRA BKN003 OV3015= C – TAF LSZH 1601 VRB02KT 8000 SCT280 BECMG 1618 00000KT
3500 MIFG BECMG 1820 1500 BCFG BECMG 2022 0100 FG W001= D – TAF LSZH 1601 32008KT
9999 SCT030TCU TEMPO 2201 32020G32KT 3000 TSRA BKN020CB= Ref: all Ans: C 10192.
(Refer to figure 050-43) Where do the westerly waves occur in this picture? A – In
Central Europe B – Over Scandinavia C – In the Mediterranean Sea D – Over the
Baltics Ref: all Ans: B 10193. How, if at all, is the relative humidity of an
unsaturated air mass influenced by temperature changes? A – It increases with
increasing temperature B – It is not influenced by temperature changes C – It
decreases with increasing temperature D – It is only influenced by the amount of
water vapour Ref: all Ans: C
16563. In a high relative humidity condition, which of the following sets of
conditions is true? A – High evaporation rate; reduced latent heat absorption;
small wet/dry bulb difference B – Increased latent heat absorption; low evaporation
rate; large wet/dry bulb difference C – Small wet/dry bulb difference; high
evaporation rate; increased latent heat absorption D – Small wet/dry bulb
difference; reduced latent heat absorption; low evaporation rate Ref: all Ans: D
24385. What is true for the water vapour distribution in the layer between the
surface and the 500 hPa pressure surface in the trade wind belt? A – The lower part
is relatively dry and the upper part is relatively moist B – The whole layer is
relatively dry C – The whole layer is relatively moist D – The lower part is
relatively moist and the upper part is relatively dry Ref: all Ans: D

050-03-02 Change of state of aggregation


9009. Super-cooled droplets can occur in: A – clouds but not in precipitation B –
clouds, fog and precipitation C – precipitation but not in clouds D – clouds but
not in fog Ref: AIR: atpl, ir; HELI: all Ans: B
9038. The process by which water vapour is transformed directly into ice is known
as: A – super cooling B – sublimation C – super saturation D – radiation cooling
Ref: AIR: atpl, ir; HELI: all Ans: B 9042. In which of the following changes of
state is latent heat released? A – Liquid to gas B – Solid to liquid C – Solid to
gas D – Gas to liquid Ref: AIR: atpl, ir; HELI: all Ans: D 9043. When water
evaporates into unsaturated air: A – relative humidity is decreased B – heat is
released C – relative humidity is not changed D – heat is absorbed Ref: AIR: atpl,
ir; HELI: all Ans: D 9048. A super cooled droplet is: A – a water droplet that is
mainly frozen B – a droplet still in liquid state at a temperature below freezing C
– a small particle of water at a temperature below 50oC D – a water droplet that
has been frozen during its descent Ref: AIR: atpl, ir; HELI: all Ans: B
9057. Super cooled droplets can be encountered: A – only in winter at high altitude
B – in winter only in high clouds C – only in winter above 10,000 ft D – at any
time of the year Ref: AIR: atpl, ir; HELI: all Ans: D 9072. Clouds, fog or dew will
always be formed when: A – relative humidity reaches 98% B – water vapour is
present C – water vapour condenses D – temperature and dew point are nearly equal
Ref: AIR: atpl, ir; HELI: all Ans: C 9073. A super cooled droplet is one that: A –
is at an above freezing temperature in below freezing air B – has frozen to become
an ice pellet C – has a shell of ice with water inside it D – remains liquid at a
below freezing temperature Ref: AIR: atpl, ir; HELI: all Ans: D 9081. How are high
level condensation trails formed that are to be found occasionally behind jet
aircraft? A – Only through un-burnt fuel in the exhaust gases B – Through a
decrease in pressure, and the associated adiabatic drop in temperature at the wing
tips while flying through relatively warm but humid air C – Through water vapour
released during fuel combustion D – In conditions of low humidity, through the
particles of soot contained in the exhaust gases Ref: AIR: atpl, ir; HELI: all Ans:
C
10186. Super cooled droplets are always: A – large and at a temperature below
freezing B – small and at a temperature below freezing C – at a temperature below
freezing D – at a temperature below -60oC Ref: AIR: atpl, ir; HELI: all Ans: C
10189. Which of the following changes of state is known as sublimation? A – Solid
direct to liquid B – Solid direct to vapour C – Liquid direct to solid D – Liquid
direct to vapour Ref: AIR: atpl, ir; HELI: all Ans: B

050-03-03 Adiabatic processes


9003. What is the dry adiabatic lapse rate per 1000 ft? A – 1.5oC B – 2.0oC C –
3.0oC D – 3.5oC Ref: AIR: atpl, ir; HELI: all Ans: C 9005. If the surface
temperature is 15oC, then the temperature at 10000 ft in a current of ascending
unsaturated air is: A – 5oC B – 0oC C - -15oC D - -5oC Ref: AIR: atpl, ir; HELI:
all Ans: C
9016. A parcel of unsaturated air is lifted to just below the condensation level
and then returned to its original level. What is the final temperature of the
parcel of air? A – Lower than the starting temperature B – Higher than the starting
temperature C – The same as the starting temperature D – It depends upon the QFE
Ref: AIR: atpl, ir; HELI: all Ans: C 9030. A parcel of moist but not saturated air
rises due to adiabatic effects. Which of the following changes? A – Specific
humidity B – Absolute humidity C – Mixing ratio D – Relatie humidity Ref: AIR:
atpl, ir; HELI: all Ans: D 9035. A sample of moist but unsaturated air may become
saturated by: A – expanding it adiabatically B – raising the temperature C –
lowering the pressure, keeping temperature constant D – compressing it
adiabatically Ref: AIR: atpl, ir; HELI: all Ans: A 9046. The decrease in
temperature, per 100 metres, in an unsaturated rising parcel of air is: A – 0.65oC
B – 2oC C – 1oC D – 0.5oC Ref: AIR: atpl, ir; HELI: all Ans: C
9058. The rate of cooling of ascending saturated air is less than the rate of
cooling of ascending unsaturated air because: A – water vapour absorbs the incoming
heat from the sun B – moist air is heavier than dry air C – water vapour does not
cool as rapidly as dry air D – heat is released during the condensation process
Ref: AIR: atpl, ir; HELI: all Ans: D 9062. If a saturated air mass descends down a
slope its temperature increases at: A – the same rate as if the air mass were dry B
– a lower rate than in dry air, as evaporation absorbs heat C – a lower rate than
in dry air, as condensation gives out heat D – a higher rate than in dry air, as it
gives up latent evaporation heat Ref: AIR: atpl, ir; HELI: all Ans: B 9076. Which
of the following statements concerning the lifting of a parcel of air is correct? A
– Unsaturated parcels cool less rapidly than saturated parcels B – Unsaturated
parcels cool more rapidly than saturated parcels C – Unsaturated parcels cool at a
rate of 0.65oC per 100m D – Saturated parcels always cool at a rate of 0.65oC per
100m Ref: AIR: atpl, ir; HELI: all Ans: B 9079. The stability in a layer is
increasing if: A – warm air is advected in the lower part and cold air in the upper
part B – warm air is advected in the upper part and cold air in the lower part C –
warm and moist air is advected in the lower part D – cold and dry air is advected
in the upper part Ref: AIR: atpl, ir; HELI: all Ans: B
9084. (Refer to figure 050-41) What can be said about this weather situation? A –
Air mass thunderstorms may develop during summertime B – Strong gradient winds may
occur over Central Europe C – Foehn conditions lead to severe weather south of the
Alps D – No ground fog will be present in Paris and Zurich during the winter Ref:
AIR: atpl, ir; HELI: all Ans: A 9085. A layer in which the temperature increases
with height is: A – conditionally unstable B – absolutely unstable C – absolutely
stable D – neutral Ref: AIR: atpl, ir; HELI: all Ans: C 9089. Which statement is
true for a conditionally unstable layer? A – The wet adiabatic lapse rate is
0.65oC/100m B – The environmental lapse rate is less than 0.65oC/100m C – The layer
is unstable for unsaturated air D – The environmental lapse rate is less than
1oC/100m Ref: AIR: atpl, ir; HELI: all Ans: D 9095. In a layer of air the decrease
in temperature per 100 metres increase in height is more than 1oC. This layer can
be described as being: A – conditionally stable B – absolutely stable C –
conditionally unstable D – absolutely unstable Ref: AIR: atpl, ir; HELI: all Ans: D
9099. (Refer to figure 050-41) What does this picture depict? A – A westerly wave
over Central Europe B – A high pressure area over Central Europe C – Uniform
pressure pattern D – North foehn Ref: AIR: atpl, ir; HELI: all Ans: C 9101. An
inversion is: A – an absolutely stable layer B – a conditionally unstable layer C –
an unstable layer D – a layer that can be either stable or unstable Ref: AIR: atpl,
ir; HELI: all Ans: A 9141. Rising air cools because: A – it becomes more moist B –
surrounding air is cooler at higher levels C – it expands D – it contracts Ref:
AIR: atpl, ir; HELI: all Ans: C 10175. If in a 100m thick layer the temperature at
the bottom of the layer is 10oC and at the top of the layer is 8oC then this layer
is: A – absolutely unstable B – absolutely stable C – conditionally unstable D –
neutral Ref: AIR: atpl, ir; HELI: all Ans: A
10177. A moist but unsaturated parcel of air becomes saturated by: A – lowering the
parcel to a lower level B – lifting the parcel to a higher level C – moving the
parcel to an area with lower pressure and equal temperature D – moving the parcel
to an area with higher pressure and equal temperature Ref: AIR: atpl, ir; HELI: all
Ans: B 10180. A layer in which the temperature remains constant with height is: A –
neutral B – unstable C – absolutely stable D – conditionally unstable Ref: AIR:
atpl, ir; HELI: all Ans: C 10181. During an adiabatic process heat is: A – neither
added nor lost B – added C – lost D – added but the result is an overall loss Ref:
AIR: atpl, ir; HELI: all Ans: C 10183. A layer is absolutely unstable if the
temperature decrease with height is: A – between 1oC per 100m and 0.65oC per 100m B
– more than 1oC per 100m C – 0.65oC per 100m D – less than 0.65oC per 100m Ref:
AIR: atpl, ir; HELI: all Ans: B
10184. The height of the lifting condensation level is determined by: A –
temperature and dew point at the surface B – temperature at surface and air
pressure C – wind and dew point at the surface D – wet adiabatic lapse rate and dew
point at the surface Ref: AIR: atpl, ir; HELI: all Ans: A 10187. The decrease in
temperature, per 100 metres, in a saturated rising parcel of air at lower level of
the atmosphere is approximately: A – 1.5oC B – 1oC C – 0.6oC D – 0.35oC Ref: AIR:
atpl, ir; HELI: all Ans: C 10202. A layer in which the temperature decreases with
1oC per 100m is: A – absolutely unstable B – absolutely stable C – neutral for dry
air D – conditionally unstable Ref: AIR: atpl, ir; HELI: all Ans: C 16588. What is
the final temperature of unsaturated surface air at 12oC, which rises to 6000 ft? A
- +30oC B - +18oC C - +30oC D - -06oC Ref: AIR: atpl, ir; HELI: all Ans: D
16592. The average value of the ELR is: A – 1.5oC / 1000 ft B – 2.0oC / 1000 ft C –
3.0oC / 1000 ft D – It varies daily Ref: AIR: atpl, ir; HELI: all Ans: B 16593. An
ELR of 2.9oC per 1000 ft, is by definition: A – stable B – conditionally stable C –
unstable D – conditionally unstable Ref: AIR: atpl, ir; HELI: all Ans: D
050-04 CLOUDS AND FOG 050-04-01 Cloud formation and description
9104. Which of the following is a cause of stratus forming over flat land? A –
Radiation during the night from the earth surface in moderate wind B – Unstable air
C – Convection during the day D – The release of latent heat Ref: AIR: atpl, ir;
HELI: all Ans: A 9105. What process in an air mass leads to the creation of wide
spread NS, AS and ST cloud coverage? A – Convection process B – Sinking C – Lifting
D – Radiation Ref: AIR: atpl, ir; HELI: all Ans: C 9108. What type of cloud is
being described? A generally grey cloud layer with fairly uniform base and uniform
appearance, which may give drizzle or snow grains. When the sun is visible through
the cloud, the outline is clearly discernible. Sometimes it appears in the form of
ragged patches. A – Stratus B – Altostratus C – Nimbostratus D – Cirrostratus Ref:
all Ans: A
9109. Which of the following clouds are classified as medium level clouds in
temperate regions? A – C1, CC B – SC, NS C – AS, AC D – CS, ST Ref: all Ans: C
9110. Which of the following types of clouds are evidence of unstable air
conditions? A – ST, CS B – CU, CB C – SC, NS D – C1, SC Ref: all Ans: B 9111.
(Refer to figure 050-02) Which one of the displayed cloud forms is representative
of altocumulus castellanus? A–B B–D C–A D–C Ref: all Ans: D 9139. Which of the
following processes within a layer of air may lead to the building of CU and CB
clouds? A – Frontal lifting within stable layers B – Radiation C – Subsidence D –
Convection Ref: AIR: atpl, ir; HELI: all Ans: D
9142. Which of the following cloud types is a medium level cloud? A – ST B – CS C –
AS D – SC Ref: all Ans: C 9143. A cumulonimbus cloud at mid latitudes in summer
contains: A – only water droplets B – ice crystals, water droplets and super cooled
water droplets C – only ice crystals D – ice crystals and water droplets but never
super cooled water droplets Ref: AIR: atpl, ir; HELI: atpl, ir Ans: B 9146. Which
of the following cloud types is found at high levels? A – SC B – C1 C – AS D – CU
Ref: all Ans: B 9151. Which of the following are medium level clouds? A –
Altostratus and altocumulus B – Cirrocumulus and cirrostratus C – Cumulonimbus D –
All convective clouds Ref: all Ans: A
9154. A plain in Western Europe with an average height of 500m (1600 ft) above sea
level is covered with a uniform SC layer of cloud during the summer months. At what
height above the ground is the base of this cloud to be expected? A – 7000 – 15000
ft above ground B – 100 – 1500 ft above ground C – 1500 – 7000 ft above ground D –
15000 – 25000 ft above ground Ref: all Ans: C 9159. What flying conditions may be
encountered when flying in cirrus clouds? A – Average horizontal visibility more
than 1000m; light to moderate rime ice B – Average horizontal visibility less than
500m; nil icing C – Average horizontal visibility less than 500m; light to moderate
icing D – Average horizontal visibility more than 1000m; nil icing Ref: AIR: atpl,
ir; HELI: atpl, ir Ans: D 9166. Clouds, classified as being low level are
considered to have bases from: A – 500 to 1000 ft B – 1000 to 2000 ft C – the
surface to 6500 ft D – 100 to 200 ft Ref: all Ans: C 9169. Which of the following
types of cloud can extend over the low, medium and high cloud levels? A – CB B – AC
C – ST D – C1 Ref: AIR: atpl, ir; HELI: atpl, ir Ans: A
9171. Altostratus clouds are classified as: A – convective clouds B – low level
clouds C – high level clouds D – medium level clouds Ref: all Ans: D 9185. Which of
the following clouds may extend into more than one layer? A – Stratus B –
Nimbostratus C – Altocumulus D – Cirrus Ref: AIR: atpl, ir; HELI: atpl, ir Ans: B
9186. Which of the following cloud is classified as low level cloud? A – ST B – CS
C – AS D – CC Ref: all Ans: A 9189. Which types of clouds are typical evidence of
stable air conditions? A – ST, AS B – CU, CB C – NS, CU D – CB, CC Ref: AIR: atpl,
ir; HELI: atpl, ir Ans: A
9191. Fall-streaks or virga are: A – water or ice particles falling out of a cloud
that evaporate before reaching the ground B – strong down draughts in the polar jet
stream, associated with jet streaks C – gusts associated with a well developed Bora
D – strong katabatic winds in mountainous areas and accompanied by heavy
precipitation Ref: AIR: atpl, ir; HELI: atpl, ir Ans: A 9193. Convective clouds are
formed: A – in stable atmosphere B – in unstable atmosphere C – in summer during
the day only D – in mid latitudes only Ref: AIR: atpl, ir; HELI: atpl, ir Ans: B
9194. (Refer to figure 050-02) Which one of the displayed cloud forms is
representative of a cumulonimbus capillatus? A–D B–A C–B D–C Ref: all Ans: A 9196.
Strongly developed cumulus clouds are an indication of: A – the presence of a low
level inversion B – instability in the atmosphere C – the presence of warm air
aloft D – poor surface visibility Ref: AIR: atpl, ir; HELI: all Ans: B
10213. (Refer to figure 050-03) Which one of the displayed cloud forms is
representative of altocumulus lenticularis? A–2 B–1 C–4 D–3 Ref: all Ans: A 10223.
In an unstable layer thee are cumuliform clouds. The vertical extent of these
clouds depends on the: A – air pressure at the surface B – wind direction C –
thickness of the unstable layer D – pressure at different levels Ref: AIR: atpl,
ir; HELI: atpl, ir Ans: C 10231. A plain in Western Europe with an average height
of 500m (1600 ft) above sea level is covered with a uniform CC layer of cloud
during the summer months. At what height above the ground is the base of this cloud
to be expected? A – 15000 – 35000 ft above the terrain B – 7000 – 15000 ft above
the terrain C – 1500 – 7000 ft above the terrain D – 100 – 1500 ft above the
terrain Ref: all Ans: A 10235. What is the main composition of clouds classified as
high level clouds? A – Super cooled water droplets B – Ice crystals C – Water
droplets D – Water vapour Ref: AIR: atpl, ir; HELI: atpl, ir Ans: B
10237. What are the characteristics of cumuliform clouds? A – Large water droplets,
stability, no turbulence, showers and mainly rime ice B – Small water droplets,
stability, no turbulence and extensive areas of rain C – Large water droplets,
instability, turbulence, showers and mainly clear ice D – Small water droplets,
instability, turbulence, extensive areas of rain and rime ice Ref: AIR: atpl, ir;
HELI: atpl, ir Ans: C 10243. A plain in Western Europe with an average elevation of
500m (1600 ft) above sea level is covered with a uniform AC layer of cloud during
the summer months. At what height above the ground is the base of this cloud to be
expected? A – 1500 – 7000 ft above the terrain B – 100 – 1500 ft above the terrain
C – 7000 – 15000 ft above the terrain D – 15000 – 25000 ft above the terrain Ref:
all Ans: C 10244. The presence of altocumulus lenticularis is an indication of the:
A – presence of valley winds B – risk or orographic thunderstorms C – development
of thermal lows D – presence of mountain waves Ref: AIR: atpl, ir; HELI: atpl, ir
Ans: D 10246. In which of the following conditions is moderate to severe airframe
icing most likely to be encountered? A – Within cloud of any type B – Below the
freezing level in clear air C- In clear air above the freezing level D – In
Nimbostratus cloud Ref: AIR: atpl, ir; HELI: atpl, ir Ans: D
10247. Cumulus clouds are an indication for: A – stability B – up and downdrafts C
– the approach of a cold front D – the approach of a warm front Ref: AIR: atpl, ir;
HELI: atpl, ir Ans: B 10248. The presence of altocumulus castellanus indicates: A –
stability in the higher troposphere B – strong convection at low height C –
instability in the middle troposphere D – subsidence in a large part of the
troposphere Ref: AIR: atpl, ir; HELI: atpl, ir Ans: C 15793. Which of the following
cloud types can project up into the stratosphere? A – Cumulonimbus B – Cirrostratus
C – Altocumulus D – Altostratus Ref: all Ans: A 16552. Which of the following is
most correct regarding the cloud types Stratus and Nimbostratus? A – Neither cloud
type may give precipitation B – Stratus may give drizzle, ice prisms or snow grains
and Nimbostratus may give rain showers C – Stratus may give rain showers and
Nimbostratus may give drizzle, ice prisms or snow grains D – Stratus may give
drizzle, ice prisms or snow grains and Nimbostratus may give continuously falling
rain or snow Ref: all Ans: D
16553. Altostratus (AS) and Nimbostratus (NS) are easily confused. How do you
distinguish between them? A – The cloud base is higher in AS and precipitation, if
any, is light B – Precipitation falls from AS but not from NS C – Steady
precipitation from AS D – The sun can be seen through NS Ref: all Ans: A 16554.
Which one of the following cloud types gives steady rain or snowfall? A –
Altostratus B – Nimbostratus C – Cirrostratus D – Cumulonimbus Ref: all Ans: B
16612. The presence of altocumulus lenticularis is a sign of: A – severe
instability B – anabatic winds C – katabatic winds D – lee waves Ref: AIR: atpl,
ir; HELI: atpl, ir Ans: D 24114. (Refer to figure 050-66) The cloud most likely to
be experienced in square 1E is: A – CS B – C1 C – AS D – CB Ref: all Ans: B
24115. (Refer to figure 050-34) The cloud type most applicable to square 2D is: A –
AS B – CU C – CB D – CS Ref: all Ans: A 24116. (Refer to figure 050-34) The cloud
type most applicable to square 3C is: A – NS B – AC C – AS D – CB Ref: all Ans: A
24118. (Refer to figure 050-34) During summer, the cloud type most applicable to
square 2A is: A – ST B – AC C – CB D – CS Ref: all Ans: C 24121. (Refer to figure
050-34) The cloud type most applicable to most of square 3B is: A – SC B – CS C –
AS D – NS Ref: all Ans: A
24129. (Refer to figure 050-58) The cloud type most applicable to square 1E is: A –
CS B – CB C – NS D – SC Ref: all Ans: A 24130. (Refer to figure 050-58) The cloud
type most applicable to square 2C is: A – CS B – AS C – CB D – CU Ref: all Ans: B
24131. (Refer to figure 050-58) The cloud type most applicable to square 2B is: A –
ST B – CS C – CB D – SC Ref: all Ans: C 24209. A cumulus congestus is: A – a
remnant of a CB B – a cumulus that is of great vertical extent C – a cumulus with
little vertical development D – a cumulus that only occurs in association with the
ITCZ Ref: all Ans: B
24213. A layer of stratus is most likely to be dispensed by: A – adiabatic cooling
due to subsidence B – absorption of long waved solar radiation in the stratus layer
C – the release of latent heat due to precipitation D – insulation resulting in the
lifting of the condensation level Ref: all Ans: D 24220. About ten identical clouds
are in the sky, well isolated from one another, dense, with well defined contours,
developing vertically in a cauliflower shape. The side of these clouds lit by the
sun is bright white. Their base, relatively dark, is essentially horizontal and at
FL 30, and their tops at FL 150. These clouds are: A – Altocumulus castellanus B –
broken Cumulus humilis C – towering Cumulus D – Stratocumulus Ref: all Ans: C
24225. After a clear night cumuliform clouds are formed in the morning. Why can the
base of these clouds become higher during the day? A – Because the difference
between the temperature and the dew point temperature at the initial condensation
level becomes smaller B – The wind speed is increasing, because the cold air mass
changes into a warm air mass C – Because the stability increases D – Because the
surface temperature increases Ref: all Ans: D
24254. Clouds in patches, sheets or grey or whitish layers made up of elements
resembling large pebbles or rollers, together or not, and always clear of the
ground are: A – Altostratus B – Stratus C – Stratocumulus D – Nimbostratus Ref: all
Ans: C 24255. Clouds will mainly consist of super cooled water droplets when the
temperature is: A – between -5oC and -30oC B – between 0oC and -15oC C – between
-30oC and -40oC D – below -40oC Ref: AIR: atpl, ir: HELI: all Ans: B 24303.
Lenticular clouds in mountainous areas indicate: A – unstable air B – turbulence C
– an inversion D – light variable winds Ref: AIR: atpl, ir: HELI: atpl, ir Ans: B
24306. Of what does lenticular cloud provide evidence? A – Jet streams B – Mountain
waves C – Stratospheric inversions D – Areas of high level clear air turbulence
Ref: AIR: atpl, ir: HELI: atpl, ir Ans: B
25578. (Refer to figure 050-02) Which one of the displayed cloud forms is
representative of altocumulus astellanus? A–A B–B C–C D–D Ref: all Ans: C 25622.
(Refer to figure 050-01) Which one of the displayed cloud forms is representative
of altocumulus astellanus? A–1 B–2 C–3 D–4 Ref: all Ans: C

050-04-02 Fog, mist, haze


9107. Which of the following conditions is most likely to lea to the formation of
advection fog? A – Moist cold air moving over a warm surface B – Moist warm air
moving over a cold surface C – Dry warm air moving over a cold surface D – Dry cold
air moving over a warm surface Ref: all Ans: B 9112. Which of the following is most
likely to lead to the formation of radiation fog? A – Dry, warm air passing over
warm ground B – Heat loss from the ground on clear nights C – The passage of fronts
D – Cold air passing over warm ground Ref: all
Ans: B 9113. Which of the following circumstances most favour the development of
radiation fog? A – Warm moist air at the windward side of a mountain B – Moist air
over land during clear night with little wind C – Maritime tropical air flowing
over cold sea D – Advection of very cold air over much warmer sea Ref: all Ans: B
9114. What is the average vertical extent of radiation fog? A – 2000 ft B – 500 ft
C – 5000 ft D – 10000 ft Ref: all Ans: B 9116. Frontal fog is most likely to occur:
A – in winter in the early morning B – in rear of a warm front C – in summer in the
early morning D – in advance of a warm front Ref: all Ans: D 9118. Which of the
following conditions is most likely to lead to the formation of steam fog (arctic
smoke)? A – Warm air moving over cold water B – Cold air moving over warm water C –
The sea is warmed by strong radiation from the sun D – The coastal region of the
sea cools at night Ref: all Ans: B
9119. At what time of day, or night, is radiation fog most likely to occur? A – At
sunset B – Shortly after sunrise C – Late evening D – Shortly after midnight Ref:
all Ans: B 9123. The range of wind speed in which radiation fog is most likely to
form is: A – above 15 kt B – between 10 and 15 kt C – between 5 and 10 kt D – below
5 kt Ref: all Ans: D 9124. When does frontal fog, also known as mixing fog, occur?
A – When very humid warm air meets with dry cold air B – When very humid warm air
meets with very humid cold air C – When very humid cold air meets with dry warm air
D – When very dry cold air meets with very dry warm air Ref: all Ans: B 9130.
Freezing fog consists of: A – frozen water droplets B – super cooled water droplets
C – frozen minute snow flakes D – ice crystals Ref: all Ans: B
9132. Which of the following statements is true concerning advection fog? A – It
forms slowly and disappears rapidly B – It forms at night or the early morning C –
It forms when unstable air is cooled adiabatically D – It can be formed suddenly by
day or night Ref: all Ans: D 9133. What conditions are most likely to lead to the
formation of hill fog? A – Humid stable air mass, wind blowing towards the hills B
– High relative humidity and an unstable air mass C – Clear skies, calm or light
winds, with relatively low humidity D – Precipitation which is lifted by the action
of moderate winds striking the range Ref: all Ans: A 9155. The morning following a
clear, calm night when the temperature has dropped to the dew point, is likely to
produce: A – a cold front B – radiation fog C – advection fog D – good clear
weather Ref: all Ans: B 9179. When the temperature and dew point are less than one
degree apart the weather conditions are most likely to be: A – unlimited visibility
B – clear and cool C – high scattered clouds D – fog or low cloud Ref: all Ans: D
9183. What wind conditions, occurring just before dawn, favour the formation of fog
at an airport where the temperature is 15oC and the dew point is 14oC? A –
Westerly, 10 kt variable B – Northerly, 10 kt C – Calm D – Easterly, 10 kt Ref: all
Ans: C 9188. The most likely reason for radiation fog to dissipate or become low
stratus is: A – increasing surface wind speed B – an increasingly stable atmosphere
C – surface cooling D – a low level temperature inversion Ref: all Ans: A 9198.
Which of the following is most likely to lead to the dissipation of radiation fog?
A – A build up of a high pressure area resulting in adiabatic warming associated
with a sinking air mass B – A marked decrease in wind velocity close to the ground
C – Ground cooling caused by radiation during the night D – A marked increase in
wind velocity near the ground Ref: all Ans: D 10209. Steaming fog (arctic sea
smoke) occurs in air: A – that is stable B – with warm mass properties C – that is
absolutely stable D – with cold mass properties Ref: all Ans: D
10212. Which type of fog is likely to form when air having temperature of 15oC and
dew point of 12oC blows at 10 knots over a sea surface having temperature of 5oC? A
– Radiation fog B – Advection fog C – Steam fog D – Frontal fog Ref: all Ans: B
10218. Which of the following weather conditions favour the formation of radiation
fog? A – Light wind, little or no cloud, moist air B – Light wind, extensive cloud,
dry air C – Light wind, extensive cloud, moist air D – Strong wind, little or no
cloud, moist air Ref: all Ans: A 10241. Advection fog can be formed when: A – cold
moist air flows over warmer water B – cold moist air flows over a warmer surface C
– warm moist air flows over a warmer surface D – warm moist air flows over a colder
surface Ref: all Ans: D 10242. Under which of these conditions is radiation fog
most likely to form? A – Very dry air B – Little or no cloud C – Strong surface
winds D – Very low temperatures Ref: all Ans: B
10245. What type of fog is most likely to form over flat land during a clear night,
with calm or light wind conditions? A – Radiation B – Advection C – Steam D –
Orographic Ref: all Ans: A 15786. (Refer to figure 050-04) Of the four radio
soundings, select the one that indicates ground fog: A–3 B–3 C–1 D–4 Ref: all Ans:
B 16545. Which one of the following statements regarding the meteorological
horizontal visibility is the correct one? A – If the visibility varies in different
directions, the visibility in the worst direction is reported B – If the visibility
varies in different directions, an average value of around the horizon is reported
C – The visibility is improved when darkness falls, because you distinguish a point
of light more easily than an unlighted mark D – Fog (FG) is used to describe
visibility less than 1000 m except when qualified by MI, BC, PR or VC Ref: all Ans:
D
16579. For (FG) is defined as being a: A – visibility of < 1,000m due to any
precipitation in the atmosphere B – visibility of < = 1,000m due to liquid
particles or ice crystals suspended in the atmosphere C – visibility of < 1,000m
due to liquid particles or ice crystals suspended in the atmosphere D – visibility
of < 1,000m due to solid or liquid particles suspended in the atmosphere Ref: all
Ans: C 16580. Mist (BR) is defined as being: A – reduced visibility > 1,000m but
not more than 5,000m due to the presence of water droplets in the atmosphere B –
reduced visibility > 1,000m but not more than 5,000m due to solid or liquid
particles in the atmosphere C – reduced visibility > = 1,000m but not more than
5,000m due to solid or liquid particles in the atmosphere D – reduced visibility >
= 1,000m but not more than 5,000m due to the presence of water droplets in the
atmosphere Ref: all Ans: D 16581. According to ICAO, haze (HZ) or smoke (FU), is
reduced visibility due to the presence of solid particles (lithometeors) in the
atmosphere to a value of: A - < 5,000 m B - < = 5,000m C - > 5,000m D - > = 5,000m
Ref: all Ans: B
16582. Conditions favourable for the development of radiation fog (FG) are: A –
high relative humidity, no cloud B – high relative humidity, little or no cloud C –
high relative humidity, little or no cloud, a strong sea breeze D – high relative
humidity, little or no cloud, little wind (2-8 kts or calm) Ref: all Ans: D 16583.
Which of the following statements is true? A – Radiation fog only forms on a clear
night with no wind B – Radiation fog only forms in valleys (the katabatic effect) C
– Radiation fog only forms when the air is very dry and there is an inversion D –
Radiation fog cannot form over the sea Ref: all Ans: D 16584. Radiation fog can be
dispersed by: A – insulation B – strong winds C – replacement of moist air by drier
air D – all of the above Ref: all Ans: D 16585. Advection fog is caused by: A – dry
air over a cold surface B – cold air over a warm moist surface C – air, cooler by
at least 10oC, moving over a moist surface D – none of the above Ref: all Ans: C
16586. Sea fog: A – is formed by advection B – can form a persist with moderate or
even strong wind C – is cleared by change of air mass or less commonly by an
increase in wind strength D – all of the above Ref: all Ans: D 24223. Advection fog
is most likely to form when: A – cold maritime air flows over a warmer surface and
the wind speed is greater than 15 kt B – cold air is forced over higher ground and
further adiabatic cooling occurs C – a mild moist air stream flows over snow
covered ground and the wind speed is less than 10 kt D – warm maritime air flows
over a relatively warmer surface and the wind speed is greater than 15 kt Ref: all
Ans: C 24224. Advection of very cold air over a warm sea current can cause: A –
frontal fog B – advection fog C – no fog at all D – steaming fog Ref: all Ans: D
24245. As a result of diurnal variation radiation fog is lifted and a cloud cover
is formed. Which statement is true? A – Layers of alto cumulus will develop caused
by increasing wind speed B – Low stratus will develop caused by radiation in
combination with low geostrophic wind speed C – Low stratus will develop caused by
increasing wind speed D – Stratocumulus will develop caused by insulation Ref: all
Ans: C
24272. Frontal fog can be formed by: A – cooling at night B – condensation of air
saturated by adiabatic cooling C – condensation of air saturated by evaporation of
precipitation D – evaporation of moisture at the surface Ref: all Ans: C 24279. If
radiation fog forms on a clear night with light winds, the increase in wind speed
from 5 kt to 13 kt will most likely: A – disperse the fog immediately B – change
the radiation fog to advection fog C – have no effect D – cause the fog to lift and
become low stratus Ref: all Ans: D 24300. In which situation is advection fog most
likely to form? A – A light breeze blowing colder air out to sea B – A warm moist
air mass on the windward side of the mountains C – An air mass moving inland from
the coast in winter D – Warm moist air settling over a warmer surface under no-wind
conditions Ref: all Ans: C 24314. Radiation fog most frequently occurs in: A – low
pressure systems over sea B – high pressure systems over land C – high pressure
systems over sea D – low pressure systems over land Ref: all Ans: B
24335. The formation of morning fog before sunrise is possible if A – air
temperature and dew point are equal or close to one another B – the wind is strong
C – the sky is overcast D – the turbulence in the lower layers is moderate Ref: all
Ans: A 24375. What is the difference between radiation fog and advection fog? A –
Radiation fog is formed by surface cooling in a calm wind. Advection fog is formed
by evaporation over the sea B – Radiation fog forms only on the ground, advection
fog only on the sea C – Radiation for forms due to night cooling and advection fog
due to day time cooling D – Radiation for forms due to surface cooling at night in
a light wind. Advection fog forms when warm humid air flows over a cold surface
Ref: all Ans: D 24387. What kind of fog is often observed in the coastal region of
Newfoundland in spring time? A – Radiation fog B – Frontal fog C – Advection fog D
– Steaming fog Ref: all Ans: C 24415. Which of the following layers of fog above
land is coded as MIFG? A – A layer of 5 feet deep B – A layer of 10 feet deep C – A
layer of 15 feet deep D – A layer of 20 feet deep Ref: all Ans: A
24418. Which of the following phenomena is least likely to lead to the formation of
a Cumulonimbus with thunderstorm? A – Convection B – Ground radiation C –
Convergence D – Orographic lift Ref: all Ans: B 24422. Which of the following sets
of conditions are most likely to lead to the formation of advection fog? A – Cold
maritime air flowing over a warmer land surface at a speed greater than 15 kt B –
Clear skies at night over an inland marshy area C – A mild moist air stream flowing
over colder surfaces with a wind in excess of 30 kt D – A mild moist air stream
flowing over colder surfaces with the wind speed less than 15 kt Ref: all Ans: D
24448. Which statement is correct? A – Fog can be super cooled and can also contain
ice crystals B – Mist and haze consist of water droplets C – Fog and haze do not
occur in the tropics D – Mist and haze only differ by different values of
visibility Ref: all Ans: A 24451. Which type of fog can NOT be formed over water? A
– Advection fog B – Radiation fog C – Arctic smoke D – Frontal fog Ref: all Ans: B
24498. Which is true of advection fog? A – It can appear suddenly by day or by
night B – It develops slowly and clears fast C – It forms when unstable air is
adiabatically cooled D – It usually forms by night and clears by day Ref: all Ans:
A
050-05

PRECIPITATION

050-05-01 Development of precipitation


9203. The presence of ice pellets at the surface is evidence that: A – freezing
rain occurs at a higher altitude B – a cold front has passed C – there are
thunderstorms in the area D – a warm front has passed Ref: all Ans: A 9205. Which
form of precipitation from clouds containing only water is most likely to fall in
mid-latitudes? A – Hail B – Moderate rain with large drops C – Heavy rain with
large drops D – Drizzle Ref: all Ans: D 9227. How does freezing rain develop? A –
Rain falls on cold ground and then freezes B – Rain falls through a layer where
temperatures are below 0oC C – Through melting of sleet grains D – Through melting
of ice crystals Ref: all Ans: B 24357. The widest precipitation zone occurs
usually: A – ahead of a warm front B – ahead of a cold front C – in rear of a cold
front D – in rear of a warm front Ref: all Ans: A
050-05-02 Types of precipitation
9202. Freezing precipitation occurs: A – only in the precipitation of a warm front
B – only in the precipitation of a cold front C – mainly in the form of freezing
rain or freezing drizzle D – mainly in the form of freezing hail or freezing snow
Ref: all Ans: C 9204. What type of cloud can produce hail showers? A – CS B – NS C
– CB D – AC Ref: all Ans: C 9206. With what type of cloud is GR precipitation most
commonly associated? A – CC B – AS C – ST D – CB Ref: all Ans: D 9207. From what
type of cloud does drizzle fall? A – Stratus B – Altostratus C – Cumulus D –
Cirrostratus Ref: all Ans: A
9215. Freezing rain occurs when: A – snow falls into an above freezing layer of air
B – ice pellets melt C – water vapour first turns into water droplets D – rain
falls into a layer of air with temperatures below 0oC Ref: all Ans: D 9216. With
what type of cloud is +TSRA precipitation most commonly associated? A – CB B – AS C
– SC D – NS Ref: all Ans: A 9218. Which of the following are favourable conditions
for the formation of freezing rain? A – Warm air aloft from which rain is falling
into air with a temperature below 0oC B – Water droplets falling from cold air
aloft with a temperature below 0oC C – Cold air aloft from which hail is falling
into air that is warm D – An isothermal layer aloft with a temperature just above
0oC through which rain is falling Ref: all Ans: A 9219. With what type of cloud is
heavy precipitation unlikely during the summer months? A – NS, CC B – CB, ST C –
SC, AS D – AS, NS Ref: all Ans: C
9221. What type of clouds are associated with snow showers? A – Nimbostratus B –
Cumulus and altostratus C – Altostratus and stratus D – Cumulus and cumulonimbus
Ref: all Ans: D 9222. Large hail stones: A – only occur in thunderstorms of mid
latitudes B – are typically associated with severe thunderstorms C – are entirely
composed of clear ice D – only occur in frontal thunderstorms Ref: all Ans: B 9223.
Precipitation in the form of showers occurs mainly from: A – clouds containing only
ice crystals B – stratified clouds C – cirro-type clouds D – convective clouds Ref:
all Ans: D 9224. What type of clouds are associated with rain showers? A –
Nimbostratus B – Towering cumulus and altostratus C – Altostratus and stratus D –
Towering cumulus and cumulonimbus Ref: all Ans: D
9228. Which one of the following types of cloud is most likely to produce heavy
precipitation? A – SC B – CS C – NS D – ST Ref: all Ans: C 9229. With what type of
clouds are showers most likely associated? A – Stratocumulus B – Cumulonimbus C –
Nimbostratus D – Stratus Ref: all Ans: B 9230. With what type of cloud is DZ
precipitation most commonly associated? A – CB B – ST C – CC D – CU Ref: all Ans: B
9231. With which of the following types of cloud is +RA precipitation most commonly
associated? A – NS B – AC C – SC D – ST Ref: all Ans: A
9232. The following statements deal with precipitation, turbulence and icing.
Select the list containing the most likely alternatives for NS cloud: A –
Precipitation may be snow, sleet or rain. Icing is probable and may range between
light and severe. Turbulence is rarely more than moderate B – Precipitation may be
snow, sleet or rain. Icing and turbulence are frequently severe C – Precipitation
is frequently in the form of hail. Icing and turbulence are frequently severe D –
Precipitation and icing are usually nil. Turbulence is rarely more than moderate
Ref: all Ans: A 9233. Steady precipitation, in contrast to showery precipitation
falls from: A – stratiform clouds with severe turbulence B – convective clouds with
little or no turbulence C – stratiform clouds with little or no turbulence D –
convective clouds with moderate turbulence Ref: all Ans: C 9235. Which of the
following cloud types is least likely to produce precipitation? A – C1 B – AS C –
CB D – NS Ref: all Ans: A 16578. Which precipitation type generally has the
greatest impact on visibility? A – Heavy rain B – Drizzle C – Hail D – Snow Ref:
all Ans: D
24275. Hazardous hailstones, reaching the ground, are most likely to be experienced
below CB clouds situated: A – over the sea in middle latitudes B – over the sea
near the equator C – in continental interiors near the equator D – in continental
interiors in middle latitudes Ref: all Ans: D 24319. Super cooled droplets can
occur in: A – clouds, fog and precipitation B – clouds but not in precipitation C –
precipitation but not in clouds D – clouds but not in fog Ref: all Ans: A
050-06

AIR MASSES AND FRONTS 050-06-01 Types of air masses

9236. Where does polar continental air originate? A – The region of the Baltic sea
B – Areas of arctic water C – Siberian landmass D – The region of Greenland Ref:
all Ans: C 9270. (Refer to figure 050-47) What is the classification of the air
mass affecting North-Western parts of France at 0600 UTC? A – Tropical continental
B – Polar maritime C – Polar continental D – Tropical maritime Ref: all Ans: D
9282. An air mass is unstable when: A – temperature increases with height B –
temperature and humidity are not constant C – pressure shows a marked variation
over a given horizontal area D – an ascending parcel of air continues to rise to a
considerable height Ref: all Ans: D
9285. What are the typical differences between the temperature and humidity between
an air mass with its origin in the Azores and an air mass with its origin over
northern Russia? A – The air of the Azores is warmer and more humid than the North
Russian air B – The North Russian air is colder and more humid than the air of the
Azores C – The air of the Azores is warmer and dryer than the North Russian air D –
The North Russian air is warmer and dryer than the air of the Azores Ref: all Ans:
A 9304. In which air mass are extremely low temperatures encountered? A – Arctic
maritime air B – Polar maritime air C – Tropical continental air D – Polar
continental air Ref: all Ans: D 9310. An air mass is stable when: A – pressure is
constant B – temperature in a given area drops off very rapidly with height C – the
vertical motion of a rising parcel of air tends to become weaker and disappears D –
the lapse rate is 1oC per 100m Ref: all Ans: C 9324. Where is the source of
tropical continental air that affects Europe in summer? A – The Azores region B –
Southern Italy C – Southern France D – The southern Balkan region and the Near East
Ref: all Ans: D
10264. In which of the following regions does polar maritime air originate? A –
Baltic Sea B – Region of British Isles C – East of Greenland D – Black Sea Ref: all
Ans: C 16429. A stable humid warm air mass slowly over rides a cold one. Which one
of the following weather types may originate from this condition? A – The formation
of thunder storms B – Warm front dissipation C – The forming of a warm front D –
The forming of a cold front Ref: all Ans: C 16431. Which of these phenomena usually
forms in the transition zone between two air masses? A – an unstable low pressure B
– radiation fog C – a frontal low pressure D – a ridge of high pressure Ref: all
Ans: C 24217. A stationary front is a front in which: A – there are never frontal
clouds B – there is no difference in temperature between the two air masses C –
there is no wind on both sides of the front D – there is no horizontal motion
perpendicular to the front Ref: all Ans: D
24228. Air masses which are being cooled from below are often characterised by: A –
fog, poor visibility and layered clouds B – strong winds, cumulus clouds, good
visibility C – uniform temperature, good visibility D – continuous rain and
freezing temperature Ref: all Ans: A 24230. An air mass acquires its basic
properties A – by widespread thunderstorms B – by the influence of jet streams C –
by stagnation of the air for a long period of time over areas having particular
characteristics D – in the westerlies of the mid latitudes Ref: all Ans: C 24231.
An air mass is stable when A – lifted air returns to its original level B –
temperature in a given area drops off very rapidly with height C – pressure is
constant D – the lapse rate is 1oC per 100m Ref: all Ans: A 24232. An air mass is
unstable when A – pressure shows a marked variation over a given horizontal area B
– temperature and humidity are not constant C – an ascending parcel of air
continues to rise to a considerable height D – temperature increases with height
Ref: all Ans: C
24233. An air mass is: A – an extensive body of air within which the temperature
and humidity in horizontal planes are practically uniform B – a large body of air
with temperature and humidity constant in the vertical C – a body of air with a
volume of not more than thousand cubic kilometres D – a large body of air within
which the temperature and humidity is uniform in horizontal and vertical planes
Ref: all Ans: A 24243. An unstable air mass is forced to ascend a mountain slope.
What type of clouds can be expected? A – Stratiform clouds with a temperature
inversion B – Stratiform clouds with considerable turbulence C – Layer like clouds
with little vertical development D – Clouds with considerable vertical development
and associated turbulence Ref: all Ans: D 24244. An unstable air mass will normally
be characterised by A – stratiform cloud B – cumuliform cloud and good visibility
except in precipitation C – continuous light rain from medium level layer cloud D –
poor visibility due to haze at the lower levels Ref: all Ans: B
24276. How does relative humidity and the dew point in an unsaturated air mass
change with varying temperature? A – When temperature increases, the relative
humidity decreases, and the dew point remains constant B – When temperature
increases, the relative humidity increases, and the dew point decreases C – When
temperature decreases, the relative humidity decreases, and the dew point increases
D – When temperature decreases, the relative humidity and the dew point remain
constant Ref: all Ans: A 24419. Which of the following processes will increase the
stability of an air mass? A – Addition of water vapour in the lower layer B –
Cooling by the underlying surface C – Warming of the air mass from below D –
Advection of colder air aloft Ref: all Ans: B

050-06-02 Fronts
9238. What weather conditions are prevalent during the summer, over the North Sea,
approximately 300 km behind a quickly moving cold front? A – Cloud cover mostly
scattered, isolated showers B – Showers and thunderstorms C – Rain covering a large
area, 8 octas NS D – 9 octas CS, AS without precipitation Ref: all Ans: A
9240. In a warm front occlusion: A – the warm front becomes a front aloft B – the
warm front overtakes the cold front C – the warm air is lifted D – the cold air is
lifted Ref: all Ans: C 9241. Over Central Europe what type of cloud cover is
typical of the warm sector of a depression during winter? A – C1, CS B – ST with
drizzle C – Fair weather CU D – CU, CB Ref: all Ans: B 9247. Which of the following
is typical for the passage of a cold front in the summer? A – Rapid increase in
temperature once the front has passed B – Mainly layered clouds C – Rapid drop in
pressure once the front has passed D – Mainly towering clouds Ref: all Ans: D 9251.
Frontal depressions can be assumed to move in the direction of the 2000 feet wind:
A – in the warm sector B – in front of the warm front C – behind the cold front D –
at the apex of the wave Ref: all Ans: A
9252. If you have to fly through a warm front when freezing level is at 10000 feet
in the warm air and at 2000 feet in the cold air, at which altitude is the
probability of freezing rain the lowest? A – 9000 feet B – 12000 feet C – 5000 feet
D – 3000 feet Ref: all Ans: B 9253. How are the air masses distributed in a cold
occlusion? A – The coldest air mass behind and the less cold air in front of the
occlusion; the warm air mass is above ground level B – The coldest air in front of
and the less cold air is behind the occlusion; the warm air mass is above ground
level C – The coldest air in front of and the warm air behind the occlusion; the
less cold air is above ground level D – The coldest air behind and the warm air in
front of the occlusion; the less cold air mass is above ground level Ref: all Ans:
A 9254. Which of the following describes a warm occlusion? A – The air mass ahead
of the front is drier than the air mass behind the front B – The air mass behind
the front is more unstable than the air mass ahead of the front C – The coldest air
mass is ahead of the original warm front D – The warmer air mass is ahead of the
original warm front Ref: all Ans: C
9255. In Zurich during a summer day the following weather observations were taken:
160450Z 23015KT 3000 + RA SCT008 SCT020 OVC030 13/12 Q1010 NOSIG = 160650Z 25008KT
6000 SCT040 BKN090 18/14 Q1010 RERA NOSIG = 160850Z 25006KT 8000 SCT040 SCT100
19/15 Q1009 NOSIG = 161050Z 24008KT 9999 SCT040 SCT100 21/15 Q1008 NOSIG = 161250Z
23012KT CAVOK 23/16 Q1005 NOSIG = 161450Z 23016KT 9999 SCT040 BKN090 24/17 Q1003
BECMG 25020G40KT TS = 161650Z 24018G35KT 3000 +TSRA SCT006 BKN015CB 18/16 Q1002
NOSIG = 161850Z 28012KT 9999 SCT030 SCT100 13/11 Q1005 NOSIG = What do you conclude
based on these observations? A – A cold front passed the station early in the
morning and a warm front during late afternoon B – A warm front passed the station
early in the morning and a cold front during late afternoon C – A trough line
passed the station early in the morning and a warm front during late afternoon D –
Storm clouds due to warm air came close to and grazed the station Ref: all Ans: B
9258. Which of the following conditions are you most likely to encounter when
approaching an active warm front at medium to low level? A – Severe thunderstorms
at low altitude B – Low cloud base and poor visibility C – Extreme turbulence and
severe lightning striking the ground D – High cloud base, good surface visibility,
and isolated thunderstorms Ref: all Ans: B
9261. Where is the coldest air to be found, in an occlusion with cold front
characteristics? A – Ahead of the front B – Behind the front C – At the surface
position of the front D – At the junction of the occlusion Ref: all Ans: B 9262.
What types of cloud will you meet flying towards a warm front? A – Extensive areas
of fog. At some 100 km from the front NS begin B – At some 800 km CS, later AS, and
at some 300 km NS until the front C – At some 500 km AS, later CS and at some 80 km
before the front CB D – At some 500 km from the front, groups of CB, later at some
250 km thickening AS Ref: all Ans: B 9265. What type of fronts are most likely to
be present during the winter in Central Europe when temperatures close to the
ground are below 0oC, and freezing rain starts to fall? A – Cold occlusions B –
Warm fronts, warm occlusions C – High level cold fronts D – Cold fronts Ref: all
Ans: B 9266. (Refer to figure 050-09) Examining the pictures, on which one of the
tracks (dashed lines) is this cross section to be expected? A – Track B-C B – Track
B-D C – Track A-D D – Track A-E Ref: all Ans: B
9268. What type of front/occlusion usually moves the fastest? A – Cold front B –
Warm front C – Cold occlusion D – Warm occlusion Ref: all Ans: A 9272. (Refer to
figure 050-47) This chart shows the weather conditions on the ground at 0600 UTC on
May 4. Which of the following reports reflects weather development at Geneva
Airport? A – TAF LSGG 230716 05014KT 5000 OVC015 BECMG 0810 8000 BKN 018 BECMG 1013
05015G30KT 9999 SCT025= B – TAF LSGG 230716 23016KT 8000 –RA BKN030 OVC070 BECMG
0810 5000 RA BKN 020 OVC050 TEMPO 3000 +RA BKN010 OVC030 BECMG 1215 25014KT 8000
SCT030 BKN090= C – TAF LSGG 230716 26012KT 9999 SCT030 BKN080 TEMPO 1013 25020G35KT
3000 TSRA or +SHRA BKN030CB BECMG 1316 VRB02KT 3000 BCFG SCT100= D – TAF LSGG
230716 VRB03KT 6000 BR SCT020 BECMG 0811 23005KT 9999 SCT025TCU PROB 40 TEMPO 1216
34012G30KT 3000 TSRA BKN020CB = Ref: all Ans: B 9274. During a cross-country flight
at FL 50, you observe the following sequence of clouds: Nimbostratus, Altostratus,
Cirrostratus, Cirrus. Which of the following are you most likely to encounter? A –
Strong, gusty winds B – Decreasing temperatures C – A strong down draught D –
Increasing temperatures Ref: all Ans: B
9275. In a polar front depression, an occlusion is called a warm occlusion when the
cold air: A – behind is less cold than the cold air in front, with the warm air at
a high altitude B – in front of the surface position of front is only at a high
altitude C – behind is colder than the cold air in front D – behind is colder than
the cold air in front, with the warm air being at a high altitude Ref: all Ans: A
9287. An observer on the northern hemisphere is under influence of the wind system
of a depression, which is moving from West to East. The centre of the depression
passes to the South of the observer. For this observer the wind direction is: A –
continuously veering B – continuously backing C – initially backing, then veering D
– initially veering, then backing Ref: all Ans: B 9291. (Refer to figure 050-103)
Where might we find the warmest air? A–B B–C C–A D–D Ref: all Ans: A
9298. What will be the effect on the reading of an altimeter of an aircraft parked
on the ground as an active cold front is passing? A – It will first increase then
decrease B – It will remain unchanged C – It will first decrease then increase D –
It will fluctuate up and down by about +/- 50 feet Ref: all Ans: A 9303. What
characterises a stationary front? A – The surface wind usually has its direction
perpendicular to the front B – The surface wind usually has its direction parallel
to the front C – The warm air moves at approximately half the speed of the cold air
D – The weather conditions that it originates is a combination between those of an
intense cold front and those of a warm and very active front Ref: all Ans: B 9306.
(Refer to figure 050-08) Which one of the tracks (dashed lines) is represented by
the cross section shown on the left? A – Track B-A B – Track C-A C – Track D-A D –
Track B-C Ref: all Ans: C 9307. When do cold occlusions occur more frequently in
Europe? A – Summer B – Winter C – Autumn and winter D – Winter and spring Ref: all
Ans: A
9309. (Refer to figure 050-104) What change in pressure, will occur at point A,
during the next hour? A – Approximately constant pressure B – A drop in pressure C
– Irregular fluctuations D – A rise in pressure Ref: all Ans: D 9312. The
approximate inclined plane of a warm front is: A – 1/50 B – 1/150 C – 1/300 D –
1/500 Ref: all Ans: B 9314. What is the relative movement of the two air masses
along a cold front? A – Cold air slides over a warm air mass B – Warm air pushes
over a cold air mass C – Cold air pushes under a warm air mass D – Warm air pushes
under a cold air mass Ref: all Ans: C 9319. What type of low pressure area is
associated with a surface front? A – A low on lee side of a mountain B – A cold air
pool C – Polar front low D – Heat low Ref: all Ans: C
9320. At what time of the year, are the paths of north Atlantic lows moving from
west to east generally, at their most southerly position? A – Autumn B – Summer C –
Spring D – Winter Ref: all Ans: D 9322. What is the surface visibility most likely
to be, in a warm sector of tropical maritime air, during the summer? A – Very good
(greater than 50 km) B – Very poor (less than 1 km) C – Good (greater than 10 km) D
– Moderate (several km) Ref: all Ans: D 9328. On an aerodrome, when a warm front is
approaching: A – QFE increases and QNH decreases B – QFE and QNH increase C – QFE
and QNH decrease D – QFE decreases and QNH increases Ref: all Ans: C 9330. In which
approximate direction does the centre of a frontal depression move? A – In the
direction of the warm sector isobars B – In the direction of the isobars ahead of
the warm front C – In the direction of the sharpest pressure increase D – In the
direction of the isobars behind the cold front Ref: all Ans: A
9332. How do air masses move at a warm front? A – Cold air over rides a warm air
mass B – Warm air over rides a cold air mass C – Cold air undercuts a warm air mass
D – Warm air undercuts a cold air mass Ref: all Ans: B 9333. (Refer to figure 050-
11) Assuming the usual direction of movement, to which position will the polar
frontal wave have moved? A – Position 4 B – Position 3 C – Position 1 D – Position
2 Ref: all Ans: B 9334. What cloud cover is typical for a wide warm sector of a
polar front depression over Central Europe in the summer? A – BKN CU and CB B –
Fair weather CU C – Sky clear D – ST with drizzle Ref: all Ans: B 10250. What will
be the effect on the reading of an altimeter of an aircraft parked on the ground
during the period following the passage of an active cold front? A – It will have
increased B – It will remain unchanged C – It will have decreased D – It will show
a small increase or decrease Ref: all Ans: C
10258. In which of the following situations can freezing rain be encountered? A –
Ahead of a warm front in the winter B – Ahead of a cold front in the winter C –
Behind a warm front in the summer D – Ahead of a cold front in the summer Ref: all
Ans: A 10262. In which main direction does a polar front depression move? A – Along
the front towards the east B – Along the front towards the west C – Across the
front towards the north D – Across the front towards the south Ref: all Ans: A
10270. The polar front is the boundary between: A – maritime polar aid and
continental polar air B – arctic air and polar air C – arctic air and tropical air
D – polar air and tropical air Ref: all Ans: D 10271. What cloud formation is most
likely to occur at low levels when a warm air mass over rides a cold air mass? A –
Cumulus B – Nimbostratus C – Altostratus D – Cumulonimbus Ref: all Ans: B
10274. What will be the effect on the reading of an altimeter of an aircraft parked
on the ground shortly before an active cold front passes? A – It will be decreasing
B – It will remain unchanged C – It will be increasing D – It will fluctuate up and
down by about +/- 50 feet Ref: all Ans: C 10277. (Refer to figure 050-10) Which
cross-section of air mass and cloud presentation is applicable to the straight line
A-B? A–1 B–3 C–2 D–4 Ref: all Ans: B 10278. Thunderstorms in exceptional
circumstances can occur in a warm front if: A – the cold air is convectively stable
B – the cold air is convectively unstable C – the warm air is convectively unstable
D – the warm air is convectively stable Ref: all Ans: C
10279. Read this description: “After such a fine day, the ring around the moon was
a bad sign yesterday evening for the weather today. And, sure enough, it is pouring
down outside. The clouds are making an oppressively low ceiling of uniform grey;
but at least it has become a little bit warmer.” Which of these weather phenomena
is being described? A – A warm front B – A blizzard C – Weather at the back of a
cold front D – A cold front Ref: all Ans: A 10281. A frontal depression passes
through the airport. What form of precipitation do you expect? A – Rain or snow
during about 12 hours until the warm front arrives. Within the warm sector the rain
increases. Improvement on the passage of the cold front B – Continuous rain or snow
while the frontal wave passes for a period of some 24 hours C – Showers during some
2 hours until the warm front arrives. Drizzle in the warm sector within 12 hours.
Rain or snow on the passage of the cold front D – Continuous rain or snow during 6
hours until the warm front arrives. The precipitation stops for several hours
within the warm sector. On the arrival of the cold front, showers within a couple
of hours Ref: all Ans: D 10282. After passing at right angles through a very active
cold front in the direction of the cold air, what will you encounter, in the
northern hemisphere immediately after a marked change in temperature? A – A
decrease in head wind B – A backing in the wind direction C – An increase in tail
wind D- A veering in the wind direction Ref: all Ans: D
10284. The main factor which contributes to the formation of very low clouds ahead
of a warm front is the: A – saturation of the warm air by rain falling into it and
evaporating B – saturation of the cold air by rain falling into it and evaporating
C – reduction of outgoing radiation due to clouds D – warm air moving over a cold
surface Ref: all Ans: B 10286. What type of precipitation would you expect at an
active unstable cold front? A – Showers associated with thunderstorms B – Freezing
rain C – Light to moderate continuous rain D – Drizzle Ref: all Ans: A 15867. A
squall line usually is most likely to be encountered: A – in an air mass with cold
mass properties B – behind of a stationary front C – ahead of a cold front D – at
an occluded front Ref: all Ans: C 16354. Ahead of a warm front: A – winds back and
increase with height B – wind back slightly but veer on passage and increase with
height C – winds veer and decrease with height D – winds back and decrease with
height Ref: all Ans: B
16361. Jets are very rare near occluded front because: A – the air is too cold
across the fronts B – the air is too warm across the fronts C – there is not enough
pressure difference across the fronts D – there is not enough temperature
difference across the fronts Ref: all Ans: D 16366. The boundary between polar and
tropical air is known as: A – Tropical front B – Cold front C – Warm front D –
Polar front Ref: all Ans: D 16427. If cold air is being “replaced” by warm air, the
boundary between the air masses is called: A – a warm front B – a cold front C – a
polar front D – an arctic front Ref: all Ans: A 16430. Which one of the following
alternatives indicates how an occluded front is generated? A – warm air supersedes
cold air B – cold air wedges under warm air C – a cold front overtakes a warm front
and the warm air between the fronts is lifted D – a cold front is halted and
becomes almost stationary Ref: all Ans: C
16432. When flying through a cold front in the summer, the following flying weather
may be expected: A – towering clouds with showery precipitation B – horizontally
extended clouds with drizzle C – horizontally extended clouds with even tops and
bases D – towering clouds without turbulence Ref: all Ans: A 16434. The passage of
a cold front over a meteorological station will result in: A – a steady fall in
pressure and a backing of the surface wind B – a steady rise in pressure and a
veering of the surface wind C – a steady fall in pressure and a veering of the
surface wind D – a steady rise in pressure and a backing of the surface wind Ref:
all Ans: B 16435. The mean position of the polar front in the North Atlantic is: A
– from Florida to SW UK in July B – from Florida to North of the UK in January C –
from north of UK to Newfoundland in July D – from SW UK to Newfoundland in January
Ref: all Ans: C 24171. (Refer to figure 050-113) The diagram of the system in annex
represents a A – cold occlusion B – warm occlusion C – warm front D – cold front
Ref: all Ans: B
24210. A gust front is: A – normally encountered directly below a thunderstorm B –
formed by the cold air outflow from a thunderstorm C – characterised by heavy
lightning D – another name for a cold front Ref: all Ans: B 24218. A stationary
observer in the northern hemisphere is situated in front of a depression. The
centre of the depression passes from west to east and south of the observer. For
this observer the wind: A – backs B – veers C – initially veers, then backs D –
initially backs, then veers Ref: all Ans: A 24241. An occlusion has the
characteristics of a warm front when: A – the cold air behind is colder than the
cold air ahead B – the cold air behind is warmer than the cold air ahead C – the
cold air behind is lifted by the warm air D – the cold air ahead is lifted Ref: all
Ans: B 24242. An occlusion is called a warm occlusion when the cold air: A – at the
rear of the occlusion is colder than the cold air ahead, with the warm air at a
higher altitude B – ahead of the surface position of the occlusion is only at a
higher altitude C – at the rear of the occlusion is colder than the cold air ahead
D – at the rear of the occlusion is less cold that the cold air ahead, with the
warm air at a higher altitude Ref: all Ans: D
24247. At a cold front: A – warm air is lifted as cooler air pushes under it B –
warm air is compressed as cold air rises over it C – temperature rises owing to
increased pressure D – fog will form from the interaction of cold and warm air Ref:
all Ans: A 24248. At a station at the surface the significant weather with a warm
front will come: A – after the warm sector has passed B – only at the same time as
the front passes C – after the front has passed D – mostly before the front passes
Ref: all Ans: D 24266. During the passage of a front in the northern hemisphere the
wind veers. This statement is: A – not true B – only true for the passage of a cold
front C – only true for the passage of a warm front D – true Ref: all Ans: D 24295.
In the northern hemisphere advection of warm air aloft indicates: A – the approach
of a warm occlusion B – backing winds with increasing heights C – increasing
probability for showers D – the formation of advection fog Ref: all Ans: A
24324. The air mass in the warm sector of a polar front is: A – arctic air B –
polar air C – tropical air D – equatorial air Ref: all Ans: C 24326. The arctic
front is the boundary between: A – arctic air and tropical air B – polar air and
tropical air C – cold polar air and less cold polar air D – polar air and arctic
air Ref: all Ans: D 24330. The first clouds are thin, wispy cirrus, followed by
sheets of cirrus and cirrostratus, and altostratus. The sun is obscured as the
altostratus thickens and drizzle or rain begins to fall. The cloud base is lowering
as Nimbostratus arrives. These phenomena: A – warm front B – cold front C – trade
wind front D – sea breeze front Ref: all Ans: A 24332. The following sequence of
clouds is observed at an airport; cirrus, cirrostratus, altostratus, nimbostratus.
This is typical for: A – the passage of a squall line B – the passage of a cold
front C – anti-cyclonic weather D – the passage of a warm front Ref: all Ans: D
24338. The lowest cloud type observed is Stratus fractus and there is moderate
continuous rain. The area of the system in which you are at this moment is: A –
behind the cold front B – the main body of the warm or cold front, or of the
occlusion C – the warm sector D – the high pressure area Ref: all Ans: B 24342. The
passage of a warm front can be associated with areas of fog. The types of fog just
in advance and just after the passage are respectively A – arctic smoke and frontal
fog B – advection fog and radiation fog C – frontal fog and advection fog D –
advection fog and steaming fog Ref: all Ans: C 24347. The reason for the fact, that
the atmospheric pressure of a polar front depression is normally lower in winter
than in summer is that A – converging air currents are of greater intensity in
winter B – the low pressure activity of the sea east of Canada is higher in winter
C – the strong winds of the north Atlantic in winter are favourable for the
development of lows D – the temperature contrasts between arctic and equatorial
areas are much greater in winter Ref: all Ans: D 24348. The slope and speed of a
warm front compared to the slope and speed of a cold front is in general: A –
smaller and slower B – greater and faster C – greater and slower D – smaller and
faster Ref: all Ans: A
24396. When a front has to cross a chain of mountains, its activity: A –
strengthens “upwind” of the mountains B – decreases when it reaches the mountains C
– is not disturbed by the mountains D – ceases immediately Ref: all Ans: A 24406.
Where is the projection of the polar front jet stream on the surface most likely to
be found in relation to the cold and warm fronts of a depression? A – 30 to 450 NM
behind the cold front and 50 to 200 NM ahead of the warm front B – Up to 100 NM
either side of the cold front and up to 200 NM either side of the warm front C – Up
to 200 NM either side of the cold front and up to 200 NM either side of the warm
front D – 50 to 200 NM behind the cold front and 300 to 450 NM ahead of the warm
front Ref: all Ans: D 24438. Which statement concerning the cold front and warm
front of a frontal depression in the northern hemisphere is correct? A – The
precipitation zone of the cold front is in general wider than the precipitation
zone of the warm front B – The risk of fog is greater ahead of and behind the warm
front than ahead of and behind the cold front C – While occluding the warm front
always becomes a front aloft D – The wind backs more at the warm front than at the
cold front Ref: all Ans: B
24443. Which statement is correct for a warm occlusion? A – The warm front
overtakes the cold front B – The cold front becomes a front aloft C – The warm
front becomes a front aloft D – Both fronts become fronts aloft Ref: all Ans: B
050-07

PRESSURE SYSTEMS

050-07-01 Location of the principal pressure areas


9353. In which of the following areas do surface high pressure systems usually
predominate over the North Atlantic region between 30oN and 65oN and the adjoining
land areas during the northern summer? A – Greenland, SW Europe, NE Canada B –
Greenland, Azores, NE Canada C – Iceland, SW USA, Azores D – Azores, SE USA, SW
Europe Ref: all Ans: D 9367. Select the answer which you consider will complete
correctly the following statement in relation to the main pressure systems
affecting the North Atlantic region between 30oN and 65oN. During winter the
predominant mean low pressure system at the surface is usually centred over: A –
USA B – Iceland/Greenland C – Siberia D – Azores Ref: all Ans: B 9391. Considering
the North Atlantic region between 30oN and 65oN together with the adjacent land
areas during winter, the normal disposition of the main anticyclones at the surface
is: A – Azores, Siberia B – Siberia, Iceland, Canaries C – NE Canada, Iceland D –
Greenland, Iberian peninsula Ref: all Ans: A
9405. Considering the North Atlantic region between 30oN and 65oN and the adjacent
land areas during mid summer, the predominant pressure systems are: A – weak low
over NE Canada and Scandinavian high B – Scandinavian high and Azores high C –
Azores high and weak low over NE Canada D – Azores low and Icelandic high Ref: all
Ans: C

050-07-02 Anti-cyclone
9344. In temperate latitudes what weather conditions may be expected over land
during the summer in the centre of a stationary high pressure zone? A – NS B – TS,
SH C – CB, TS D – Calm winds, haze Ref: all Ans: D 9348. If the pressure surfaces
bulge upwards in all levels then the pressure system is a: A – cold low B – warm
high C – cold high D – warm low Ref: all Ans: B 9358. The most effective way to
dissipate cloud is by: A – convection B – subsidence C – a decrease in temperature
D – a decrease in pressure Ref: all Ans: B
9362. Subsidence is: A – vertically upwards motion of air B – horizontal motion of
air C – vertically downwards motion of air D – the same as convection Ref: all Ans:
C 9390. The stable layer at some height in the low troposphere of an older high
pressure area in the mid-latitudes is called: A – subsidence inversion B – friction
inversion C – radiation inversion D – trade wind inversion Ref: all Ans: A 9399. A
blocking anti-cyclone on the northern hemisphere is: A – a warm anti-cyclone/quasi
stationary/situated between 50oN and 70oN B – quasi stationary/situated between
50oN and 70oN/a cold anti-cyclone C – situated between 50oN and 70oN/a cold anti-
cyclone/steering depressions D – a cold anti-cyclone/steering depressions/situated
over Scandinavia Ref: all Ans: A 9410. What surface weather is associated with a
stationary high pressure region overland in the winter? A – Thunderstorms B – A
tendency for fog and low ST C – NS with continuous rain D – The possibility of snow
showers Ref: all Ans: B
9415. What is the most likely cause of a lack of clouds at higher levels in a
stationary high? A – Instability B – Rising air C – Sinking air D – Divergence at
higher levels Ref: all Ans: C 9416. Areas of sinking air are generally cloudless
because as air sinks it: A – reaches warmer layers B – is heated by compression C –
is heated by expansion D – loses water vapour Ref: all Ans: B 9420. What is the
correct term for the descending air flow in a large high pressure area? A –
Convection B – Convergence C – Advection D – Subsidence Ref: all Ans: D 16433.
Polar air moving south will become: A – increasingly stable B – increasingly
unstable C – conditionally unstable D – it depends on the humidity Ref: all Ans: B
24206. A blocking anti-cyclone in the northern hemisphere is: A – quasi
stationary/situated between 50oN and 70oN/a cold anti-cyclone B – a warm anti-
cyclone/quasi stationary/situated between 50oN and 70oN C – situated between 50oN
and 70oN/a cold anti-cyclone/steering depressions D – a cold anti-cyclone/steering
depressions/situated over Scandinavia Ref: all Ans: B 24263. During summer an anti-
cyclone covers the British isles giving mainly clear skies. At 0600 UTC a south
coast airfield in Southern England reports a surface wind of 350/06. The coastline
at the airfield perimeter is aligned in an east/west direction. During: A – back to
NW and strengthen by mid afternoon B – become southerly to south westerly and
increase in velocity by afternoon C – increase from the north by mid morning
becoming calm towards evening D – veer to easterly before becoming calm by the
afternoon Ref: all Ans: B 24399. When flying at 5000 feet in the northern
hemisphere over plains (flat country) with an anti-cyclone on the left and a
depression on the right, the wind will be: A – from the right B – from the left C –
a head wind D – a tail wind Ref: all Ans: C 24457. You are flying in the northern
hemisphere at 2000 ft over a flat country area. An anti-cyclone is ahead of you and
a depression is behind you. The wind affecting you, will be: A – from your right B
– from your left C – from ahead D – from behind Ref: all Ans: A
050-07-03 Non-frontal depressions
9349. Which is true of a secondary depression in the northern hemisphere? A – It
tends to move round the primary in a cyclonic sense B – It tends to move round the
primary in an anti-cyclonic sense C – It rapidly closes on, and merges with the
primary D – It tends to maintain its position relative to the primary Ref: all Ans:
A 9352. What type of air movement is associated with the centre line of a trough? A
– Divergence with lifting B – Divergence with descending air C – Convergence with
lifting D – Convergence with descending air Ref: all Ans: C 9366. With an intense
trough of low pressure over Iceland during wintertime the weather likely to be
experienced is: A – strong wind shear, convection and snow showers B – light wind,
good visibility and a high cloud ceiling C – strong wind with subsidence at low
levels D – strong wind associated with an almost clear sky Ref: all Ans: A 9392.
How do you recognise a cold air pool? A – A cold air pool may only be recognised on
the surface chart as a low pressure area B – As a high pressure area aloft (eg. On
the 500 hPa chart) C – As a low pressure area aloft (eg. On the 500 hPa chart) D –
A cold air pool may only be recognised on the surface chart as a high pressure area
Ref: all Ans: C
9395. What is encountered during the summer, over land, in the centre of a cold air
pool? A – Nothing (CAVOK) B – Strong westerly winds C – Fine weather CU D – Showers
and thunderstorms Ref: all Ans: D 9407. A trough of low pressure on a surface
synoptic chart is an area of: A – divergence and subsidence B – convergence and
widespread ascent C – divergence and widespread ascent D – convergence and
subsidence Ref: all Ans: B 9408. Extensive cloud and precipitation is often
associated with a non-frontal thermal depression because of: A – surface divergence
and upper level convergence causing widespread descent of air in the depression B –
surface convergence and upper level divergence causing widespread descent of air in
the depression C – surface convergence and upper level divergence causing
widespread ascent of air in the depression D – surface divergence and upper level
convergence causing widespread ascent of air in the depression Ref: all Ans: C
9412. Which is true of a typical non-frontal thermal depression? A – It forms over
the ocean in summer B – It forms over land in summer C – It forms over the ocean in
winter D – It forms over land in winter Ref: all Ans: B
24208. A cold air pool: A – occurs frequently in winter to the south of the Alps
when this region is under the influence of cold north westerly air stream B –
develops usually in winter when very unstable maritime polar or maritime arctic air
currents stream southwards along the eastern side of an extensive ridge of high
pressure, in association with occluded systems C – normally disappears at night and
occurs almost exclusively in summer D – is usually most evident in the circulation
and temperature fields of the middle troposphere and may show little or no sign on
a surface chart Ref: all Ans: D 24256. Cold air pools: A – only occur in winter B –
can easily be recognised on synoptic surface charts C – only occur at mid-latitudes
D – are most evident in the temperature and wind fields of the upper levels Ref:
all Ans: D

050-07-04 Tropical revolving storms


9339. What type of clouds, visible even at a long distance, could indicate the
presence of a tropical revolving storm? A – NS spread over a large area B –
Frequent SC C – Excessive accumulation of CU D – Dense C1 Ref: AIR: atpl; HELI:
atpl, cpl Ans: D 9345. What is the likely track for a hurricane in the Caribbean
area? A – West in the earlier stages and later turning north east B – East then
south C – West deep into the USA D – West in the earlier stages and later turning
south east Ref: AIR: atpl; HELI: atpl, cpl
Ans: A 9354. What is the main energy source of a tropical revolving storm? A – The
equatorial jet stream B – Temperature difference between equatorial low pressure
trough and subtropical high pressure belt C – Cold air advancing from temperate
latitudes D – Latent heat released from condensing water vapour Ref: AIR: atpl;
HELI: atpl, cpl Ans: D 9355. During which months is the Hurricane season in the
Caribbean? A – January until April B – October until January C – July until
November D – April until July Ref: AIR: atpl; HELI: atpl, cpl Ans: C 9363. On which
coast of North America is the danger of tropical revolving storms the greatest? A –
NE coast B – W coast C – N coast D – SE coast Ref: AIR: atpl; HELI: atpl, cpl Ans:
D 9364. When, if at all, is a tropical revolving storm most likely to affect
Darwin, on the central north coast of Australia? A – December to April B – May to
July C – August to October D – Not experienced at Darwin Ref: AIR: atpl; HELI:
atpl, cpl Ans: A
9369. During which seasons are hurricanes most likely to appear in the northern
hemisphere? A – All seasons B – Winter C – Winter and spring D – Summer and autumn
Ref: AIR: atpl; HELI: atpl, cpl Ans: D 9376. Tropical revolving storms do not occur
in the south-east Pacific and the south Atlantic because: A – there is no coriolis
force present B – of the low water temperature C – of the strong southeast wind D –
the southeast trade winds cross over into the northern hemisphere Ref: AIR: atpl;
HELI: atpl, cpl Ans: B 9379. Why to tropical revolving storms tend to develop
mostly in the western parts of the tropical oceans? A – Because there is a maximal
temperature difference between land mass and sea B – Because they are areas in
which there is a strong progressive wind shear with increase of height C – Because
there is a maximum of humidity as a result of the trade winds long sea passage D –
Because the gulf formation of the coastlines triggers a strong rotary circulation
Ref: AIR: atpl; HELI: atpl, cpl Ans: C 9380. What is the track most likely to be
taken by a hurricane in the Caribbean area? A – West in the earlier stages and
later south east B – East C – West deep into the US D – West in the earlier stages
and later north east Ref: AIR: atpl; HELI: atpl, cpl
Ans: D 9388. The region of the globe where the greatest number of tropical
revolving storms occur is: A – the northern Indian ocean, affecting India, Sri
Lanka and Bangladesh B – the Caribbean sea, affecting the West Indies, Mexico and
the south east coastline of the USA C – the south western Indian ocean, affecting
Madagascar, Mauritius and the island of Reunion D – the north west Pacific,
affecting Japan, Taiwan, Korea and the Chinese coastline Ref: AIR: atpl; HELI:
atpl, cpl Ans: D 9400. (Refer to figure 050-18) The arrows labelled “U” represent
the tracks of tropical revolving storms which occur mainly from: A – January to
March and are called willy willies B – July to October and are called typhoons C –
May to July and are called cyclones D – December to April and are called tornadoes
Ref: AIR: atpl; HELI: atpl, cpl Ans: B 9402. Which statement is true for hurricanes
in the North Atlantic? A – From the earth’s surface up to the tropopause the core
is warmer than its surroundings B – They intensify rapidly after landfall C – The
diameter is 50-500m D – Their greatest frequency of occurrence is in winter Ref:
AIR: atpl; HELI: atpl, cpl Ans: A
9403. At what time of the year are typhoons most likely to occur over the southern
islands of Japan? A – January to May B – September to January C – July to November
D – May to July Ref: AIR: atpl; HELI: atpl, cpl Ans: C 9409. (Refer to figure 050-
18) The arrows labelled “S” represent the mean tracks of tropical revolving storms
which occur mainly from: A – December to April and are called cyclones B – May to
November and are called cyclones C – May to November and are called hurricanes D –
December to April and are called typhoons Ref: AIR: atpl; HELI: atpl, cpl Ans: B
9411. (Refer to figue 050-18) The arrows labelled “R” represent the mean tracks of
tropical revolving storms which occur mainly from: A – June to October and are
called typhoons B – December to April and are called tornadoes C – December to
April and are called cyclones D – June to October and are called hurricanes Ref:
AIR: atpl; HELI: atpl, cpl Ans: C 9419. Where is the most dangerous zone in a
tropical revolving storm? A – Anywhere in the eye B – In the wall of clouds around
the eye C – In the centre of the eye D – About 600 km away from the eye Ref: AIR:
atpl; HELI: atpl, cpl Ans: B
050-08

CLIMATOLOGY 050-08-01 Climatic zones

9468. The reason for the fact that the Icelandic low is normally deeper in winter
than in summer is that: A – the strong winds of the north Atlantic in winter are
favourable for the development of lows B – the low pressure activity of the sea
east of Canada is higher in winter C – the temperature contrasts between arctic and
equatorial areas are much greater in winter D – converging air currents are of
greater intensity in winter Ref: AIR: atpl; HELI: atpl, cpl Ans: C 9473. What is
the type, intensity and seasonal variation of precipitation in the equatorial
region? A – Precipitation is generally in the form of showers but continuous rain
occurs also. The greatest intensity is in July B – Warm fronts are common with
continuous rain. The frequency is the same throughout the year C – Rain showers,
hail showers and thunder storms occur the whole year, but frequency is highest
during two periods: April-May and OctoberNovember D – Showers of rain or hail occur
throughout the year; the frequency is highest in January Ref: AIR: atpl; HELI:
atpl, cpl Ans: C 10312. At about what geographical latitude as average is assumed
for the zone of prevailing westerlies? A – 50oN B – 10oN C – 30oN D – 80oN Ref:
AIR: atpl; HELI: atpl, cpl Ans: A
24257. Considering the North Atlantic area north of 60oN during winter, the mean
height of the tropopause is approximately: A – 37000 ft B – 56000 ft C – 29000 ft D
– 20000 ft Ref: AIR: atpl; HELI: atpl, cpl Ans: C 24258. Considering the North
Atlantic at latitude 50oN during winter, the mean height of the tropopause is
approximately A – 23000 ft B – 31000 ft C – 43000 ft D – 54000 ft Ref: AIR: atpl;
HELI: atpl, cpl Ans: B 24260. Considering the North Atlantic region between 30oN
and 65oN, the mean position of the polar front during summer extends from: A –
Florida to SW England B – Newfoundland to N Scotland C – NE Canada to Iceland D –
Greenland to Spain Ref: AIR: atpl; HELI: atpl, cpl Ans: B

050-08-02 Tropical climatology


9435. What winds are mainly associated with the winter monsoon in the monsoon
regions of the Indian sub-continent? A – North easterly winds bringing dry and hazy
air B – South westerly winds carrying warm and humid air C – North westerly winds
bringing dry and hazy air D – South easterly winds carrying warm and humid air Ref:
AIR: atpl; HELI: atpl, cpl
Ans: A 9456. Which one of the following statements regarding the inter-tropical
convergence zone (ITCZ) is correct? A – The ITCZ does not change its position
during the course of the year B – Thunderstorms seldom occur within the area of the
ITCZ C – The ITCZ is always associated with a strong jet stream D – Frequent and
widespread thunderstorms are to be expected within the area of the ITCZ Ref: AIR:
atpl; HELI: atpl, cpl Ans: D 9457. The inter-tropical convergence zone (ITCZ)
particularly affects: A – western Africa, at a latitude of 25oN in July B – western
Africa between 10o and 20oN and the northern coasts of the Arabian sea in July C –
the Atlantic ocean, between latitudes of 10oN and 30oN depending on the time of
year D – western Africa, where it is situated between the 10oN and 30oN parallels,
depending on the time of the year Ref: AIR: atpl; HELI: atpl, cpl Ans: B 9463.
Which wind systems converge on the ITCZ, when it lies at the equator? A – SE trade
winds and NE trade winds B – SW monsoon and NW monsoon C – SW monsoon and NW trade
winds D – NW monsoon and SW trade winds Ref: AIR: atpl; HELI: atpl, cpl Ans: A
9470. Which of the following statements concerning trade winds is correct? A – They
reach up to the tropopause and are more pronounced over the continents B – They
occur only in the lower part of the troposphere and more pronounced over the oceans
C – They reach up to the tropopause and are more pronounced over the oceans D –
They occur only in the lower part of the troposphere and are more pronounced over
the continents Ref: AIR: atpl; HELI: atpl, cpl Ans: B 9475. Weather conditions at
Bombay during January are mainly influenced by the: A – SW monsoon B – NE monsoon C
– NW monsoon D – SE monsoon Ref: AIR: atpl; HELI: atpl, cpl Ans: B 9479. In which
month does the humid monsoon in India start? A – In October B – In June C – In
December D – In March Ref: AIR: atpl; HELI: atpl, cpl Ans: B
9481. An easterly wave is a: A – wave in a trade wind belt, moving from east to
west, with severe convective activity in rear of its trough B – wave-like
disturbance in the monsoon regime of India, moving from east to west, with severe
convective activity ahead of its trough C – small scale wave disturbance in the
tropics, moving from east to west, with severe convective activity ahead of its
trough D – disturbance in the higher levels associated with the equatorial easterly
jet, moving from east to west, with severe convective activity in rear of its
trough Ref: AIR: atpl; HELI: atpl, cpl Ans: A 9484. (Refer to figure 050-21) What
weather conditions are most likely to affect an approach to Dakar during July? A –
Reduced visibility due to the rising sand of the Harmattan B – Dry and clear due to
the influence of the Azores high pressure system C – Generally clear skies – NW
trade winds D – Wet and thundery due to the proximity of inter-tropical convergence
zone (ITCZ) Ref: AIR: atpl; HELI: atpl, cpl Ans: D 9486. Which one of the following
statements is correct concerning the movement of the ITCZ in the region of West
Africa? A – It reaches its maximum southerly position of 5oS in January B – It
reaches its maximum northerly position of 15o – 20oN in July C – It oscillates
during the year between 10oN and 10oS D – It oscillates during the year between the
Equator and 10oN Ref: AIR: atpl; HELI: atpl, cpl Ans: B
9490. What name is given to the low level wind system between the sub-tropical high
pressure belt and the equatorial trough of low pressure (ITCZ)? A – Doldrums B –
Westerly winds C – Monsoon D – Trade winds Ref: AIR: atpl; HELI: atpl, cpl Ans: D
9519. Weather conditions at Bombay during early July are mainly influenced by the A
– NE monsoon and the proximity of the ITCZ B – SW monsoon C – passage of frontal
system generated in the south Indian ocean D – high incidence of tropical revolving
storms originating in the Persian Gulf Ref: AIR: atpl; HELI: atpl, cpl Ans: B 9523.
What weather is prevalent in the zone of easterly waves? A – Thunderstorms and rain
B – Continuous rain C – Clear skies D – Frontal weather Ref: AIR: atpl; HELI: atpl,
cpl Ans: A 10297. During July flights from Bangkok (13oN – 100oE) to Karachi (25oN
– 67oE) experience an average tailwind component of 22 kt. In January the same
flights, also operating at FL 370, have an average head wind of 50 kt. What is the
reason for this difference? A – The flights happen to be in the area of the polar
front jet stream B – The flights during the summer encountered, by chance, very
unusual, favourable conditions C – The flights in January encountered, by chance,
very unusual, adverse conditions D – The wind components correspond to the seasonal
change of the regional wind system Ref: AIR: atpl; HELI: atpl, cpl Ans: D
10299. Along the West coast of India the prevailing winds are the: A – NE monsoon
in July and a SW monsoon in January B – SW monsoon in July and a SE monsoon in
January C – SW monsoon in July and a NE monsoon in January D – SE monsoon in July
and a SW monsoon in January Ref: AIR: atpl; HELI: atpl, cpl Ans: C 10300. Where,
during a flight from Marseille to Dakar, in July, may the ITCZ be encountered? A –
At the latitudes of Algeria B – Near the Canary Islands C – At the latitudes of
Gibraltar D – In the vicinity of Dakar Ref: AIR: atpl; HELI: atpl, cpl Ans: D
10302. From which direction do the trade winds blow, in the southern hemisphere? A–
N B – NE C – SW D – SE Ref: AIR: atpl; HELI: atpl, cpl Ans: D 10307. When are the
rainy seasons in equatorial Africa? A – March to May and October to November B –
December to February and July to October C – March to May and August to October D –
April to July and December to February Ref: AIR: atpl; HELI: atpl, cpl Ans: A
10316. The transition from SW to NE monsoon in India occurs in: A – September,
October, November B – July, August, September C – December, January, February D –
February, March, April Ref: AIR: atpl; HELI: atpl, cpl Ans: A 10328. What weather
conditions are indications of the summer monsoon in India? A – Fog B – Stratus
clouds and drizzle C – Sandstorms D – Thunderstorms, showers of heavy rain Ref:
AIR: atpl; HELI: atpl, cpl Ans: D 10333. In which of the following bands of
latitude is the inter-tropical convergence zone most likely to be encountered in
January, between Dakar and Rio de Janeiro? A – 8o – 12oS B – 3o – 8oS C – 0o – 7oN
D – 7o – 12oN Ref: AIR: atpl; HELI: atpl, cpl Ans: C 10334. Which of the following
best describes the inter-tropical convergence zone? A – The zone where the
Harmattan meets the north easterly trade winds over Africa B – The zone where the
trade winds of the northern hemisphere meet those of the southern hemisphere C –
The zone where cold fronts form in the tropics D – The zone where the west winds
meet the sub-tropical high pressure belt Ref: AIR: atpl; HELI: atpl, cpl Ans: B
10337. What is the name of the wind or air mass which gives to the main part of
India its greatest proportion of precipitation? A – Indian, maritime tropical air
mass B – South east trade wind C – South west monsoon D – Winter monsoon Ref: AIR:
atpl; HELI: atpl, cpl Ans: C 10563. (Refer to figure 050-48) Assuming a generalised
zonal system of world wind circulation, the NE trade winds are applicable to zone:
A–U B–T C–V D–W Ref: AIR: atpl; HELI: atpl, cpl Ans: A 16333. In January, mean high
pressure areas are usually present over: A – the Aleutians, Australia, South
America B – the Azores, Siberia, the South Pacific C – the South Pacific, the
Azores, Australia D – Greenland, Siberia, North West Australia Ref: AIR: atpl;
HELI: atpl, cpl Ans: B 16334. Which of the following statements concerning the ITCZ
is correct? A – In the southern summer it is normally positioned entirely south of
the equator B – Its furthest displacement from the equator is normally about 45oS C
– It is normally fed with converging northern and southern trade winds D – Its
associated weather is invariably strong convergence and heavy cumuliform cloud Ref:
AIR: atpl; HELI: atpl, cpl Ans: D
16336. The Doldrums are: A – another name for the sub-tropical anti-cyclones B –
cold between weak fronts encountered in low latitudes C – weak inter-tropical
convergence zones D – associated with light and variable monsoon winds Ref: AIR:
atpl; HELI: atpl, cpl Ans: C 24106. (Refer to figure 050-61) Considering the sector
from 10oN to Nairobi of the route indicated, during January the upper winds at the
300 hPa level are most likely to be: A – westerlies in excess of 60 kt B – easterly
jet streams in excess of 70 kt C – light easterlies D – westerlies at 40 kt Ref:
AIR: atpl; HELI: atpl, cpl Ans: C 24133. (Refer to figure 050-18) The arrows
labelled “t” represents the mean tracks of tropical revolving storms which occur
mainly from: A – June to October and are called cyclones B – December to April and
are called hurricanes C – June to October and are called tornadoes D – June to
October and are called hurricanes Ref: AIR: atpl; HELI: atpl, cpl Ans: A 24134.
(Refer to figure 050-19) Considering that portion of the route indicated from 30oE
to 50oE, the upper winds in January above FL 300 are most likely to be: A – a sub-
tropical westerly jet stream, maximum speed exceeding 90 kt B – a westerly polar
front jet stream, maximum speed exceeding 90 kt C – variable in direction and less
than 30 kt D – light easterlies Ref: AIR: atpl; HELI: atpl, cpl Ans: A
24135. (Refer to figue 050-61) Which is true of Nairobi (Kenya)? A – There are two
wet seasons B – Apart from the wet season(s) it is cloud and rain free C – There is
one wet season D – In July it is frequently affected by clear skies and northerly
winds from North Africa Ref: AIR: atpl; HELI: atpl, cpl Ans: A 24150. (Refer to
figure 050-20) The typical weather conditions affecting the Darwin area during July
are: A – NE monsoon – continuous heavy rain but little thunder storm activity B –
dry season – mainly SE winds – visibility reduced by dust and haze C – NW monsoon –
very wet – proximity of the inter-tropical convergence zone causes widespread
thunderstorm activity D – mainly clear skies – advection fog drifting inland with
the typical NE wind Ref: AIR: atpl; HELI: atpl, cpl Ans: B 24265. During the
approach to Bombay (19oN – 73oE) on the west coast of India, you are listening to
the ATIS on 15 July at 0700 T. Which of the following reports is most likely? A –
25014KT 4500 SHRA SCT015 BKN025CB 25/24 Q1006 NOSIG = B – 05013KT 3500 MIFG SCT003
BKN005 19/14 Q1012 BECMG 8000 = C – 02005KT CAVOK 24/09 Q1030 NOSIG = D – 30012KT
9999 SCT030 SCT200 20/16 Q1025 BECMG 4000 BR = Ref: AIR: atpl; HELI: atpl, cpl Ans:
A
24310. On the West coast of India it can be said in general that the wind blows: A
– the whole year from the North East B – for six months from the North West and for
six months from the South East C – for six months from the North East and for six
months from the South West D – the whole year from the South East Ref: AIR: atpl;
HELI: atpl, cpl Ans: C 24312. Over which areas can tropical cyclones occur? A –
Australia, Gulf of Bengal, Atlantic Ocean at 20oS B – India, Arabic Sea, Atlantic
Ocean at 2oS C – Caribbean Sea, Gulf of Bengal, Indian Ocean East of Madagascar D –
Caribbean Sea, Indian Ocean at 20oS, Pacific Ocean at 2oN Ref: AIR: atpl; HELI:
atpl, cpl Ans: C 24404. Where do the trade winds blow? A – At the equator B –
Between the “horse latitudes” and the mid latitudes C – Between the “horse
latitudes” and the equatorial highs D – Between the “horse latitudes” and the
doldrums Ref: AIR: atpl; HELI: atpl, cpl Ans: D
050-08-03 Typical weather situations in midlatitudes
9446. A cold pool: A – normally disappears at night and occurs almost exclusively
in summer B – develops usually in winter when very unstable maritime polar or
maritime arctic air currents stream southwards along the eastern side of an
extensive ridge of high pressure, in association with occluded systems C – is
usually most evident in the circulation and temperature fields of the middle
troposphere and may show little or no sign on a surface chart D – occurs frequently
in winter to the south of the Alps when this region is under the influence of cold
north westerly air stream Ref: all Ans: C 9458. Which of the following statements
concerning the inter-tropical convergence zone is true? A – There are frequent
occurrences of CB B – It lies totally in the northern hemisphere in July and
totally in the southern hemisphere in January C – It does not change its position
over the oceans during the year D – It is an area of low pressure and low relative
humidity Ref: all Ans: A 9461. (Refer to figure 050-07) Which typical weather
situation is shown on the weather chart? (spacing of the isobars: 5 hPa) A –
Uniform pressure pattern B – Cutting wind C – West wind condition D – Warm south
and condition (Foehn) Ref: all Ans: A
10289. (Refer to figure 050-43) Which typical weather condition is shown by the
design for northern Italy? A – Westerly wind B – Warm southerly wind C – High
pressure D – Easterly wind Ref: all Ans: C 10304. With a uniform pressure pattern
and no thunderstorms around, what will the indication of the aneroid altimeter of
an aircraft parked on the ground do over a period of about ten minutes? A –
Apparently nothing, because any changes would be small B – Increase rapidly C –
Show strong fluctuations D – Decrease rapidly Ref: all Ans: A 10319. (Refer to
figure 050-42) Which typical weather condition is shown by the design for the area
of Central Europe? A – Westerly waves B – Uniform pressure pattern C – Cutting wind
D – Easterly waves Ref: all Ans: A 16339. Which of the following statements is
correct? A – In winter the Savannah climatic region is governed by the equatorial
rains B – Trade wind seldom extend much above 5000 feet C – The outflow of air from
the Siberian high over northern China and Japan is initially north westerly D – The
easterly jet stream normally appears at the 200 mb level Ref: all Ans: C
24102. (Refer to figure 050-99) Assuming a generalised zonal system of world
climatic and wind circulation, zone “x” is an area of: A – NE trade winds B –
travelling low pressure systems C – sub-tropical high pressure systems D – the
inter-tropical convergence zone (ITCZ) Ref: all Ans: C 24287. In mid-latitudes, the
tops of Cumulus are often limited by: A – a radiation inversion B – a layer of
unstable air C – a temperature inversion D – the tropopause Ref: all Ans: C 24337.
The length, width and height of a typical mid-latitude jet stream are respectively:
A – 1000 nautical miles, 150 nautical miles, 30000 feet B – 1000 nautical miles,
150 nautical miles, 18000 feet C – 1000 nautical miles, 5000 to 8000 feet, 30000
feet D – 200 nautical miles, 5 nautical miles, 18000 feet Ref: all Ans: B 24437.
Which statement about hurricanes in the North Atlantic is correct? A – Their eye
can be well observed by weather satellite B – They intensify after landfall C –
They move towards the equator D – They move with a speed of at least 60 kt Ref: all
Ans: A
050-08-04 Local seasonal weather and wind
9429. The Chinook is a: A – warm and dry wind that forms as air descends on the
leeward side of the Rocky Mountains B – very cold wind with blowing snow C – down
slope wind that occurs particularly at night as air cools along mountain slopes D –
warm anabatic wind up the slopes of snowfields or glaciers Ref: all Ans: A 9430.
What is the name of the northerly, cold and strong wind, that sometimes blows over
a certain part of Europe? A – Bora B – Foehn C – Mistral D – Typhoon Ref: all Ans:
C 9441. Which weather phenomena are typical for the northern side of the Alps with
stormy winds from the south (Foehn)? A – Good visibility, turbulence B – Continuous
precipitation, severe turbulence C – Decrease in temperature, moderate to severe
icing D – Icing, huge mass of clouds Ref: all Ans: A 9449. A dry sand and dust
laden North Easterly wind that blows in winter over large parts of North West
Africa is known as a: A – Scirocco B – Harmattan C – Pampero D – Khamsin Ref: all
Ans: B 9451. For an aircraft what are the meteorological dangers associated with a
Harmattan wind? A – Sand up to FL 150 B – Thunderstorms C – Dust and poor
visibility D – Hail Ref: all Ans: C 9466. What are the characteristics of the Bora?
A – It is a warm and moist, south westerly wind experienced in the eastern
Mediterranean, that usually carries precipitation B – It is a very cold wind that
blows mainly in winter fro a north westerly direction in the Mediterranean C – It
is a cold and very strong wind that blows mainly in winter from a table and
downwards to the Adriatic D – It is a dry and hot southerly wind experienced in the
Sahara desert, that often carries dust Ref: all Ans: C 9474. (Refer to figure 050-
41)? Under the weather conditions depicted, which of the following statements is
likely to apply? A – Severe gradient wind likely over Central Europe B –
Thunderstorms may occur in the summer months over Central Europe C – Moderate to
strong Foehn in the Alps D – Radiation fog is unlikely in Central Europe in the
winter Ref: all Ans: B
9488. Which of the following zones is most likely to encounter little or no
precipitation? A – The north side of the Alps with a prevailing Foehn from the
south B – The north side of the Alps with a prevailing Foehn from the north C –
Frontal zones D – Occlusions Ref: all Ans: A 9495. What weather conditions in the
region of the Alps would you expect with Foehn from south? A – Heavy airframe icing
conditions on the northern side of the Alps B – Heavy clear air turbulence on the
southern side of the Alps C – Strong north winds on the southern side of the Alps D
– Clouds, on the southern sides of passes in the Alps Ref: all Ans: D 9507. The
Foehn wind is a: A – warm anabatic wind B – cold fall wind C – warm fall wind D –
cold anabatic wind Ref: all Ans: C 9524. What is characteristic of the pamperos? A
– Foehn conditions in the Spanish Pyrenees B – Katabatic winds in the Atlas
Mountains C – A marked advance of cold arctic air in Northern America D – A marked
advance of cold air in South America Ref: all Ans: D
9527. What is the strong relatively cold katabatic wind, blowing down the northern
Adriatic coast, mainly during the winter and spring called? A – Mistral B – Ghibli
C – Bora D – Scirocco Ref: all Ans: C 9747. What is the reason for seasonal changes
in climate? A – Because the Earth’s spin axis is inclined to the plane of its orbit
round the Sun B – Because the distance between the Earth and the Sun varies over a
year C – Because the Earth’s orbital speed round the Sun varies according to the
time of the year D – Because of the difference between the Tropical Year and the
Calendar Year Ref: all Ans: A 9808. At what times of the year does the length of
the hours of daylight change most rapidly? A – Spring Equinox and Autumn Equinox B
– Summer Solstice and Winter Solstice C – Spring Equinox and Summer Solstice D –
Autumn Equinox and Winter Solstice Ref: all Ans: A 10301. A strong, dry and warm
katabatic wind, produced by prior enforced ascent of air over hills or mountains is
known as a: A – Bora B – Harmattan C – Mistral D – Foehn Ref: all Ans: D
10326. (Refer to figure 050-56) Considering the route indicates from Lisbon to
Freetown, the Harmattan is a: A – warm southerly dust bearing wind affecting the
coast of North Africa B – SW monsoonal wind causing extensive areas of advection
fog along the West African coast south of 15oN C – NE wind affecting north west
Africa during November to April reducing visibility in rising dust D – localised
depression giving squally winds Ref: all Ans: C 10329. A Foehn wind occurs on the:
A – leeward side of a mountain range and is caused by the condensation level being
lower on the leeward side than on the windward side B – windward side of a mountain
range and is caused by surface heating C – windward side of a mountain range and is
caused by surface cooling and reverse air flow D – leeward side of a mountain range
and is caused by significant moisture loss by precipitation from cloud Ref: all
Ans: D 10330. What is the name of the hot, local wind, that blows downwards from
mountain chains? In the Alps, for instance, it may exist both as a southerly or
northerly wind depending on the weather situation. A – Foehn B – Mistral C – Bora D
– Scirocco Ref: all Ans: A
10331. The Bora is a: A – cold katabatic wind with gusts associated with a maritime
air mass B – squally warm katabatic wind which occurs mainly in summer C – cold
katabatic wind with the possibility of violent gusts D – cold katabatic wind always
associated with clouds and heavy showers Ref: all Ans: C 10336. Which one of the
following local winds is a Foehn wind? A – Harmatan B – Scirocco C – Chinook D –
Bora Ref: all Ans: C 15829. 8/8 stratus base 200 ft/AGL is observed at sunrise at
an aerodrome in the north of France; the QNH is 1028 hPa and there is a variable
wind of 3 kt. What change in these clouds is likely at 12:00 UTC in summer and
winter? A – Winter: SCT base 3,000 ft/AGL; summer OVC base 500 ft/AGL B – Winter:
OVC base 500 ft/AGL; summer SCT base 3000 ft/AGL C – Winter: clear sky; summer BKN
CB base 1500 ft/AGL D – Winter: BKN base 2500 ft/AGL; summer BKN base 3500 ft/AGL
Ref: all Ans: B 24101. (Refer to figure 050-60) Considering the route indicated
from Recife to Dakar the mean height of the tropopause during January is
approximately: A – 56000 ft B – 43000 ft C – 36000 ft D – 29000 ft Ref: all Ans: A
24103. (Refer to figure 050-99) Assuming a generalised zonal system of world wind
circulation, the SE trade winds are applicable to zone: A–t B–w C–u D–v Ref: all
Ans: B 24104 (Refer to figure 050-99) Assuming a generalised zonal system of world
wind circulation the travelling low pressure systems are applicable to zone: A – s
and y B – t only C – t and x D – u and w Ref: all Ans: A 24250. At which height and
at what time of the year can an aircraft be affected by the equatorial jet stream?
A – FL 500 from June to August B – FL 500 from November to February C – FL 400
during the winter in the northern hemisphere D – FL 400 during the winter in the
southern hemisphere Ref: all Ans: A 24259. Considering the North Atlantic between
30oN and 65oN, the mean position of the polar front during winter extends from: A –
Florida to SW England B – Newfoundland to Iceland C – Iceland to Norway D – NE
Canada to Portugal Ref: all Ans: A
24268. For an airfield located in the British Isles, the passage of a warm front
will usually be indicated by: A – a fall in temperature, rise in dew point
temperature, wind backing and decreasing B – rise in temperature, rise in dew point
temperature, wind veers and decreases C – rapid improvement in visibility, pressure
falling rapidly, wind veering and increasing D – rise in temperature, rapid rise in
pressure, wind backs and becomes gusty Ref: all Ans: B 24271. From summer to winter
the polar front jet stream over the North Atlantic moves: A – towards the south and
the speed increases B – towards the north and the speed increases C – towards the
south and the speed decreases D – towards the north and the speed decreases Ref:
all Ans: A 24290. In summer in the northern hemisphere the maximum wind speeds
associated with sub-tropical jet streams are usually located A – below the
tropopause at about 300 hPa B – above the tropopause at about 100 hPa C – below the
tropopause at about 200 hPa D – above the tropopause at about 250 hPa Ref: all Ans:
C 24311. Over the Indian Ocean and the Bay of Bengal tropical cyclones are: A –
frequently observed, in the average 15 per year over the Indian Ocean, but never
over the Bay of Bengal B – rare, in the average one every two years C – never
observed D – occasionally observed, in the average 12 per year Ref: all Ans: D
24325. The air masses that are observed most frequently over western Europe are: A
– polar air and equatorial air B – arctic air and polar air C – polar air and
tropical air D – arctic air and tropical air Ref: all Ans: C 24327. The average
position of the polar front in the northern hemisphere is: A – more southerly
during the summer than during the winter B – more southerly during the winter than
during the summer C – located near 55oN during the whole year D – located near 65oN
during the whole year Ref: all Ans: B 24329. The easterly jet is a jet stream that
occurs: A – only in the winter of the northern hemisphere at approx 30000 ft B –
only in the summer of the northern hemisphere at approx. 45000 ft C – during the
whole year in the southern hemisphere D – during the whole year in the northern
hemisphere Ref: all Ans: B 24331. The Foehn wind is a: A – cold katabatic wind B –
warm katabatic wind C – warm anabatic wind D – cold anabatic wind Ref: all Ans: B
24344. The prevailing surface wind in the area of the west coast of Africa north of
the equator (Gulf of Guinea) is a: A – SW monsoon in winter and NE monsoon in
summer B – SW monsoon in summer and NE trade wind in winter C – NE trade wind in
summer and SE trade wind in winter D – NE monsoon in winter and SE trade wind in
summer Ref: all Ans: B 24350. The SW monsoon starts in the month of: A – December
in southern India to reach Pakistan in May B – September in Pakistan to reach
southern India in November C – June in southern India to reach Pakistan in July D –
March in southern India to reach Pakistan end of April Ref: all Ans: C 24359.
Tropical revolving storms are NOT formed in: A – South Indian Ocean B – Gulf of
Bengal C – Waters around the Philippines D – South Atlantic Ocean Ref: all Ans: D
24368. What is a favourable synoptic situation for the development of a Scirocco? A
– Extension of the Azores high pressure area over the Alps B – High pressure area
over Italy C – Low pressure area in the western part of the Mediterranean D – High
pressure area in the western part of the Mediterranean Sea Ref: all Ans: C
24393. What type of weather can usually be expected in a polar maritime air mass
over Central Europe in the daytime during summer? A – Sky clear B – Continuous rain
and poor visibility C – Drizzle and low stratus D – Showers and good visibility
Ref: all Ans: D 24397. When and where is an easterly jet stream likely to be
encountered? A – Throughout the year to the south of the Azorian high B – In winter
along the Russian coast facing the Arctic ocean C – In summer from the Middle East
extending over the southern part of the Mediterranean to southern Spain D – In
summer from south east Asia extending over southern India to central Africa Ref:
all Ans: D 24426. Which of the following statements concerning the variation in
wind speed between summer and winter on the North Atlantic between FL 300 and FL
400 is most correct? A – The average westerly component is greater in the winter
than in the summer. The latitude of the axis of greatest seasonal wind speed is
further south in winter than in summer B – The average westerly component remains
the same but the axis of greatest speeds moves south in winter C – The average
westerly component remains the same but the altitude of greatest wind speed reduces
in winter D – The average westerly component is greater in the summer than in the
winter. The latitude of the axis of greatest wind speed is also further north in
summer than in winter Ref: all Ans: A
24439. Which statement concerning the Sirocco is correct? A – It is a north
easterly wind over the western part of North Africa with much dust and sand B – It
blows from southerly directions and can carry dust and sand which may reach Europe
C – The carried dust and sand does not reach great heights. This is caused by the
presence of a trade wind inversion D – It is formed by a strong increase of air
pressure over North Africa Ref: all Ans: B 24446. Which statement is correct for
the southern hemisphere? A – The jet streams are easterly B – In the friction layer
the wind backs with increasing height C – The wind veers at the passage of a cold
front D – If the wind veers with increasing height then warm air is advected Ref:
all Ans: B 24449. Which two air masses are most likely to govern weather in western
Europe? A – Continental tropical warm and continental polar cold B – Maritime
tropical warm and maritime polar cold C – Maritime tropical warm and continental
polar cold D – Maritime polar warm and continental tropical warm Ref: all Ans: B
24450. Which type of air mass never occurs over central Europe? A – Tropical air B
– Arctic air C – Polar air D – Equatorial air Ref: all Ans: D
25621. (Refer to figure 050-20) Considering Melbourne (C) in July, the weather is
predominantly influenced by the zone of: A – sub-tropical high pressure, with the
occasional passage of fronts originating in the adjacent zone of westerly waves B –
Antarctic high pressure due to the absence of any protective land mass between
south Australia and Antarctica C – disturbed temperate low pressure, bringing an
almost continuous succession of fronts resulting in strong winds, low cloud and
rain D – equatorial low pressure due to the proximity of the inter-tropical
convergence zone over central Australia Ref: AIR: atpl; HELI: atpl, cpl Ans A
25631. Between which latitudes are you most likely to find the region of travelling
low pressure systems during summer? A – 45o – 75o B – 35o – 45o C – 15o – 25o D –
10o – 15o Ref: all Ans: A 25635. What is the surface visibility most likely to be
in a warm sector of maritime tropical air during a summer afternoon in western
Europe? A – Very poor (less than 1 km) B – Moderate (several km) C – Very good
(greater than 50 km) D – Good (greater than 10 km) Ref: all Ans: B
050-09

FLIGHT HAZARDS 050-09-01 Icing

9536. Large super cooled water drops, which freeze on impact on an airplane form: A
– rime ice B – clear ice C – hoar frost D – cloudy ice Ref: AIR: atpl, ir; HELI:
all Ans: B 9537. In which of these cloud types can icing be virtually ruled out? A
– CU B – SC C – NS D – C1 Ref: all Ans: D 9541. You have been flying for some time
in dense layered cloud. The outside air temperature is -25oC. Which of the
following statements is true? A – If you do not have weather radar on board there
is no need to worry, as CB is unlikely to form in such cloud B – Severe airframe
icing is quite likely under these conditions C – Severe airframe icing is unlikely
under these conditions D – In a dense layered cloud icing is unlikely also at an
outside air temperature of -5oC Ref: all Ans: C
9548. In which of these temperature bands is ice most likely to form on the
aircraft’s surface? A - -20oC to -35oC B - +10oC to 0oC C – 0oC to -10oC D - -35oC
to -50oC Ref: all Ans: C 9568. In which of these cloud types can icing be virtually
ruled out? A – NS B – AS C – SC D – CS Ref: all Ans: D 9570. On the approach, the
surface temperature is given as -5oC. The freezing level Is at 3000 ft/AGL. At 4000
ft/AGL, there is a solid cloud layer from which rain is falling. According to the
weather briefing, the clouds are due to an inversion caused by warm air sliding up
and over an inclined front. Would you expect icing? A – Yes, between ground level
and 3000 ft/AGL B – Yes, but only between 3000 and 4000 ft/AGL C – No. flights
clear of cloud experience no icing D – No. absolutely no icing will occur Ref: all
Ans: A 9581. Which of the following conditions is mot likely to cause airframe
icing? A – PE B – GR C – SHSN D - +FZRA Ref: all Ans: D
9595. A small super cooled cloud droplet that collides with an airfoil will most
likely: A – freeze immediately and create rime ice B – freeze immediately and
create clear ice C – travel back over the wing, creating rime ice D – travel back
over the wing, creating clear ice Ref: all Ans: A 9604. While descending through a
cloud cover at high level, a small amount of a white and rough powder like
contamination is detected along the leading edge of the wing. This contamination is
called: A – Clear ice B – Rime ice C – Mixed ice D – Frost Ref: AIR: atpl, ir;
HELI: all Ans: B 9608. In which of the following situations is an aircraft most
susceptible to icing? A – Level flight below a rain producing cloud when OAT is
below zero degrees C B – Flying in dense cirrus clouds C – Level flight in snowfall
below a nimbostratus layer D – Flying in heavy drizzle Ref: all Ans: A 9612. Which
one of the following statements concerning the formation of aircraft icing is most
correct? A – Risk for icing increases when cloud temperature decreases well below
minus 12 degrees C B – Greatest risk of icing conditions is experienced in cirrus
clouds C – A cloud consisting of both super cooled cloud droplets and ice crystals
produces aircraft icing D – Probability of icing increases when dry snow starts to
fall from a cloud Ref: all Ans: C
9615. Hoar frost is most likely to form when: A – flying inside convective clouds B
– taking off from an airfield with a significant ground inversion C – flying inside
stratiform clouds D – flying in super cooled drizzle Ref: all Ans: B 9616. Which of
the following factors have the greatest effect on the formation of the various
types of ice on an aircraft? A – Relative humidity inside the cloud B – Aircraft
speed and size of cloud droplets C – Aircraft speed and curvature of the airfoil D
– Cloud temperature and droplet size Ref: all Ans: D 9618. Freezing fog exists if
fog droplets: A – are frozen B – are super cooled C – are freezing very rapidly D –
freeze when temperature falls below zero Ref: AIR: atpl, ir; HELI: all Ans: B 9621.
Two aircraft, one with a sharp wing profile (S), and the other with a thick profile
(T), are flying through the same cloud with same true airspeed. The cloud consists
of small super cooled droplets. Which of the following statements is most correct
concerning ice accretion? A – Aircraft T experiences more icing than S B – Aircraft
S experiences more icing than T C – Aircraft S and T experience the same amount of
icing D – Neither of the aircraft accumulate ice due to the small size of droplets
Ref: all Ans: B
9624. A winter day in northern Europe with a thick layer of stratocumulus clouds
and temperature close to zero degrees C at ground level, you can expect: A –
Reduced visibility and light icing in clouds B – Decreasing visibility due to
snowfall below cloud base, but only light icing in clouds C – A high probability
for icing in clouds. Severe icing in the upper part due to accumulation of large
droplets D – Turbulence due to a strong inversion, but no icing because clouds
consist of ice crystals Ref: all Ans: C 9626. Which of the following cloud types
are most likely to produce light to moderate icing when they are not subject to
orographic lifting and consist of super cooled cloud droplets? A – Stratocumulus
and cirrostratus B – Altocumulus and altostratus C – Stratus and cumulonimbus D –
Altostratus and cirrocumulus Ref: all Ans: B 10341. At what degree of icing should
ICAOs change course and/or altitude immediately instruction be followed? A – Light
B – Severe C – Moderate D – Extreme Ref: AIR: atpl, ir; HELI: all Ans: B 10345.
Clear ice forms on an aircraft by the freezing of: A – water vapour B – large super
cooled water drops C – small super cooled water drops D – snow Ref: AIR: atpl, ir;
HELI: all Ans: B
10353. Clear ice is dangerous because it: A – spreads out and contains many air
particles B – is translucent and only forms at the leading edges C – is not
translucent and forms at the leading edges D – is heavy and is difficult to remove
from the aircraft surfaces Ref: AIR: atpl, ir; HELI: all Ans: D 10358. The most
dangerous form of airframe icing is: A – rime ice B – hoar frost C – dry ice D –
clear ice Ref: AIR: atpl, ir; HELI: all Ans: D 10368. In which environment is
aircraft structural ice most likely to have the highest rate of accumulation? A –
Cirrus clouds B – Freezing rain C – Stratus clouds D – Snow Ref: AIR: atpl, ir;
HELI: all Ans: B 10373. At what degree of icing should ICAOs change of course
and/or altitude desirable recommendation be followed: A – Light B – Moderate C –
Severe D – Extreme Ref: AIR: atpl, ir; HELI: all Ans: B
10376. Atmospheric soundings give the following temperature profile: 3000 ft 15oC
6000 ft 8oC 10000 ft 1oC 14000 ft -6oC 18000 ft -14oC 24000 ft -26oC. At which of
the following flight levels is the risk for aircraft icing, in cloud, greatest? A –
FL 80 B – FL 150 C – FL 180 D – FL 220 Ref: all Ans: B 10377. Clear ice forms as a
result of: A – super cooled water droplets spreading during the freezing process B
– water vapour freezing to the aircraft C – ice pellets splattering on the aircraft
D – super cooled droplets freezing on impact Ref: AIR: atpl, ir; HELI: all Ans: A
10378. Rime ice forms through the freezing onto aircraft surfaces of: A – water
vapour B – large super cooled water drops C – small super cooled water drops D –
snow Ref: AIR: atpl, ir; HELI: all Ans: C 10379. How does a pilot react to heavy
freezing rain at 2000 ft/AGL, when he is unable to de-ice nor land? A – He turns
back before the aircraft loses manoeuvrability B – He descends to the warm air
layer below C – He ascends to the cold air layer above D – He continues to fly at
the same altitude Ref: all Ans: A
10386. Under which conditions would you expect the heaviest clear ice accretion to
occur in a CB? A – Close to the freezing level B – Between -20oC and -30oC C –
Between -2oC and -15oC D – Between -30oC and -40oC Ref: all Ans: C 10391. Glaze or
clear ice is formed when super cooled droplets are: A – small and at a temperature
just below freezing B – large and at a temperature just below freezing C – small
and freeze rapidly D – of any size at temperatures below -35oC Ref: AIR: atpl, ir;
HELI: all Ans: B 10392. The type of icing that occurs in dense clouds with large
super cooled drops that have a temperature of -5oC is most likely to be: A – clear
ice B – hoar frost C – rime ice D – cloudy ice Ref: AIR: atpl, ir; HELI: all Ans: A
10430. At what degree of icing can ICAO (no change of course and altitude
necessary) recommendation be followed? A – Moderate B – Light C – Severe D –
Extreme Ref: AIR: atpl, ir; HELI: all Ans: B
10435. Hoar frost forms on an aircraft as a result of: A – water vapour turning
directly into ice crystals on the aircraft surface B – freezing rain striking the
aircraft C – droplets forming on the aircraft and then freezing D – small super
cooled droplets striking the aircraft Ref: AIR: atpl, ir; HELI: all Ans: A 10442.
Which of the following statements is true regarding moderate-to-severe airframe
icing? A – It may occur in the uppermost levels of a cumulonimbus capillatus
formation B – It always occurs in altostratus cloud C – It is unlikely to occur in
nimbostratus cloud D – It will not occur in clear sky conditions Ref: all Ans: D
15839. Which of the following statements is true regarding moderate to severe
airframe icing? A – It may occur in the uppermost levels of a cumulonimbus
capillatus formation B – It always occurs in altostratus cloud C – It is likely to
occur in nimbostratus cloud D – It will occur in clear sky conditions Ref: all Ans:
C 15873. During the formation of rime ice in flight, water droplets freeze: A –
rapidly and do not spread out B – slowly and do not spread out C – slowly and
spread out D – rapidly and spread out Ref: all Ans: A
15875. A vertical temperature profile indicates the possibility of severe icing
when the temperature profile: A – coincides with a dry adiabatic lapse rate B –
indicates temperatures below -40oC C – indicates temperatures above 3oC D –
intersects the 0oC isotherm twice Ref: all Ans: D 16327. The icing in cloud which
forms over hills is likely to be more severe than in the same type of cloud over
level terrain because: A – orographic lifting causes the freezing level to rise and
increases the free water content of the cloud B – increases the temperature inside
the cloud by forcing the release of latent heat so causing the air to hold more
water vapour C – enforced ascent of air releases more water, which is retailed in
the cloud by the increased upward components D – adiabatic cooling lowers the
freezing level and the water content of the cloud Ref: all Ans: C 16441. Which one
of the following statements concerning icing in freezing rain is correct? A – A
coating of clear ice forms when water vapour sublimates B – Rime ice forms when
large water drops freeze C – Clear ice forms when large water drops freeze D – Hoar
frost form when small water droplets freeze Ref: AIR: atpl, ir; HELI: all Ans: C
16442. One of the most serious consequences of icing on the wings of an aircraft
is? A – The wing construction being unable to bear the increased load B – Lift
becoming too great as the aircraft becomes heavier C – The lift co-efficient
increasing as the aircraft becomes heavier D – The stalling speed increasing
substantially Ref: AIR: atpl, ir; HELI: all Ans: D
16443. Icing on the wings of an aircraft normally causes: A – the stalling speed to
increase B – the stalling speed to decrease C – the interference drag to increase D
– the interference drag to decrease Ref: AIR: atpl, ir; HELI: all Ans: A 16444. A
thin coating of hoar frost on the airfoil surface: A – does not affect take-off
performance B – has deleterious effects on the lift of the wing C – affects the
aspect ratio of the wing D – only affects stability Ref: AIR: atpl, ir; HELI: all
Ans: B 16445. Which one of the following precipitation types gives the most severe
icing? A – Snowfall B – Mixed rain and snow C – Freezing rain D – Ice pellets Ref:
all Ans: C 24226. After a prolonged VMC descent in very cold air, you penetrate a
humid air mass. What type of icing will you encounter? A – Hoar frost B – Rime ice
C – Clear ice D – Smooth icing Ref: AIR: atpl, ir; HELI: all Ans: A
24234. An aircraft descends in layered clouds; the freezing level is situated at FL
60. At what levels is the probability for airframe icing the highest? A – Between
FL 120 and FL 60 B – Between FL 120 and FL 180 C – Between FL 60 and Fl 20 D – At
FL 140 Ref: all Ans: A 24235. An aircraft flies into super cooled rain in an area
with a temperature below 0oC. The type of icing it will most likely encounter is A
– granular frost B – hoar frost C – rime ice D – clear ice Ref: AIR: atpl, ir;
HELI: all Ans: D 24267. For a VFR aircraft, the conditions in which it could
encounter severe airframe icing are: A – flight into an area outside of clouds
where the temperature is below 0oC, resulting in rime ice formation B – flight into
super cooled rain, resulting in rime ice formation C – flight into freezing rain,
resulting in clear ice formation D – flight between two cloud layers, without
precipitation, resulting in clear ice formation Ref: all Ans: C 24278. Ice
accretion to the airframe is likely to be most hazardous at temperatures: A –
between 0oC and -17oC in ST B – between 0oC and -17oC in AS C – between 0oC and
-23oC in large CU D – below -40oC in CB Ref: all Ans: C
24286. In mature CB’s the probability of severe icing, according to meteorological
rules, is greatest in the following temperature range: A - -23oC to -40oC B - +5oC
to 0oC C – 0oC to -23oC D - -40oC to -60oC Ref: all Ans: C 24298. In which
conditions would you most likely encounter clear icing, and how would it normally
appear? A – Cumuliform clouds, small water droplets, temperatures between -20oC and
25oC. Appears transparent and tends to take the shape of the surface on which it
freezes B – Stratiform clouds, small water droplets, temperatures between -10oC and
–20oC. Appears granular and tends to accumulate forward into the air stream C –
Cumuliform clouds, large water droplets, temperatures between 0oC and – 15oC.
Appears smooth and tends to spread back over an aircraft wing D – Stratiform
clouds, large water droplets, temperatures well below freezing. Appears opaque and
builds forward from leading surface into a sharp edge Ref: all Ans: C 24334. The
formation of clear ice on the leading edges of an aircraft is most likely to be
caused by the: A – relatively slow freezing of small super cooled water droplets
and ice crystals B – instantaneous freezing of small super cooled water droplets C
– instantaneous freezing of large super cooled water droplets and snow crystals D –
relatively slow freezing of large super cooled water droplets Ref: all Ans: D
24340. The most dangerous icing conditions are encountered in: A – icy clouds at
high levels B – super cooled precipitation C – unstable clouds at medium levels D –
zones where the air temperature is below -15oC Ref: all Ans: B 24360. What are the
characteristics of rime ice, and what conditions are most favourable for its
formation? A – Milky granular appearance, forming on leading edges and accumulating
forward into the air stream. Stratiform clouds at temperatures of -10oC to -20oC
are most conducive to its formation B – Opague rough appearance, tending to spread
back over an aircraft surface. Most frequently encountered in cumuliform clouds at
temperatures slightly below freezing C – Smooth appearance and builds forward from
leading surfaces into a sharp edge. Most common in cumuliform clouds at
temperatures of -20oC to – 25oC D – Transparent appearance and tendency to take the
shape of the surface on which it freezes. Stratiform clouds and temperatures only
slightly below freezing promote its formation Ref: all Ans: A 24364. What intensity
and type of aircraft icing is likely to occur at FL 100 in a thunderstorm, with the
freezing level at 7000 ft? A – Moderate to severe icing due to clear ice B – Light
icing due to rime ice C – Moderate to severe icing due to rime ice D – Light icing
due to clear ice Ref: AIR: atpl, ir; HELI: all Ans: A
24365. What intensity and type of airframe icing is most likely to occur when
aircraft descends rapidly from FL 320 (temp -45oC) to FL 60 in warm, moist clear
air? A – Light or moderate hoar frost B – Moderate opaque rime C – Nil ice D –
Light opaque rime and light clear ice Ref: AIR: atpl, ir; HELI: all Ans: A 24366.
What intensity and type of airframe icing is most likely to occur when flying at FL
170 in AS with the outside air temperature at -20oC? A – Moderate – hoar frost B –
Severe – clear C – Light – rime D – Moderate – clear Ref: AIR: atpl, ir; HELI: all
Ans: C 24428. Which of the following statements is correct? A – Airframe icing can
occur in clear air B – Haze is a reduction of visibility due to the presence of
water vapour C – Mountain waves are always accompanied by rotor clouds D – Above
the tropopause no turbulence occurs Ref: AIR: atpl, ir; HELI: all Ans: A 24433.
Which of the following statements is true regarding moderate to severe airframe
icing? A – It will not occur in clear sky conditions B – It always occurs in
altostratus cloud C – It is unlikely to occur in nimbostratus cloud D – It will
occur in the anvil part of the cumulonimbus cloud Ref: all Ans: A
25633. The presence of ice pellets at the surface is the evidence that: A –
temperatures are above freezing at some higher altitudes B – a cold front has
passed C – there are thunderstorms in the area D – after take-off you can climb to
a higher altitude without encountering more than light icing conditions Ref: all
Ans: A

050-09-02 Turbulence
9603. What is normally the most effective measure to reduce or avoid CAT effects? A
– Decrease of speed B – Change of course C – Increase of speed D – Change of flight
level Ref: AIR: atpl; HELI: atpl, cpl Ans: D 9625. A zone of strong convection
currents is encountered during a flight. In spite of moderate gust you decide to
continue the flight. What are your precautionary measures? A – Increase the
speed/try to descend below the zone of convective currents B – Increase the
speed/try to climb above the zone of convective currents, if aircraft performance
parameters allow C – Decrease the speed/try to descend below the zone of convective
currents D – Decrease the speed/try to climb above the zone of convective currents
if aircraft performance parameters allow Ref: all Ans: D
10360. The turbulence which occurs at high flight levels (above FL 250) is mainly
of the type Clear Air Turbulence. In what way can moderate to severe clear air
turbulence affect an aircraft, the flight and the passengers? A – The turbulence is
a small scale one and can cause damage of worn out type. The manoeuvring of the
aircraft will be made more difficult or even impossible. For the passengers the
flight will be unpleasant B – The turbulence is a large scale one (waving) so that
the aircraft will be difficult to manoeuvre. The passengers will feel some
discomfort C – The turbulence can be resembled with the roughness of a washing
board (small scale) and will not have influence on the aircraft and its solidity,
but will make flight a little more difficult. The passengers will seldom notice
anything of this turbulence D – The turbulence is wave like which makes the flight
unpleasant for the passengers but the manoeuvring will not be affected essentially
Ref: all Ans: A 10369. How does moderate turbulence affect an aircraft? A – Large,
abrupt changes in altitude or altitude occur but the aircraft may only be out of
control momentarily B – Rapid and somewhat rhythmic bumpiness is experienced
without appreciable changes in altitude or attitude C – Changes in altitude or
attitude occur but the aircraft remains in positive control at all times D –
Continued flight in this environment will result in structural damage Ref: all Ans:
C 10384. The degree of clear air turbulence experienced by an aircraft is
proportional to the: A – intensity of the solar radiation B – height of the
aircraft C – stability of the air D – intensity of vertical and horizontal wind
shear Ref: all Ans: D
10420. An aircraft is approaching a cold front from the warm air mass side at FL
270 and experiencing moderate to severe turbulence. A jet stream is forecast to be
at FL 310. The shortest way to get out of this turbulence is by: A – Climbing B –
Descending C – Turning right D – Maintain FL 270 Ref: all Ans: B 15795. The jet
stream and associated clear air turbulence can sometimes be visually identified in
flight by: A – a high pressure centre at high level B – long streaks of cirrus
clouds C – dust or haze at high level D – a constant outside air temperature Ref:
all Ans: B 15848. All pilots encountering Clear Air Turbulence are requested to
report it. You experience CAT which causes passengers and crew to feel definite
strain against their seat below or shoulder straps. Unsecured objects are
dislodged. Food service and walking are difficult. This intensity of CAT should be
reported as: A – severe B – extreme C – moderate D – light Ref: AIR: atpl; HELI:
atpl, cpl Ans: C
16324. Maximum turbulence associated with the standing waves is likely to be: A –
two wavelengths down wind and just above the surface B – approximately one
wavelength down wind of, and approximately level with, the top of the ridge C –
just below the tropopause above the ridge D – down the lee side of the ride and
along the surface Ref: all Ans: D 16346. Possible severe turbulence will be
encountered in mountain waves in: A – roll cloud B – lenticular cloud C – at the
crest of the mountain D – on the windward side Ref: all Ans: A 16608. Conditions
favourable for low-level frictional turbulence are: A – strong wind, rough terrain,
steep lapse rate B – strong wind, rough terrain, stable lapse rate C – light wind,
rough terrain, stable lapse rate D – strong wind, smooth terrain, stable lapse rate
Ref: all Ans: A 24177. (Refer to figure 050-70) Flight Zurich to Rome, ETD 1600
UTC, ETA 1800 UTC. At what flight level would you first expect to encounter clear
air turbulence on the climb out from Zurich? A – FL 140 B – FL 160 C – FL 320 D –
FL 220 Ref: all Ans: D
24204. (Refer to figure 050-73) On which of the following routes would you not have
to worry about oderate and/or severe turbulence on the cruising level? A – Keflavik
to Oxlo at FL 220 B – Rome to Frankfurt at FL 320 C – Beirut to Athens at FL 310 D
– Madrid to Zurich at FL 280 Ref: all Ans: A 24212. A layer is conditionally
unstable if the air: A – becomes stable by lifting it B – is stable for satuated
air and unstable for dry air C – is unstable for saturated air and stable for dry
air D – is unstable for saturated air as well as for dry air Ref: all Ans: C 24301.
In which zone of a polar front jet stream is the strongest CAT to be expected? A –
Exactly in the centre of the core B – On the tropical air side of the core C – On
the polar air side of the core D – About 12000 ft above the core Ref: AIR: atpl;
HELI: atpl, cpl Ans: C 24318. Stratus formed by turbulence will occur when: A – the
wind speed is greater than 10 kt and the condensation level is situated just above
the turbulent layer B – the wind speed is less than 10 kt and the air is heated by
the earth’s surface C – in the friction layer mixing occurs by turbulence and the
condensation level is situated below the top of the turbulent layer D – absolute
instability exists at low level Ref: all Ans: C
24341. The most likely place to encounter clear air turbulence associated with a
jet stream is: A – well below the core B – close to the core on the side facing the
polar air C – on the tropical side of the core D – 5000 feet or more above the core
Ref: all Ans: B 24441. Which statement does correspond to the definition of severe
turbulence? A – Aircraft will be damaged and an emergency landing will be
absolutely necessary B – Difficulty in walking, occupants feel strain against seat
belts, loose objects move about C – Aircraft gets out of control and crashes D –
Aircraft may be out of control for short periods occupants are forced violently
against seat belts, loose objects are tossed about Ref: all Ans: D

050-09-03 Windshear
9594. Above and below a low level inversion the wind is likely to: A – experience
little or no change in speed and direction B – change in speed but not in direction
C – change in direction but not in speed D – change significantly in speed and
direction Ref: all Ans: D 9605. Vertical wind shear is: A – horizontal variation in
the horizontal wind B – vertical variation in the vertical wind C – vertical
variation in the horizontal wind D – horizontal variation in the vertical wind Ref:
all Ans: C
10342. The most dangerous low level wind shears are encountered: A – when strong
ground inversions are present and near thunderstorms B – in areas with layered
clouds and wind speeds higher than 35 kts C – during any period when wind speed is
greater than 35 kt and near valleys D – near valleys and at the windward side of
mountains Ref: all Ans: A 10356. Low level wind shear is likely to be greatest…. A
– at the condensation level when there is a stong surface friction B – at the
condensation level when there is no night radiation C – at the top of the friction
layer D – at he top of a marked surface-based inversion Ref: all Ans: D 10417. Low
level vertical wind shear can be expected during the night: A – and early morning
only in summer B – in unstable atmospheres C – and early morning only in winter D –
in association with radiation inversions Ref: all Ans: D 10431. What is the effect
of a strong low level inversion? A – It results in good visual conditions B – It
promotes extensive vertical movement of air C – It prevents vertical wind shear D –
It promotes vertical wind shear Ref: all Ans: D
16326. An important characteristic of wind shear is that: A – it is encountered
most frequently near mountain waves during winter months B – it is an atmospheric
condition that is associated exclusively with zones of convergence C – it only
occurs with jet streams D – it is an atmospheric condition that may be associated
with a low level temperature inversion, a jet stream or a frontal zone Ref: all
Ans: D 16386. An additional hazard to aircraft taking-off or landing in or near a
thunderstorm is: A – extra turbulence B – wind shear C – compass error D – pilot
disorientation Ref: all Ans: B 24139. (Refer to figure 050-27) In which squares are
conditions most likely to cause the occurrence of low level wind shear? A – 3B and
3C B – 3A and 3B C – 3A and 3C D – 3B and 3D Ref: all Ans: C 24288. In relation to
the polar front jet stream, the greatest rate of wind shear is most likely to occur
A – on the tropical side of the core B – well below the core C – close to the core
on the polar side D – 5000 ft or more above the core Ref: all Ans: C
24405. Where does wind shear occur? A – At any level in the atmosphere if
associated with either a change of wind direction and/or wind speed B – Wind shear
of any significance occurs only in connection with jet streams C – Wind shear
occurs primarily at lower altitudes in the vicinity of mountain waves D – Wind
shear occurs only when there is a strong temperature inversion, or when the jet
stream is associated with a strong depression Ref: all Ans: A 24489. Vertical wind
shear is: A – a change of horizontal wind direction and/or speed with height B – a
change of vertical wind speed with horizontal distance C – a change of horizontal
wind direction and/or speed with horizontal distance D – a horizontal shear of
vertical wind Ref: all Ans: A

050-09-04 Thunderstorms
9531. Which thunderstorms move forward the fastest? A – Thunderstorms formed by
lifting processes B – Frontal thunderstorms C – Thermal thunderstorms D –
Orographic thunderstorms Ref: all Ans: B 9538. Where is a squall line to be
expected? A – At the surface position of a warm front B – Behind a cold front C –
In front of a cold front occlusion at higher levels D – In front of an active cold
front Ref: all Ans: D
9539. A microburst phenomenon can arise in the: A – down-draught of a cumulonimbus
at the mature stage B – up-draught of a cumulonimbus at the mature stage C – down-
draught of a cumulonimbus at the formation stage D – up-draught of a cumulonimbus
at the growth stage Ref: all Ans: A 9547. Which of the following statements
describes a microburst? A – A high speed down burst of air with a generally lower
temperature than its surroundings B – A high speed down draft of air with a higher
temperature than its surroundings C – An extremely strong wind gust associated with
a troical revolving storm D – A small low pressure system where the wind circulates
at high speed Ref: all Ans: A 9554. What is a microburst? A – A small low pressure
system where the wind circulates with very high speeds B – A concentrated down
draft with high speeds and a high temperature than the surrounding air C – An
extremely strong wind gust in a tropical revolving storm D – A concentrated down
draft with high speeds and a lower temperature than the surrounding air Ref: all
Ans: D 9555. A gust front is: A – normally encountered directly below a
thunderstorm B – formed by the cold air outflow from a thunderstorm C –
characterised by heavy lightning D – another name for a cold front Ref: all Ans: B
9574. Continuous up-draughts occur in a thunderstorm during the: A – dissipating
stage B – mature stage C – initial stage D – period in which precipitation is
falling Ref: all Ans: C 9575. At which altitude, at temperate latitudes, may hail
be expected in connection with a CB? A – From the ground up to a maximum of FL 450
B – From the ground up to about FL 200 C – From the ground up to about FL 100 D –
From the base of the clouds up to FL 200 Ref: all Ans: A 9577. What are squall
lines? A – Unusual intensive cold fronts B – Bands of intensive thunderstorms C –
The surface weather associated with upper air troughs D – The paths of tropical
revolving storms Ref: all Ans: B 9583. The diameter and the life time of a typical
microburst are in the order of: A – 8 km and 5-15 minutes B – 4 km and 30-40
minutes C – 4 km and 1-5 minutes D – 12 km and 5-10 minutes Ref: all Ans: C
9596. During which stage of thunderstorm development are rotor winds characterised
by roll clouds most likely to occur? A – Dissipating stage B – Cumulus stage C –
Mature stage D – Cumulus stage and mature stage Ref: all Ans: C 9607. What are the
meteorological pre-requisites, at low level, for thunderstorms formed by lifting
processes over land? A – Low temperatures, low humidity B – High air pressure
(>1013 hPa), high temperatures C – High temperatures, high humidity D – Subsidence,
inversion Ref: all Ans: C 9622. What weather condition would you expect at a squall
line? A – Strong steady rain B – Thunderstorms C – Fog D – Strong whirlwinds
reaching up to higher levels Ref: all Ans: B 10343. In addition to a lifting
action, what are two other conditions necessary for thunderstorm formation? A –
Stable conditions and high moisture content B – Unstable conditions and high
moisture content C – Stable conditions and low atmospheric pressure D – Unstable
conditions and low atmospheric pressure Ref: all Ans: B
10359. How long does a typical microburst last? A – About 30 minutes B – Less than
1 minute C – 1 to 2 hours D – 1 to 5 minutes Ref: all Ans: D 10366. Which
thunderstorms generally develop in the afternoon in summer over land in moderate
latitudes? A – Occlusion thunderstorms B – Warm front thunderstorms C – Cold mass
thunderstorms D – Air mass thunderstorms Ref: all Ans: D 10381. In Central Europe
when is the greatest likelihood for thunderstorms due to warm updrafts? A – Mid-
afternoon B – Around midnight C – Early morning D – Late morning Ref: all Ans: A
10383. Aircraft struck by lightning may sometimes get considerable damage and at
least temporarily the manoeuvring of the aircraft will be made more difficult.
Which one of the following statements is correct? A – An aircraft has in the
atmosphere the same qualities as a “Faradays Cage”, which means that struck of
lightning seldom occurs. But if it happens, the result will be an occasional engine
failure. The crew may get a shock B – An aircraft made by metal has a certain
capacity to attract a lightning, but the lightning will follow the surface and
therefore no damage will be caused C – Aircraft made by composite material may get
severe damage, the crew may be blinded and temporarily lose the hearing D –
Aircraft made by composite material cant conduct a lightning and will therefore
very seldom be struck Ref: all Ans: C 10385. During the life cycle of a
thunderstorm, which stage is characterised predominantly by down drafts? A –
Dissipating stage B – Cumulus stage C – Mature stage D – Anvil stage Ref: all Ans:
A 10394. What feature is normally associated with the initial stage of a
thunderstorm? A – Frequent lightning B – Roll cloud C – Continuous updraft D – Rain
or hail at the surface Ref: all Ans: C
10400. Thunderstorms reach their greatest intensity during the: A – dissipating
stage B – cumulus stage C – mature stage D – period in which precipitation is not
falling Ref: all Ans: C 10402. In which stage of the life cycle of a single
thunderstorm cell occur both up and down drafts simultaneously? A – Dissipating
stage B – Cumulus stage C – Mature stage D – In all stages Ref: all Ans: C 10409. A
microburst: A – is always associated with thunderstorms B – has a life time of more
than 30 minutes C – has a diameter up to 4 km D- occurs only in tropical areas Ref:
all Ans: C 10410. The initial phase of a thunderstorm is characterised by: A – rain
starting at surface B – continuous down draughts C – frequent lightning D –
continuous up draughts Ref: all Ans: D
10421. The most hazardous type of cloud that may be encountered on a cross country
flight is: A – cirrus B – stratocumulus C – cumulus D – cumulonimbus Ref: all Ans:
D 10425. Which of the following meteorological phenomenon indictes upper level
instability which may lead to thunderstorm development? A – Red cirrus B – AC
lenticularis C – Halo D – AC castellanus Ref: all Ans: D 10438. Which thunderstorms
generally product the most severe conditions, such as heavy hail and destructive
winds? A – Warm front thunderstorms B – Squall line thunderstorms C – Nocturnal air
mass thunderstorms D – Daytime air mass thunderstorms Ref: all Ans: B 15817.
Isolated thunderstorms of a local nature are generally caused by: A – frontal
occlusion B – thermal triggering C – frontal lifting (warm flood) D – frontal
lifting (cold front) Ref: all Ans: B
15869. What are the requirements for the formation of a thunderstorm? A – A cumulus
cloud with sufficient moisture associated with an inversion B – An adequate supply
of moisture, conditional instability and a lifting action C – Water vapour and high
pressure D – A stratocumulus cloud with sufficient moisture Ref: all Ans: B 15870.
In which of the following areas is the highest frequency of thunderstorms
encountered? A – Sub-tropical B – Temperate C – Tropical D – Polar Ref: all Ans: C
15871. Thunderstorms can occur on a warm front if the: A – cold air is moist and
the environmental lapse rate is less than the dry adiabatic lapse rate B – warm air
is moist and the environmental lapse rate is less than the saturated adiabatic
lapse rate C – cold air is moist and the environmental lapse rate exceeds the
saturated adiabatic lapse rate D – warm air is moist and the environmental lapse
rate exceeds the saturated adiabatic lapse rate Ref: all Ans: D 16359. In general
terms, an intense contour low will indicate: A – quiet settled weather B –
changeable weather but getting better C – heavy rain, hail and thunderstorm D – no
change to the weather Ref: all Ans: C
16367. Which of the following is consered to be one of the conditions necessary for
thunderstorm development? A – A marked temperature inversion extending well above
the 0 degree isotherm B – ELR less than the DALR extending well above the 0 degree
isotherm C – Instability throughout a deep layer extending well above the 0 degree
isotherm D – Stability throughout a deep layer extending well above the 0 degree
isotherm Ref: all Ans: C 16368. Which of the following conditions are necessary for
thunderstorms? A – Low surface temperatures and high moisture content B – High
moisture content and a trigger action C – Instability at night D – Low upper
temperatures, and a high 0 degree isotherm Ref: all Ans: B 16369. There are two
principal types of origin of thunderstorm A – Air mass thunderstorms and frontal
thunderstorms B – Air mass thunderstorms and heat thunderstorms C – Convection
thunderstorms and frontal thunderstorms D – Convection thunderstorms and orographic
uplift thunderstorms Ref: all Ans: A 16372. Air mass thunderstorms are triggered
off by: A – fronts and/or orographic uplift B – convection at air mass boundaries C
– standing wves in the lee of hills D – convection and/or orographic uplift Ref:
all Ans: D
16373. Frontal thunderstorms are triggered off by: A – rising air dur to falling
pressure at air mass boundaries B – convection by intense daytime heating or cold
air moving over a warm surface C – rising air due to falling pressure or due to
orographic uplift D – mass ascent over large areas of cold air moving over a warm
surface Ref: all Ans: A 16374. A thunderstorm has the following stages in its life
cycle: A – cumulus, mature or building and declining B – building, mature,
declining and dissipating C – cumulus, declining and dissipating D – cumulus of
building, mature and dissipating Ref: all Ans: D 16375. The building stage of a
thunderstorm last for approximately: A – 20 min B – 30 min C – 20/.30 min D – 40
min Ref: all Ans: A 16376. In the building stage of a thunderstorm, which of the
following is true? A – Only up-currents are present B – Only down-currents are
present C – Both up-currents and down-currents are present D – Up-currents and
side-currents are present Ref: all Ans: A
16377. The mature stage of a thunderstorm lasts for approximately: A – 15 min B –
30 min C – 20/30 min D – 40 min Ref: all Ans: C 16378. Which of the following
statements is true with regard to a mature thunderstorm: A – both up-currents and
down-currents appear concurrently B – the cloud top assumes an anvil shape C –
water droplets, hail and snow are all present D – the top of the cloud is
negatively charged and the base is positively charged Ref: all Ans: A 16379. The
final stage of a thunderstorm is reached when: A – no further electrical charge is
developed B – the lower portion of the cloud dissipates C – a well developed anvil
can be seen D – all of the above Ref: all Ans: C 16381. The most common hazards of
thunderstorms to aircraft on or near the ground are: A – lightning and hail B –
heavy precipitation and surfaces squalls C – turbulence and static D – all of the
above Ref: all Ans: D
16383. Which types of thunderstorm can be the most difficult to avoid? A – Mature
thunderstorms because they spread over such a wide area B – Frontal thunderstorms C
– Air mass thunderstorms D – Low level thunderstorms Ref: all Ans: B 16384. Which
thunderstorms are the more difficult to forecast and detect? A – cumulus
thunderstorms B – air mass thunderstorms C – frontal thunderstorms D – cumulus
thunderstorms because they are smaller Ref: all Ans: B 16385. If you cannot avoid
penetrating a thunderstorm, which is the best area to penetrate? A – The top B –
The middle C – The bottom D – The sides Ref: all Ans: D 24180. (Refr to figure 050-
72) At which position is the development of thunderstorms most likely, and what is
the maximum height of the CB clouds at 00 UTC? A – Position C, FL 200 B – Position
A, FL 200 C – Position B, FL 270 D – Position D, FL 290 Ref: all Ans: C
24207. A Cb with thunderstorm has reached the mature stage. Which statement is
correct? A – The freezing level in the whole cloud lies lower than outside the
cloud B – In temperatures lower than -23oC icing is still possible C – If hail
occurs, it only occurs in down drafts D – Severe turbulence occurs in the cloud,
but hardly ever below the cloud Ref: all Ans: B 24215. A microburst with its
damaging winds at the surface: A – has a diameter up to 4 km B – has a life time of
more than 30 minutes C – is always associated with thunderstorms D – occurs only in
tropical areas Ref: all Ans: A 24246. Assuming that an initial “trigger” force is
present, the conditions most likely to result in the formation of thunderstorms
are: A – high surface temperature, low dew point and high dry adiabatic lapse rate
B – high relative humidity and instability throughout a deep layer C – rapid
orographic cooling of cloud containing ice crystals D – intense surface heating,
anti-cyclonic pressure system and relatively high freezing level Ref: all Ans: B
24249. At the surface the lifetime of a typical microburst and the diameter of the
area affected by damaging winds are in the order of: A – 1-5 minutes and 4 km B –
30-40 minutes and 4 km C – 5-15 minutes and 8 km D – 5-10 minutes and 12 km Ref:
all Ans: A
24261. Convective weather phenomena include: A – heavy showers, lightning strikes,
hail, mist, squalls, light rain over a large area B – thunderstorms, tornadoes,
hail, haze, wind gusts, advection fog C – thunderstorms, hail, tornadoes, wind
gusts, heavy showers, lightning strikes D – hail, lightning strikes, wind lulls,
squalls, stratocumulus, low level wind maximum Ref: all Ans: C 24299. In which of
the following situations is the probability for severe thunderstorms the highest? A
– Advection of continental cold air over a warm land surface B – Advection of warm
air over a cold land surface C – Advection of maritime cold air over a warm sea
surface D – Advection of maritime warm air over a cold sea surface Ref: all Ans: C
24358. Thunderstorms are often preceded by: A – Altostratus B – Nimbostratus C –
Altocumulus Castellanus D – Altocumulus Lenticularis Ref: all Ans: C 24367. What is
a down burst? A – A concentrated downdraft with high speeds and a lower temperature
than the surrounding air B – A concentrated downdraft with high speeds and a higher
temperature than the surrounding air C – An extremely strong wind gust in a
tropical revolving storm D – A small low pressure system where the wind circulates
with very high speeds Ref: all Ans: A
24373. What is the approximate maximum diameter of a area affected by damaging
winds at the surface caused by a microburst? A – 20 km B – 400m C – 4 km D – 50 km
Ref: all Ans: C 24421. Which of the following sets of conditions are most
favourable to the development of thunderstorms? A – Extensive isothermal layer, ice
particles and water droplets must exist just below the freezing level and
orographic lifting B – Environmenal lapse rate greater than saturated adiabatic
lapse rate through a great vertical extent, high relative humidity and an initial
lifting process C – Environmental lapse rate less than saturated adiabatic lapse
rate with dew point below 0oC and considerable surface heating D – Environmental
lapse rate less than dry adiabatic lapse rate with freezing level below the cloud
base, high relative humidity and strong surface winds Ref: all Ans: B 24423. Which
of the following situations favours the formation of heavy thunderstorms? A – A
cold front approaching a mountain range in the evening B – The passage of a warm
front in the morning C – A cold front on the leeward side of a mountain range D – A
warm sector moving over a snow-covered ground Ref: all Ans: A
24424. Which of the following statements about lightnings and lightning strikes is
correct? A – The aircraft is temporarily part of the lightning trajectory B –
Spherical lightnings often penetrate into aircraft C – Lightning strikes always
cause heavy damage D – Compasses and electronics are always affected Ref: all Ans:
A 24436. Which one of the following can provide the initial lifting leading to air
mass thunderstorms? A – Advection of cold air over a warm sea B – Mountain waves C
– Advection of warm air over a cold sea D – Low level wind shear Ref: all Ans: A
24445. Which statement is correct for microbursts? A – They only develop below
convective clouds with heavy rain B – The diameter of the affected area on the
surface does not exceed 4 km C – They occur in the tropics only D – Their downdraft
is warmer than the surroundings Ref: all Ans: B 24455. With the development of a
thunderstorm, at what stage will there be only updraughts of air? A – Anvil stage B
– Mature stage C – Initial stage D – Dissipating stage Ref: all Ans: C
050-09-05 Tornadoes
9544. The diameter of a typical tornado is: A – only a few metres B – 100 to 150
metres C – about 2 to 6 km D – in the order of 10 km Ref: all Ans: B 9606. At what
time of the year are tornadoes most likely to occur in North America? A – Spring,
summer B – Summer, autumn C – Autumn, winter D – Winter Ref: all Ans: A 10413. With
which type of cloud are tornadoes normally associated? A – Cumulonimbus B – Cumulus
C – Stratus D – Nimbostratus Ref: all Ans: A

050-09-06 Low and high level inversions


24320. Surface temperature inversions are frequently generated by: A – terrestrial
radiation on a calm clear night B – an unstable air mass causing convection
currents and mixing of the atmosphere at lower levels C – gusting winds increasing
surface friction during the day with consequent mixing at the lower levels D –
compression causing the release of latent heat in a layer of stratiform cloud Ref:
all
Ans: A 24321. Surface-based temperature inversions are common during: A – cloud
free nights in winter when the ground is dry B – cloud free days in summer when the
ground is dry C – cloudy days in summer when the ground is wet D – cloudy days in
winter when the ground is wet Ref: all Ans: A

050-09-07 Stratospheric conditions


No questions in this sub-chapter

050-09-08 Hazards in mountainous areas


10352. You intend to carry out a VFR flight over the Alpa, on a hot summer day,
when the weather is unstable. What is the best time of day to conduct this flight?
A – Morning B – Mid day C – Afternoon D – Early evening Ref: all Ans: A 10380.
Which of the following phenomena are formed when a moist, stablelayer of air is
forced to rise against a mountain range? A – Inversions B – Stratified clouds C –
Showers and thunderstorms D – Areas of severe turbulence Ref: all Ans: B
10398. You are flying with an outside air temperature of -12oC and a TAS of 250 kt
at FL 150 through 8 octas NS. What type and degree of icing is most probable? A –
In clouds pushed up against he mountains, moderate to severe rime ice B – In clouds
pushed up against the mountains, moderate to severe mixed ice C – Over flat
terrain, away from fronts, moderate to severe mixed ice D – Over flat terrain,
moderate hoar frost Ref: all Ans: B 15811. Which of the following conditions are
most favourable to the formation of mountain waves? A – Unstable air at mountain
top altitude and a wind at least 20 knots blowing across the mountain ridge B –
Either stable or unstable air at mountain top and a wind of at lest 30 knots
blowing parallel to the mountain ridge C – Moist unstable air at mountain top and
wind of less than 5 knots blowing across the mountain ridge D – Stable air at
mountain top altitude and a wind at least 20 knots blowing across the mountain
ridge Ref: all Ans: D 16325. For the formation of mountain waves, the wind above
the level of the ridge should: A – decrease or even reverse direction B – increase
initially, then decrease C – increase with little change in direction D – increase
and then reverse in direction Ref: all Ans: D
16337. With reference to rotor streaming: A – It can occur at levels up to 80,000
ft B – It is a low level phenomenon C – It is found at approximately the tropopause
level D – It is another name for the rotor zones associated with mountain waves
Ref: all Ans: B 24305. Mountain waves should be expected: A – when instability is
high B – on the downwind side of the mountain chain C – directly over the mountain
chain D – on the upwind side of the mountain chain Ref: all Ans: B 24401. When
severe mountain waves are present, where would the area of most severe turbulence
be located? A – Just below the tropopause B – Just above the cap cloud C – On the
windward side of the mountain range D – in the rotor zone Ref: all Ans: D 24434.
Which of the following statements is true with regard to mountain waves? A – The
absence of cloud over high ground indicates the absence of mountain waves B –
Mountain waves are not experienced beyond 100 miles downwind from initiating high
ground, regardless of the height of the ground C – Flight with headwind toward high
ground is likely to be more hazardous than flight with tailwind toward high ground
D – Flight with tailwind toward high ground is likely to be more hazardous than
flight with headwind toward high ground Ref: all Ans: C
24442. Which statement is correct concerning a mountain ridge where a marked
mountain wave has been reported? A – The atmosphere is unstable at the level of the
mountain tops B – There are always rotor clouds C – Ragged altocumulus lenticularis
is an indication for the presence of moderate/severe turbulence at the level of
these clouds D – The axis of a rotor is horizontal and perpendicular to the
mountains Ref: all Ans: C 25632. The conditions most favourable to the formation of
mountain waves are: A – wind direction parallel to the mountain range – wind speed
increasing with height – extensive isothermal layer between mountain crests and the
tropopause B – wind diretion parallel to the general alignment of the mountain
range – wind speed increasing with height – intense surface heating C – wind speed
less than 15 kt and wind direction at right angles to mountains – intense radiation
cooling at night particularly at the higher levels D – wind direction approximately
at right angles to the mountain range – wind speed 30 kt and steadily increasing
with height – an inversion just above the crest level with less stable air above
and below Ref: all Ans: D

050-09-09 Visibility reducing phenomena


9558. In general, the meteorological visibility during rainfall compared to during
drizzle is: A – less B – the same C – greater D – in rain – below 1 km, in drizzle
– more than 2 km Ref: all Ans: C
9572. Visibility is reduced by haze when: A – a cold front just passed B – a light
drizzle falls C – dust particles are trapped below an inversion D – small water
droplets are present Ref: all Ans: C 9600. In the vicinity of industrial areas,
smoke is most likely to affect surface visibility when: A – the surface wind is
strong and gusty B – there is a low level inversion C – cumulus clouds have
developed in the afternoon D – a rapid moving cold front has just passed the area
Ref: all Ans: B 10390. Below a low level inversion visibility is often: A –
moderate or poor due to heavy snow showers B – very good at night C – very good in
the early morning D – moderate or poor because there is no vertical exchange Ref:
all Ans: D 10405. Flight visibility from the cockpit during approach in a tropical
downpour can decrease to minimal: A – about 500 metres B – about 200 metres C –
tens of metres D – about 1000 metres Ref: all Ans: C
10422. In unstable air, surface visibility is most likely to be restricted by: A –
low stratus B – haze C – drizzle D – showers of rain or snow Ref: all Ans: D 16575.
The solid particles which reduce visibility in the atmosphere are: A – atmospheric
pollution, dust, ozone and moisture B – atmospheric pollution, volcanic fumes, dust
and sand C – atmospheric pollution, dust, sand and spray D – atmospheric pollution,
dust, sand and volcanic ash Ref: all Ans: D 16576. The forms of visible water which
reduce visibility in the atmosphere are: A – fog, spray, cloud and hail (but only
for short periods) B – cloud, fog, smog, spray or precipitation C – cloud, fog,
mist, spray or smog D – cloud, fog, mist spray or precipitation Ref: all Ans: D
16577. Reduction in visibility due to precipitation depends on: A – precipitation
type and duration B – precipitation duration and droplet size C – precipitation
intensity and duration D – precipitation intensity and type Ref: all Ans: D
24296. In the weather pattern behind a cold front, the visibility outside
precipitation is: A – low and the precipitation is showers B – good and the
precipitation is steady rain C – good and the precipitation is showers D – low and
the precipitation is steady rain Ref: all Ans: C 24361. What conditions are
necessary for vertical visibility to be reported? A – It is always reported B –
Whenever the sky is obscured by fog or heavy precipitation and the height of the
cloud base cannot be measured C – Whenever the lowest ceiling is below 1500 ft
above ground D – Whenever the lowest cloud base is below 1500 ft above ground Ref:
all Ans: B 24400. When fog is reported, the visibility is below: A – 0.8 km B – 1
km C – 1.5 km D – 3 km Ref: all Ans: B 24402. When visibility is reduced by water
droplets to less than 1000 metres it is classified as: A – fog B – dust fog C –
haze D – mist Ref: all Ans: A
050-10

METEOROLOGICAL INFORMATION
050-10-01 Observation

9660. What are the images of satellites provided daily by the Weather Service used
for? A – To locate fronts in areas with few observation stations B – To measure
wind currents on the ground C – To help provide 14-day forecasts D – To locate
precipitation zones Ref: all Ans: A 9663. The wind indicator for a weather
observation receives the measured value from an anemometer. Where is this
instrument placed? A – On the roof of the weather station B – 1m above the runway C
– Close to the station about 2m above the ground D – On a mast 6-10m above the
runway Ref: all Ans: D 9685. While approaching your target aerodrome you receive
the following message: RVR runway 23: 400m This information indicates the: A –
portion of runway which a pilot on the threshold of any of the runways would see,
with runway 23 in service B – meteorological visibility on runway 23 C – length of
runway which a pilot in an aircraft on the ground would see, on the threshold of
runway 23 D – minimum visibility at this aerodrome, with runway 23 being the one in
service Ref: all Ans: C
9710. What is the relationship between meteorological visibility (met.vis) and RVR
in homogeneous fog? A – The met. vis. generally is the same as the RVR B – The met.
vis. generally is greater than the RVR C – The met. vis. is generally less than the
RVR D – There is no specific relationship between the two Ref: all Ans: C 9715.
Which of the following causes echoes on meteorological radar screens? A – Hail B –
Water vapour C – Fog D – Any cloud Ref: all Ans: A 10464. When is the RVR reported
at most airports? A – When the RVR decreases below 2000m B – When the
meteorological visibility decreases below 800m C – When the RVR decreases below
800m D – When the meteorological visibility decreases below 1500m Ref: all Ans: D
10493. An airborne weather radar installation makes it possible to detect the
location of: A – cumulonimbus, but provided that cloud of this type is accompanied
by falls of hail B – all clouds C – zones of precipitation, particularly liquid
state precipitation, and also their intensity D – stratocumulus and its vertical
development Ref: all Ans: C
10558. Runway Visual Range (RVR) is: A – measured with ceilometers alongside the
runway B – reported when meteorological visibility is less than 2000m C – reported
in TAF and METAR D – usually better than meteorological visibility Ref: all Ans: D
15796. Which of the following meteorological phenomena can rapidly change the
braking action of a runway? A – MIFG B – FG C – HZ D - +FZRA Ref: all Ans: D 15818.
What is a SPECI? A – An aerodrome forecast issued every 9 hours B – A selected
special aerodrome weather report, issued when a significant change of the weather
conditions have been observed C – A routine aerodrome weather report issued every 3
hours D – A warning of meteorological dangers at an aerodrome, issued only when
required Ref: all Ans: B 15831. (Refer to figure 050-85) Looking at the chart, at
what altitude above Frankfurt would you expect the tropopause to be located? A – FL
410 B – FL 360 C – FL 330 D – FL 390 Ref: all Ans: B
15847. (Refer to figure 050-82) Look at the chart. Assuming a normal vertical
temperature gradient, at what altitude will the freezinglevel above Shannon be
found? A – FL 20 B – FL 140 C – FL 120 D – FL 60 Ref: all Ans: D 16548. The meaning
of RVR is: A – cross-wind component B – runway visual range C – meteorological
visibility D – braking action Ref: all Ans: B 16561. Normally, temperature readings
are taken at a height of ___ in a Stevenson Screen: A – 4 ft (1.25 m) B – 4m C –
10m D – 40cm Ref: all Ans: A 16562. A cup ___ and remote transmitting vane form the
transmitting head of the electrical ___ which enables a continuous record of wind
direction and speed to be made on a moving chart. A – anemograph; anemometer B –
anemometer; barograph C – anemometer; anemograph D – barograph; anemometer Ref: all
Ans: C
16564. The three elements in a cloud report are: A – amount, cover, height of base
B – type, cover, amount C – amount, type, height of base D – type, amount, cloud
species Ref: all Ans: C 16565. Barometric tendency is measured on ___ and recorded
on ___ A – an anemometer; an aneroid barometer B – an anemograph; an anemometer C –
barograph; aneroid barometer D – aneroid barometer; barograph Ref: all Ans: D
16569. LFCG 221100Z 1219 22010KT 4500 RA BKN 010 OVC015 TEMPO 1500 + RA OVC005
FM1430 29020G35KT 6000 SHRA BKN008TCU OVC015 TEMPO 1619 25010 KT 9999 NSW BKN020
PROB30 TEMPO 1619 1500 TSGR BKN007CB What is the most likely visibility for landing
at 1345Z? A – 4,500m B – 1,500m C – 6,000m D – 12,000m Ref: all Ans: A
16570. LFCG 221100Z 1219 22010KT 4500 RA BKN010 OVC015 TEMPO 1200 + RA OVC005
FM1430 29020G35KT 6000 SHRA BKN008TCU OVC015 TEMPO 1619 25010 KT 9999 NSW BKN020
PROB30 TEMPO 1619 1500 TSGR BKN007CB What is the lowest cloud base you may
encounter at 1345Z? A – 1,000 ft B – 500 ft C – 700 ft D – 12,000 ft Ref: all Ans:
B 24159. (Refer to figure 050-33) For 1300 UTC select a METAR which you consider to
be most appropriate to position “T”: A – 19010KT 6000 RA BKN016 OVC090 08/06 Q1004=
B – 24020KT 5000 RA BKN100 11/10 Q1002= C – 18015KT 9999 SCT020 03/01 Q1000= D –
27030KT 8000 SCT020 07/03 Q1004= Ref: all Ans: A 24183. (Refer to figure 050-106)
Which of the following METAR’s reflect the weather conditions expected at Brussels
airport (EBBR) at 1200 UTC? A – 21014KT 5000 RA BKN015 OVC030 15/12 Q1011 NOSIG= B
– 28010KT 9999 BKN030 22/11 Q1016 NOSIG= C – VRB03KT 8000 BKN280 19/12 Q1012 BECMG
3000 BR= D – 19022G46KT 1200+TSGR BKN003 BKN015 17/14 Q1014 BECMG NSW= Ref: all
Ans: A
24192. (Refer to figure 050-05) Of the four radio surroundings, select the one that
indicates low stratus: A–3 B–1 C–2 D–4 Ref: all Ans: A 24214. A METAR message is
valid A – at the time of observation B – for 2 hours C – for the hour following the
observation D – for 9 hours Ref: all Ans: A 24277. How is a ceiling defined? A –
Height above ground or water of the highest layer of cloud or obscuring phenomena
aloft that covers 4 oktas or more of the sky B – Height above ground or water of
the lowest layer of cloud independent on the amount of clouds C – Height above
ground or water of the lowest layer of cloud that contributes to the overall
overcast D – Height above ground or water of the lowest layer of cloud below 20000
ft covering more than half of the sky Ref: all Ans: D 24281. In a METAR message, BR
and HZ mean respectively: A – BR = mist HZ = widespread dust B – BR = fog HZ = haze
C – BR = mist HZ = smoke D – BR = mist HZ = haze Ref: all Ans: D
24282. In a METAR message, the wind group is 23010 MPS. This means: A – Wind from
230o magnetic at 10 miles per hour B – Wind from 230o true at 10 miles per hour C –
Wind from 230o magnetic at 20 knots D – Wind from 230o true at 20 knots Ref: all
Ans: D 24292. In the METAR code the abbreviation VC indicates: A – volcanic ash B –
present weather within the approach area C – present weather within a range of 8
km, but not at the airport D – present weather at the airport Ref: all Ans: C
24293. In the METAR code the abbreviation VCBLDU means: A – an active dust storm B
– blowing dust in the vicinity C – an active sandstorm D – an active dust and sand
storm Ref: all Ans: B 24352. The temperature at the surface is given as +15oC and
at 4000 ft it is +9oC the state of this layer is said to be: A – absolutely
unstable B – conditionally unstable C – stable D – unstable Ref: all Ans: C
24356. The visibility transmitted in a METAR is: A – given by transmissometer when
the visibility is below 1500 metres B – the maximum determined by a meteorologist
in a 360o scan at the horizon C – the lowest observed in a 360o scan from the
meteorological station D – measured by an observer counting the number of lights
visible on the runway Ref: all Ans: C 24362. What does the code R24R/P1500 mean? A
– Snow clearance in progress on RWY 24 right, useable runway length 1500 metres B –
RVR RWY 24 right below 1500 metres C – RVR RWY 24 right above 1500 metres D – RVR
RWY 24 right 1500 metres Ref: all Ans: C 24374. What is the best approximation for
the wind speed at flight level 250? A – By simple interpolation of wind information
available from the 500 and 300 hPa charts B – By interpolation of the wind
information available from the 500 and 300 hPa charts, while also considering the
maximum wind information found on the Significant Weather Chart C – By reading wind
direction and speed from the 500 hPa chart D – By reading wind direction and speed
from the 300 hPa chart Ref: all Ans: B
24378. What is the lowest cloud base that can be expected from the forecast for
1500 UTC? KCHS 280430Z 280606 VRB05KT 4000 BR SCT005 OVC013 PROB40 SHRA BECMG 1314
9000 SHRA OVC015 PROB40 TEMPO 1416 VRB15G25KT 1600 TSRA OVC010CB BECMG 1618 26010KT
BKN030 BECMG A – 3000 feet B – 1000 feet C – 1300 feet D – 1500 feet Ref: all Ans:
B 24379. What is the lowest possible cloud base forecast for 2300 UTC? EDDF 272200Z
280624 VRB05KT 4000 BR SCT005 OVC013 BECMG 1314 9000 SHRA OVC015 PROB40 TEMPO 1416
VRB15G25KT 1600 TSRA OVC010CB BECMG 1618 26010KT BKN030 BECMG 2122 CAVOK A – 5000
feet B – 3000 feet C – 500 feet D – 1500 feet Ref: all Ans: A 24381. What is the
lowest visibility forecast at ETA Zurich 1430 UTC? LSZH 260900Z 261019 20018G30KT
9999 –RA SCT050 BKN080 TEMPO 1215 23012KT 6000 –DZ BKN015 BKN030 BECMG 1518
23020G35KT 4000 RA OVC010= A – 10 km B – 6 NM C – 4 km D – 6 km Ref: all Ans: D
24394. What visibility is most likely to be experienced at 1400 UTC? EDDF 272200Z
280624 VRB05KT 4000 BR SCT005 OVC013 BECMG 1314 9000 SHRA OVC015 PROB40 TEMPO 1416
VRB15G25KT 1600 TSRA OVC010CB BECMG 1618 26010KT BKN030 BECMG 2122 CAVOK A – 1600
metres B – 4000 metres C – 1000 metres D – 9000 metres Ref: all Ans: D 24413. Which
of the following extracts of weather reports could be, in accordance with the
regulations, abbreviated to “CAVOK”? (MSA minus airport elevation equals: LSZB
10000 ft, LSZH 8000 ft, LSGG 12000 ft, LFSB 6000 ft) A – LSZH 26024G52KT 9999
BKN060 17/14 Q1012 RETS TEMPO 5000 TSRA= B – LSGG 22003KT 9999 SCT120 KBN280 09/08
Q1026 BECMG 5000 BR= C – LSZB 30004KT 9999 SCT090 10/09 Q1006 NOSIG= D – LFSB
00000KT 9000 SCT080 22/15 Q1022 NOSIG= Ref: all Ans: B 24427. Which of the
following statements is an interpretation of the SIGMET? LSAS SIGMET 2 VALID
030700/031100 LSZH- SWITZERLAND FIR/UIR MOD TO SEV CAT FCST N OF ALPS BTN FL
260/380 STNR INTSF A – Moderate to severe clear air turbulence to be expected north
of the Alps. Intensity increasing. Danger zone between FL 260 and FL 380 B – Zone
of moderate to severe turbulence moving towards the area north of the Alps.
Intensity increasing. Pilots advised to cross this area above FL 260 C – Severe
turbulence observed below FL 260 north of the Alps. Pilots advised to cross this
area above FL 380 D – Moderate to severe clear air turbulence of constant intensity
to be expected north of the Alps Ref: all Ans: A
24435. Which of the following weather reports could be, in accordance with the
regulations, abbreviated to “CAVOK”? A – 27019G37KT 9999 BKN050 18/14 Q1016 NOSIG=
B – 34004KT 7000 MIFG SCT260 09/08 Q1029 BECMG 1600= C – 00000KT 0100 FG VV001
11/11 Q1025 BECMG 0500= D – 26012KT 8000 SHRA BKN025 16/12 Q1018 NOSIG= Ref: all
Ans: A 24452. While appreciating your destination aerodrome you receive the
following message: RVR runway 23: 400m This information indicates the A – portion
of runway which a pilot on the threshold of any of the runways would see, with
runway 23 in service B – visibility on runway 23 C – length of runway which a pilot
in an aircraft on the ground would see, on the threshold of runway 23 D – minimum
visibility at this aerodrome, with runway 23 being the one in service Ref: all Ans:
C 24453. Why can the following METAR not be abbreviated to CAVOK? DLLO 121550Z
31018G30KT 9999 FEW060TCU BKN070 14/08 Q1016 TEMPO 4000 TS= A – Because the cloud
base in below the highest minimum sector altitude B – Because there are gusts
reported C – Because towering cumulus are observed D – Because thunderstorms are
forecast temporary for the next 2 hours Ref: all Ans: A
25596. (Refer to figure 050-04) Of the four radio soundings, select the one that
indicates ground fog: A–1 B–2 C–3 D–4 Ref: AIR: atpl, ir; HELI: all Ans: B 25634.
The radiosonde can directly measure: A – below 5 kt B – between 10 and 15 kt C –
between 5 and 10 kt D – above 15 kt Ref: AIR: atpl, ir; HELI: all Ans: A

050-10-02 Weather charts


24119. (Refer to figure 050-57) The weather most likely to be experienced at
position “A” is: A – cumulus, cumulonimbus clouds, heavy rain or snow showers,
medium to strong winds B – clear skies, good visibility in light winds C – mainly
overcast with stratus or stratocumulus and drizzle, medium to strong winds D –
radiation fog, low stratus, drizzle, no medium or upper cloud, light wind Ref: all
Ans: C 2847. In what hPa range is an upper weather chart for FL 340 situated? A –
500 – 400 hPa B – 600 – 500 hPa C – 300 – 200 hPa D – 400 – 300 hPa Ref: all Ans: C
8989. (Refer to figure 050-102) On which route of flight would you expect no icing
at FL 180? A – Zurich-Hamburg B – Zurich-Madrid C – Hamburg-Stockholm D – Zurich-
Vienna Ref: all Ans: C 9640. (Refer to figure 050-98) The cold front is indicated
at position: A–A B–B C–C D–D Ref: all Ans: B 9644. (Refer to figure 050-98) The
warm sector is indicated by letter: A–A B–B C–D D–C Ref: all Ans: D 9649. When
planning a flight at FL 110, which upper wind and temperature chart would be
nearest you rflight level? A – 300 hPa B – 850 hPa C – 700 hPa D – 500 hPa Ref: all
Ans: C
9658. (Refer to figure 050-45) Which of the following best describes Zone B? A –
Col B – Ridge of high pressure C – Depression D – Trough of low pressure Ref: all
Ans: A 9680. If you are planning a flight at FL 290, which of these upper wind and
temperature charts would be nearest your flight level? A – 850 hPa B – 500 hPa C –
700 hPa D – 300 hPa Ref: all Ans: D 9703. What positions are connected with contour
lines on the weather chart? A – Positions with the same air density B – Positions
with the same thickness between two constant pressure levels C – Positions with the
same height in a chart of constant pressure D – Positions with the same wind
velocity Ref: all Ans: C 9713. (Refer to figure 050-105_A) According to ICAO, which
symbol indicates severe icing? A–1 B–4 C–5 D–6 Ref: all Ans: A
9723. (Refer to figure 050-45) Which of the following best describes Zone C? A –
Ridge of high pressure B – Col C – Trough of low pressure D – Depression Ref: all
Ans: A 10449. Which constant pressure altitude chart is standard for a FL 300? A –
500 hPa B – 200 hPa C – 700 hPa D – 300 hPa Ref: all Ans: D 10456. If you are
planning a flight at FL 170, which of these upper wind and temperature charts would
be nearest your flight level? A – 300 hPa B – 500 hPa C – 850 hPa D – 700 hPa Ref:
all Ans: B 10472. On which of the following aviation weather charts can a pilot
most easily find a jet stream? A – Upper air chart B – Wind/temperature chart C –
Surface chart D – Significant weather chart Ref: all Ans: D
10476. (Refer to figure 050-06) What does the symbol indicate on a significant
weather chart? A – The centre of a tropopause high, where the tropopause is at FL
400 B – The centre of a high pressure area at 400 hPa C – The upper limit of
significant weather at FL 400 D – The lower limit of the tropopause Ref: all Ans: A
10498. (Refer to figure 050-105_A) According to ICAO, which symbol indicates a
tropical revoving storm? A–2 B–7 C – 10 D–6 Ref: all Ans: A 10503. (Refer to figure
050-45) Which of the following best describes Zone D? A – Anti-cyclone B – Ridge of
high pressure C – Depression D – Trough of low pressure Ref: all Ans: C 10504.
Which constant pressure altitude chart is standard for a FL 180? A – 200 hPa B –
300 hPa C – 500 hPa D – 700 hPa Ref: all Ans: C
10514. Which constant pressure altitude chart is standard for a FL 390? A – 700 hPa
B – 300 hPa C – 500 hPa D – 200 hPa Ref: all Ans: D 10524. Which weather chart
gives information about icing and the height of the freezing level? A – Surface
chart B – 500 hPa chart C – 700 hPa chart D – Significant weather chart Ref: all
Ans: D 10532. Which constant pressure altitude chart is standard for a FL 100? A –
850 hPa B – 700 hPa C – 500 hPa D – 300 hPa Ref: all Ans: B 10535. How are well
separated CB clouds described on the Significant Weather Chart? A – ISOL CB B –
EMBD CB C – FREQ CB D – OCNL CB Ref: all Ans: D
10538. In which meteorological forecast chart is information about CAT regions
found? A – 300 hPa chart B – 24 hour surface forecast C – 500 hPa chart D –
Significant Weather Chart Ref: all Ans: D 10543. What information is given on a
Significant Weather Chart? A – The significant weather in a period 3 hours before
and 3 hours after the time given on the chart B – The significant weather that is
observed at the time given on the chart C – The significant weather forecast for
the time given on the chart D – The significant weather forecast for a period 6
hours after the time given on the chart Ref: all Ans: C 10544. How is the direction
and speed of upper winds described in forecasts? A – The direction is relative to
magnetic north and the speed is in miles per hour B – The direction is relative to
magnetic north and the speed is in knots C – The direction is relative to true
north and the speed is in knots D – The direction is relative to true north and the
speed is in miles per hour Ref: all Ans: C 15788. Which constant pressure altitude
chart is standard for a FL 50? A – 850 hPa B – 700 hPa C – 500 hPa D – 300 hPa Ref:
all Ans: A
15799. (Refer to figure 2050-71) You are flying from Munich to Amsterdam. Which of
the following flight levels would you choose in order to avoid turbulence and
icing? A – FL 140 B – FL 260 C – FL 320 D – FL 300 Ref: all Ans: B 15802. (Refer to
figure 050-100) What is the average wind at FL 160 between Zurich and Rome? A –
020/50 B – 050/40 C – 200/40 D – 350/40 Ref: all Ans: C 15834. (Refer to figure
050-44) The attached chart shows the weather conditions on the ground at 1200 UTC
on October 10. Which of the following reports reflects weather development at
Zurich airport? A – TAF LSZH 101601 05020G35KT 8000 BKN01S TEMPO 1720 05018KT 0300
+SHSN W002= B – TAF LSZH 101601 23012KT 6000 RA BKN012 OVC030 TEMPO 2023 22025G40KT
1600 +SNRA BKN003 OVC015 C – TAF LSZH 101601 32008KT 9999 SCT030TCU TEMPO 2201
32020G32KT 3000 TSRA BKN020CB= D – TAF LSZH 101601 VRB02KT 8000 SCT280 BECMG 1618
00000KT 3500 MIFG BECMG 1820 1500 BCFG BECMG 2022 0100 FG W001= Ref: all Ans: D
15835. The upper wind and temperatue chart of 250 hPa corresponds, in a standard
atmosphere, to about: A – 39,000 ft B – 30,000 ft C – 32,000 ft D – 34,000 ft Ref:
all Ans: D 15842. When planning a flight at FL 60, which upper wind and temperature
chart would be nearest your flight level? A – 500 hPa B – 300 hPa C – 850 hPa D –
700 hPa Ref: all Ans: C 15846. (Refer to figure 050-82) Select from the map the
average wind for the route Zurich-Rome at FL 110: A – 230/10 B – 200/30 C – 040/10
D – 250/20 Ref: all Ans: A 16356. The 500 mb contour chart is equivalent to Flight
Level: A – FL 300 B – FL 90 C – FL 50 D – FL 180 Ref: all Ans: D
16362. The lines on a contour chart join points at: A – equal pressure B – equal
temperature C – equal height D – equal humidity Ref: all Ans: C 24100. (Refer to
figure 050-59) Considering the North Atlantic route from the Azores to Bermuda, the
mean height of the tropical tropopause during summer is approximately: A – 33,000
ft B – 39,000 ft C – 51,000 ft D – 60,000 ft Ref: all Ans: C 24105. (Refer to
figure 050-61) Considering the sector Rom to 25oN on the route indicated, what mean
upper winds may be expected at FL 300 during winter? A – Easterlies at 40 kt B –
Variable easterlies C – Light westerlies D – Westerly jet streams in excess of 70
kt Ref: all Ans: D 24107. (Refer to figure 050-51) The air mass affecting positon
“A” is most likely to be: A – Continental tropical B – Maritime polar C –
Continental polar D – Maritime tropical Ref: all Ans: B
24108. (Refer to figure 050-49) The air mass affecting position “C” is most likely
to be: A – Continental tropical B – Maritime tropical C – Maritime polar D –
Continental polar Ref: all Ans: B 24109. (Refer to figure 050-50) The air mass
affecting position “S” is most likely to be: A – Maritime polar and unstable B –
Maritime tropical and stable C – Maritime polar and stable D – Continental tropical
and unstable Ref: all Ans: B 24110. (Refer to figure 050-62) The air mass affecting
position “P” is most likely to be: A – Maritime tropical B – Maritime polar C –
Continental tropical D – Continental polar Ref: all Ans: D 24111. (Refer to figure
050-63) The air mass affecting position “R” is most likely to be: A – Maritime
tropical B – Maritime polar C – Continental polar D – Continental tropical Ref: all
Ans: D
24112. (Refer to figure 050-64) The weather most likely to be experienced at
position “S” is: A – little or no cloud – hazy B – morning fog liftig to low
stratus C – OVC SC D – frequent heavy showers of rain or snow Ref: all Ans: D
24113. (Refer to figure 050-65) The weather most likely to be experienced at
position “A” is: A – advection fog B – clear skies – radiation fog forming
overnight C – snow showers – gale force wind D – continuous drizzle and hill fog
Ref: all Ans: C 24117. (Refer to figure 050-34) The occurrence of freezing rain at
FL 60 is most likely in square: A – 2D B – 2A C – 3B D – 3C Ref: all Ans: D 24120.
(Refer to figue 050-50) The weather most likely to be experienced at position “S”
is: A – increasing AS and NS with moderate to heavy rain B – scattered
thunderstorms and good visibility C – overcast with drizzle and orographic fog D –
small amounts of CU with generally good visibility Ref: all Ans: C
24122. (Refer to figure 050-27) What flight conditions are most likely to be
experienced in square 2B by an aircraft at FL 120? A – IMC in NS with risk of light
icing B – VMC below an overcast of AS and CS, generally smooth air C – IMC in
layers of AS and isolated CB risk of severe turbulence and icing D – VMC above
layers of ST and SC, generally stable conditions Ref: all Ans: D 24123. (Refer to
figure 050-34) For an aircraft making an approach to an airfield located in square
3B, the most likely weather conditions are: A – showers of rain and hail B –
moderate continuous rain C – low cloud, mist D – scattered AC base 2000 ft, good
visibility Ref: all Ans: C 24124. (Refer to figure 050-34) For an aircraft making
an approach to an airfield located in square 3B, away from the vicinity of the
fronts, the most likely weather conditions in winter are: A – generally overcast,
moderate continuous rain and risk of low level wind shear B – scattered SC and CU,
good visibility C – poor visibility in mist and drizzle D – prolonged periods of
heavy rain and hail Ref: all Ans: C
24125. (Refer to figure 050-51) The weather most likely to be experienced near to
position “A” is: A – overcast layer cloud – rain later B – advection fog and
drizzle C – frequent showers of rain and snow, good visibility outside
precipitation D – clear skies – radiation fog at night Ref: all Ans: C 24126.
(Refer to figure 050-52) The weather most likely to be experienced at position “B”
is: A – early morning fog lifting to low stratus later B – advection fog and
drizzle C – poor visibility in anty-cyclonic circulation D – frequent showers of
rain or snow Ref: all Ans: D 24127. (Refer to figure 050-53) The weather most
likely to be experienced at position “B” is: A – mainly overcast with stratus or
stratocumulus, drizzle B – scattered stratocumulus with good visibility C –
frequent showers of rain or snow, good visibility outside showers D – clear skies,
moderate wind, good visibility Ref: all Ans: C 24128. (Refer to figure 050-54) The
weather most likely to be experienced on the coast near position “Q” is: A – mainly
clear skies with fog developing overnight B – mainly overcast with poor visibility
C – thundery showers particularly at night D – showery with generally good
visibility Ref: all Ans: D
24132. (Refer to figure 050-55) The pressure system at position “D” is a: A –
secondary low B – col C – trough of low pressure D – ridge of high pressure Ref:
all Ans: B 24136. (Refer to figure 050-18) The dotted line designated “Z”
represents the: A – mean position of the inter-tropical convergence zone (ITCZ)
during July B – mean position of the inter-tropical convegency zone (ITCZ) during
January C – northerly limit of the sub-tropical jet stream during July D –
northerly limit of the SE trade winds during January Ref: all Ans: A 24137. (Refer
to figure 050-18) The dotted line labelled “Y” represents the: A – axis of the sub-
tropical jet stream during January B – mean position of the inter-tropical
convergence zone (ITCZ) during January C – mean position of the inter-tropical
convergence zone (ITCZ) during July D – axis of the equatorial jet stream during
July Ref: all Ans: B 24138. (Refer to figure 050-22) The weather most likely to be
experienced at position “R” is: A – overcast with drizzle and hill fog B – early
morning fog lifting to low stratus C – increasing amounts of AS and NS – heavy rain
D – fine and warm at first – AC castellanus and CB in late afternoon with
thunderstorms Ref: all Ans: D
24140. (Refer to figure 050-58) For an aircraft making an approach to an airfield
which is not situation near high ground, in the vicinity of the active front in
square 3B, a potential hazard exists in the form of: A – radiation fog B – low
level wind shear C – rotor streaming D – clear air turbulence (CAT) Ref: all Ans: B
24141. (Refer to figure 050-23) The front at the bottom of the diagram, south of
position C, is: A – an occlusion on the surface B – a warm front C – an occlusion
above the surface D – a cold front Ref: all Ans: A 24142. (Refer to figure 050-24)
The air mass at position “X” is most likely to be: A – Maritime polar B – Maritime
tropical C – Continental polar D – Continental tropical Ref: all Ans: B 24143.
(Refer to figure 050-17) The air mass type indicated by arrow number 4 is
designated: A – Continental polar B – Maritime tropical C – Maritime polar D –
Continental tropical Ref: all Ans: B
24144. (Refer to figure 050-25) The air mass affecting position “R” is most likely
to be: A – Maritime tropical B – Continental polar C – Continental tropical D –
Maritime polar Ref: all Ans: D 24145. (Refer to figure 050-27) For an aircraft at
FL 80, ahead of the front in square 2D, the expected flight conditions are: A – IMC
in cumuliform cloud, moderate turbulence with a risk of rime icing B – overcast
skies, moderate to heavy turbulence with the possibility of thunderstorms C – below
AS type cloud, generally smooth air with light precipitation D – high C1 and CS
type cloud, light turbulence and poor visibility Ref: all Ans: C 24146. (Refer to
figure 050-27) For an aircraft at FL 40, approaching the front (square 3C) from the
direction of square 3D a potential hazard exists before reaching the front in the
form of: A – clear ice accretion to the airframe B – severe turbulence associated
with CB C – severe turbulence and wind shear D – severe down drafts from subsiding
air Ref: all Ans: A
24147. (Refer to figure 050-27) What conditions are most likely to prevail at an
aerodrome located in square 3B? A – Mainly overcast at 8000 ft, visibility less
than 5 km in continuous moderate rain B – Broken CU base 2000 ft, visibility more
than 5 km, occasional showers of rain or snow C – 6-8 oktas SC and ST, visibility
moderate to poor in drizzle D – Intermittent thunderstorms otherwise generally
clear skies with good visibility Ref: all Ans: C 24148. (Refer to figure 050-26)
When front “G” passes position “T” the surface wind should: A – veer and decrease B
– veer and increase C – back and increase D – back and decrease Ref: all Ans: B
24149. (Refer to figure 050-66) The pressure distribution located mainly in square
2A is a: A – trough of low pressure B – col C – ridge of high pressure D –
depression Ref: all Ans: C
24151. (Refer to figure 050-29) From indications shown on the chart, when front “S”
passes position “V” the urface wind should: A – back and decrease B – veer and
increase C – back and remain more or less at the same speed D – veer and remain
more or less at the same speed Ref: all Ans: D 24152. (Refer to figure 050-29) By
the time the front “Z” has passed point “Q” the surface wind will have: A – veered
and increased B – veered and decreased C – backed and increased D – backed and
decreased Ref: all Ans: A 24153. (Refer to figure 050-93) The temperature deviation
from ISA (to the nearest oC) overhead Charleston t FL 340 is: A - +3 B - +5 C - -5
D - -7 Ref: all Ans: B 24154. (Refer to figure 050-83) At 40oN 70oW, the forecast
wind is: A – 270o/95 kt B – 320o/40 kt C – 240o/90 kt D – 280o/70 kt Ref: all Ans:
A
24155. (Refer to figure 050-85) At 40oN 20oW, the forecast wind at FL 390 is: A –
090o/45 kt B – 060o/45 kt C – 070o/30 kt D – 270o/45 kt Ref: all Ans: A 24156.
(Refer to figure 050-63) The front located from 10oW to 10oE is most likely to be:
A – a quasi-stationary front B – an active warm front moving north C – an active
occlusion moving south D – a cold front moving south Ref: all Ans: A 24157. (Refer
to figure 050-32) The front labelled “E” is a: A – cold front B – warm front C –
warm occlusion D – cold occlusion Ref: all Ans: A 24158. (Refer to figure 050-93)
Considering the route between Valencia and Charleston at FL 340, the forecast mean
temperature is: A - -45oC B - -50oC C - -55oC D - -40oC Ref: all Ans: B
24160. (Refer to figure 050-92) What is the average wind forecast for FL 300
between Edinburgh and Madrid? A – 180/20 B – 300/45 C – 240/25 D – 280/30 Ref: all
Ans: D 24161. (Refer to figure 050-92) What is the average temperature difference
from ISA at FL 300 between Edinburgh and Madrid? A - +2oC B - +12oC C - -2oC D -
-12oC Ref: all Ans: C 24162. (Refer to figure 050-94) The average forecast wind for
the leg from Madrid to Dhahran at FL 390 is: A – 270/50 B – 270/30 C – 320/70 D –
310/50 Ref: all Ans: A 24163. (Refer to figure 050-94) What is the average
temperature difference from ISA at FL 390 between Madrid and Dhahran? A - -1oC B -
+5oC C - -5oC D - +2oC Ref: all Ans: A
24164. (Refer to figure 050-95) Flight from Lisbon (LPPT) to Kingston (MKJP). At
40oN 20oW, the temperature deviation from ISA at FL 390 is: A – 0oC B - +6oC C -
+2oC D - -2oC Ref: all Ans: A 24165. (Refer to figure 050-95) Flight from Lisbon
(LPPT) to Kingston (MKJP). Considering the route segment between 60oW and 70oW, at
FL 390 the forecast mean temperature is: A – -52oC B - -55oC C - -58oC D - -61oC
Ref: all Ans: B 24166. (Refer to figure 050-96) The mean wind that may be expected
to affect the route segment from the coast of SE England to Geneva at FL 270 is: A
– 245/55 B – 240/90 C – 270/70 D – 220/70 Ref: all Ans: D
24167. (Refer to figure 050-96) The mean temperature that may be expected to affect
that segment of the route from the coast of SE England to Geneva at FL 270 is: A -
-34oC B - -38oC C - -30oC D - -42oC Ref: all Ans: B 24168. (Refer to figure 050-96)
At FL 300 between Geneva and Tunis, what mean wind would be most likely? A – 245/50
B – 225/25 C – 265/40 D – 265/25 Ref: all Ans: A 24169. (Refer to figure 050-114)
In appendix are shown four sections of the 700 hPa wind chart. The diagram
representing most accurately the wind direction and speed is: A – 210o/30 kt
(diagram B) B – 030o/30 kt (diagram A) C – 030o/30 kt (diagram C) D – 210o/30 kt
(diagram D) Ref: all Ans: B 24170. (Refer to figure 050-30) According to the
extract of the surface isobar map, the surface wind direction over the sea is
approximately: A – 140o B – 110o C – 310o D – 220o Ref: all Ans: A
24172. (Refer to figure 050-31) On a Significant Weather Chart, hail is represented
by symbol: A–1 B–4 C–2 D–3 Ref: all Ans: B 24174. (Refer to figure 050-35) What is
the classification of the air mass affecting position “Q”? A – Continental tropical
B – Maritime polar C – Continental polar D – Maritime tropical Ref: all Ans: D
24175. (Refer to figure 050-105_B) Which of the following symbols represents a
squall line? A – Symbol 5 B – Symbol 3 C – Symbol 4 D – Symbol 2 Ref: all Ans: A
24178. (Refer to figure 050-71) What is the approximate height of the tropopause
between Keflavik and Helsinki? A – FL 360 B – FL 350 C – FL 300 D – FL 320 Ref: all
Ans: D
24179. (Refer to figure 050-72) At what flight level is the jet stream core that is
situated over northern Scandinavia? A – FL 330 B – FL 280 C – FL 360 D – FL 300
Ref: all Ans: B 24181. (Reer to figure 050-73) At what approximate flight level is
the tropopause over Frankfurt? A – FL 350 B – FL 300 C – FL 330 D – FL 240 Ref: all
Ans: C 24182. (Refer to figure 050-69) The temperature at FL 330 overhead London
will be: A - -57oC B - -39oC C - -33oC D - -45oC Ref: all Ans: D 24185. (Refer to
figure 050-76) What is the average temperature at FL 160 between Oslo and Paris? A
- -25oC B - -23oC C - -15oC D - -19oC Ref: all Ans: D
24186. (Refer to figure 050-78) What is the speed of the front located over France?
A – 15 kt B – 25 kt C – 10 kt D – 30 kt Ref: all Ans: A 24191. (Refer to figure
050-111) Continuous freezing rain is observed at an airfield. Which of the four
diagrams is most likely to reflect temperatures above the airfield concerned? A –
Diagram 3 B – Diagram 4 C – Diagram 2 D – Diagram 1 Ref: all Ans: B 24193. (Refer
to figure 050-98) A convergence line is indicated by: A–C B–B C–A D–D Ref: all Ans:
C 24194. (Refer to figure 050-112) According to ICAO, which symbol indicates danger
to an aircraft flying according to instrument flight rules (IFR)? A – Symbol 1 B –
Symbol 4 C – Symbol 2 D – Symbol 3 Ref: all Ans: C
24195. (Refer to figure 050-105_A) According to ICAO, which symbol indicates danger
to an aircraft flying according to instrument flight rules (IFR)? A – Symbol 6 B –
Symbol 9 C – Symbol 3 D – Symbol 7 Ref: all Ans: C 24197. (Refer to figure 050-100)
Select from the map the average wind for the route Frankfurt – Rome at FL 170: A –
230/40 B – 200/50 C – 050/40 D – 030/35 Ref: all Ans: A 24198. (Refer to figure
050-86) If you are flying from Zurich to London at FL 240, what conditions can you
expect at cruising altitude? A – Flight largely in cloud; no turbulence B –
Moderate or severe turbulence and icing C – Prolonged severe turbulence and icing
throughout the flight D – CAT for the first half of the flight Ref: all Ans: B
24199. (Refer to figure 050-87) Judging by the chart, what wind speeds can you
expect at FL 310 above London? A – 90 kt B – 300 kt C – 140 kt D – 100 km/h Ref:
all Ans: A 24200. (Refer to figure 050-90) Which of these statements is true? A –
Turbulence is likely to be encountered at FL 400 over Malaga (LEMG) B – Freezing
level above London (EGLL) is higher than FL 065 C – The front to the east of Paris
(LEPO) is moving south D – Local snow and severe aircraft icing can be expected
over Germany Ref: all Ans: D 24201. (Refer to figure 050-91) On which of these
routes would you not need to worry about icing at FL 170? A – Paris – Lisbon B –
Madrid – Vienna C – London – Stockholm D – Zurich – Athens Ref: all Ans: C
24202. (Refer to figure 050-47) This chart shows the weather situation at 0600 UTC
on May 23. Which of the ollowing reports reflects weather development at Geneva
Airport? A – TAF LSGG 230600Z 230716 23016KT 8000 –RA BKN030 OVC070 BECMG 0810 5000
RA BKN020 OVC050 TEMPO 1012 3000 +RA BKN010 OVC030 BECMG 1215 25014KT 8000 SCT030
BKN090= B – TAF LSGG 230600Z 230716 05014KT 5000 BR OVC015 BECMG 0810 8000 BKN018
BECMG 1013 05015G30KT 9999 SCT025= C – TAF LSGG 230600Z 230716 26012KT 9999 SCT030
BKN080 TEMPO 1013 25020G35KT 3000 TSRA BKN030CB BECMG 1316 VRB02KT 3000 BCFG
SCT100= D – TAF LSGG 230600Z 230716 VRB03KT 6000 SCT020 BECMG 0811 23005KT 9999
SCT025TCU PROB 40 TEMPO 1216 34012G30KT 3000 TSRA BKN020CB= Ref: all Ans: A 24203.
(Refer to figure 050-87) Which of the following statements is true? A – The
moderate icing conditions overhead Zurich are forecast in a layer between FL 140
and above the limit of the chart B – The wind speed in the core of the jet stream
near Bordeaux (B) is about 90 kt C – The west part of the front system over Spain
moves to the south with 10 kt and the eastern part of that front over Spain moves
to the north D – On a flight from Copenhagen to Amsterdam you have not to worry
about moderate and/or severe turbulence at FL 100 Ref: all Ans: B 24205. (Refer to
figue 050-85) If you are flying from Zurich to London at FL 130 between 0600 and
0900 UTC, what can you expect at cruising level? A – You will be out of clouds more
of three quarters of your flight B – A strong wind from the right with 90 kt at the
crossing of the core of the jet stream C – You will arrive at your destination
before you have to cross the warm front D – Moderate icing about half way along the
route Ref: all Ans: D
24308. On an upper wind and temperature chart: A – the wind increases from south to
north B – the temperature is constant all over the chart C – the height is constant
all over the chart D – the pressure is constant all over the chart Ref: all Ans: D
25579. (Refer to figure 050-98) A convergence line is indicated by: A – Sector A B
– Sector B C – Sector C D – Sector D Ref: all Ans: A 25580. (Refer to figure 050-
98) The warm sector is indicated by: A – Sector A B – Sector B C – Sector C D –
Sector D Ref: all Ans: C 25581. (Refer to figue 050-98) Which of the following best
describes Zone A? A – Trough of low pressure B – Cold Front C – Ridge of high
pressure D – Depression Ref: all Ans: A
25582. (Refer to figure 050-98) Which of the following best describes Zone B? A –
Ridge of high pressure B – Cold Front C – Depression D – Trough of low pressure
Ref: all Ans: B 25583. (Refer to figure 050-45) Which of the following best
describes Zone C? A – Ridge of high pressure B – Cold Front C – Trough of low
pressure D – Depression Ref: all Ans: A 25584. (Refer to figure 050-98) Which of
the following best describes Zone D? A – Ridge of high pressure B – Trough of low
pressure C – Anti-cyclone D – Depression Ref: all Ans: D 25585. (Refer to figure
050-98) What does zone A depict? A – A trough B – A ridge C – The warm sector D –
The Cold Front Ref: all Ans: A
25586. (Refer to figure 050-98) What does zone C depict? A – A trough B – A ridge C
– The warm sector D – The Cold Front Ref: all Ans: C 25587. (Refer to figure 050-
11) In which direction does the polar front move in this picture? A–1 B–2 C–3 D–4
Ref: all Ans: C 25588. (Refer to figure 050-12) In which direction does the polar
front move in this picture? A–D B–C C–B D–A Ref: all Ans: B 25589. (Refer to figure
050-36) Which airport has the biggest probability for rain at 1200 UTC? A – LIRF B
– LEMD C – LFPO D – LOWW Ref: all Ans: C
25595. (Refer to figure 050-46) Which airport has the biggest probability for the
formation of fog the following night? A – ENFB B – EKCH C – LFPO D – EINN Ref: all
Ans: C 25597. (Refer to figure 050-42) What does this picture depict? A – A high
pressure area over France B – A westerly wave over Central Europe C – South foehn D
– North foehn Ref: all Ans: B 25598. (Refer to figure 050-43) Where do the westerly
wves occur in this picture? A – In Central Europe B – Over Scandinavia C – In the
Mediterranean D – Over the Baltics Ref: all Ans: B 25600. (Refer to figure 050-100)
Assuming a normal vertical temperature gradient, at what altitude will the freezing
level above Tunis be found? A – FL 100 B – FL 20 C – FL 180 D – FL 260 Ref: all
Ans: A
25601. (Refer to figure 050-100) What is the mean temperature deviation from ISA
for the route FrankfurtRome at FL 180? A – 4oC colder than ISA B – 4oC warmer than
ISA C – 10oC colder than ISA D – 10oC warmer than ISA Ref: all Ans: A 25602. (Refer
to figure 050-107) What is the average temperature for the route Geneva-Stockholm,
FL 260? A - -55oC B - -51oC C - -63oC D - -47oC Ref: all Ans: D 25603. (Refer to
figure 050-69) Over Paris at what flight level would you expect to find the
tropopause according to the map? A – FL 300 B – FL 330 C – FL 150 D – FL 280 Ref:
all Ans: A 25604. (Refer to figure 050-84) On which of the following routes can you
expect icing to occur, on the basis of the chart? A – Hamburg – Oslo B – Rome –
Frankfurt C – Tunis – Rome D – Copenhagen – Helsinki Ref: all Ans: A
25605. (Refer to figure 050-85) Looking at the chart, at what altitude above
Frankfurt would you expect the tropopause to be located? A – FL 360 B – FL 410 C –
FL 330 D – FL 390 Ref: all Ans: A 25606. (Refer to figure 050-108) On which of
these routes would you not have to worry about turbulence at FL 240? A – Rome –
Berlin B – Zurich – Rome C – Zurich – Athens D – Shannon – Hamburg Ref: all Ans: D
25607. (Refer to figure 050-71) You are flying from Munich to Amsterdam. Which of
the following flight levels would you choose in order to avoid turbulence and
icing? A – FL 260 B – FL 320 C – FL 180 D – FL 140 Ref: all Ans: A
25608. (Refer to figure 050-77) What is the temperature deviation in degrees
Celsius, from the ICAO Standard Atmosphere overhead Frankfurt? A – ISA -13oC B –
ISA -2oC C – ISA +13oC D – ISA +2oC Ref: all Ans: A 25609. (Refer to figure 050-
109) State the temperature deviation to ISA over Copenhagen at FL 140: A – 8oC
colder than ISA B – 4oC warmer than ISA C – 8oC warmer than ISA D – 12oC colder
than ISA Ref: all Ans: A 25610. (Refer to figure 050-105_A) According to ICAO,
which symbol indicates danger to an aircraft flying according to instrument flight
rules (IFR)? A – Symbol 7 B – Symbol 5 C – Symbol 10 D – Symbol 3 Ref: AIR: atpl,
ir; HELI: atpl, ir Ans: D 25611. Which of the following symbols represents a
thunderstorm? A – Symbol 3 B – Symbol 4 C – Symbol 9 D – Symbol 1 Ref: AIR: atpl,
ir; HELI: atpl, ir Ans: A
25612. (Refer to figure 050-104_A) According to ICAO, which symbol indicates danger
to an aircraft flying according to instrument flight rules (IFR)? A – Symbol 7 B –
Symbol 1 C – Symbol 8 D – Symbol 9 Ref: AIR: atpl, ir; HELI: atpl, ir Ans: D 25613.
(Refer to figure 050-69) Over Paris at what flight level would you expect to find
the tropopause according to the map? A – FL 300 B – FL 330 C – FL 150 D – FL 280
Ref: all Ans: A 25614. (Refer to figue 050-75) What wind is forecast at FL 390 over
Paris? A – 210/40 B – 240/20 C – 030/40 D – 190/40 Ref: all Ans: A 25615. (Refer to
figure 050-70) Over Paris at what flight level would you expect to find the
tropopause according to the map? A – FL 330 B – FL 310 C – FL 300 D – FL 280 Ref:
all Ans: C
25616. (Refer to figue 050-71) To what extend is Zurich covered by clouds? A – 3 to
5 oktas B – 5 to 8 oktas C – 1 to 4 oktas D – 5 to 7 oktas Ref: all Ans: B 25617.
(Refer to figure 050-110) Which of the following best describes Zone A? A – Ridge
of high pressure B – Trough of low pressure C – Depression D – Col Ref: all Ans: B
25618. (Refer to figure 050-104) What pressure does an observer at point A
encounter in the next hour? A – Rising pressure B – Falling pressure C – A pressure
rise first, then an immediate pressure drop D – No substantial pressure change Ref:
all Ans: A 25619. (Refer to figure 050-104) What pressure does an observer at point
C encounter in the next hour? A – Rising pressure B – Falling pressure C – A
pressure rise first, then an immediate pressure drop D – No substantial pressure
change Ref: all Ans: B
25620. (Reer to figure 050-104) What temperature does an observer at point C
encounter in the next hour? A – The temperature decreases B – The temperature
increases C – The temperature first increases and then decreases D – No substantial
temperature change Ref: all Ans: B 25623. (Refer to figue 050-105_B) Which of the
following symbols represents a squall line? A – Symbol 3 B – Symbol 4 C – Symbol 5
D – Symbol 6 Ref: all Ans: C 25624. (Refer to figure 050-71) Flight Shannon to
London. What amount and type of cloud is forecast for the eastern sector of the
route between Shannon and London at FL 220? A – Individual cumulonimbus B –
Overcast nimbo layered cumulonimbus C – Scattered towering cumulus D – Scattered
castellanus Ref: all Ans: A 25625. (Refer to figue 050-80) To what extent is Munich
covered by clouds? A – 5 to 8 oktas B – 1 to 4 oktas C – 5 to 7 oktas D – 3 to 5
oktas Ref: all Ans: A
25627. (Refer to figure 050-13) An aircraft is flying from Point A to Point B on
the upper leel contour chart. The altimeter setting is 1013.2 hPa. Which of these
statements is correct? A – Wind speed at A is higher than at B B – The true
altitude will be higher at B than at A C – The true altitude will be higher at A
than at B D – Wind speed at Paris is higher than at B Ref: all Ans: C 25628. (Refer
to figure 050-46) Which airport is most likely to have radiation fog in the coming
night? A – ESSA B – ENGM C – EKCH D – LSZH Ref: all Ans: d 25629. (Refer to figure
050-89) If you are flying from Zurich to Shannon at FL 340, where will your
cruising altitude be? A – Constantly in the troposphere B – Constantly in the
stratosphere C – First in the troposphere and later in the stratosphere D – In the
stratosphere for part of time Ref: all Ans: B

050-10-03 Information for flight planning


9628. Runway visual range can be reported in: A – a TAF B – a METAR C – a SIGMET D
– both a TA and a METAR Ref: all
Ans: B 9632. You receive the following METAR: LSGG 0750Z 00000KT 0300 R05/0700N FG
VV001 M02/M02 Q1014 NOSIG= What will be the RVR at 0900 UTC? A – 300 m B – The RVR
is unknown, because the NOSIG does not refer to RVR C – 700 m D – 900 m Ref: all
Ans: B 9633. When will the surface wind in a METAR record a gust factor? A – When
gusts are at least 10 knots above the mean wind speed B – When gusts are at least
15 knots above the mean wind speed C – With gusts of at least 25 knots D – With
gusts of at least 35 knots Ref: all Ans: A 9637. Which of the following weather
reports could be, in accordance with the regulations, abbreviated to CAVOK? A –
29010KT 9999 SCT045TCU 16/12 Q1015 RESHRA NOSIG= B – 24009KT 6000 RA SCT010 OVC030
12/11 Q1007 TEMPO 4000= C – 15003KT 9999 BKN100 17/11 Q1024 NOSIG= D – 04012G26KT
9999 BKN030 11/07 Q1024 NOSIG= Ref: all Ans: C 9639. The follwing weather report
EDDM 241332 VRB03KT 1500 HZ OVC004 BECMG 1517 00000KT 0500 FG VV002 TEMPO 2022 0400
FG VV001 is a: A – METAR B – 24 hour TAF C – SPECI D – 9 hour TAF Ref: all Ans: D
9642. (Refr to figure 050-67) Over Madrid, what intensity of turbulence and icing
is forecast at FL 200? A – Moderate turbulence, moderate icing B – Moderate
turbulence, light icing C – Severe turbulence, moderate icing D – Severe
turbulence, severe icing Ref: all Ans: A 9643. Which of the following phenomena
should be described as precipitation at the time they are observed? A – TS B – SA C
– DZ D – SQ Ref: all Ans: C 9646. What is the meaning of the expression “FEW”? A –
2-4 oktas cloud cover B – 1-2 oktas cloud cover C – 5-7 oktas cloud cover D – 1-3
oktas cloud cover Ref: all Ans: B 9647. In which of the following METAR reports, is
the probability of fog formatin in the coming night the highest? A – 1850Z 15003KT
6000 SCT120 05/04 Q1032 BECMG 1600 BR= B – 1850Z 21003KT 8000 SCT250 12/m08 Q1028
NOSIG= C – 1850Z 06018G30KT 5000 OVC010 04/01 Q1024 NOSIG= D – 1850Z 25010KT 4000
RA BKN012 OVC030 12/10 Q1006 TEMPO 1500= Ref: all Ans: A
9651. (Refer to figure 050-83) Select from the map the average temperature for the
route Zurich-Lisboa at FL 200: A - -33oC B - -30oC C - -41oC D - -49oC Ref: all
Ans: A 9652. Marseille Information gives you the following meteorological
information for Ajaccio and Calvi for 16:00 UTC: Ajaccio: wind 360o/2 kt,
visibility 2000m, rain, BKN stratocumulus at 1000 ft, OVC altostratus at 8000 ft,
QNH 1023 hPa,Calvi: wind 040o/2 kt, visibility 3000m, mist, FEW stratus at 500 ft,
SCT stratocumulus at 2000 ft, OVC altostratus at 9000 ft, QNH 1023 hPa. The
ceilings (more than 4 oktas) are therefore: A – 1000 ft at Ajaccio and 2000 ft at
Calvi B – 1000 ft at Ajaccio and 500 ft at Calvi C – 8000 ft at Ajaccio and 9000 ft
at Calvi D – 1000 ft at Ajaccio and 9000 ft at Calvi Ref: all Ans: D 9657. What is
the meaning of the abbreviation BKN? A – 6 – 8 oktas B – 3 – 4 oktas C – 5 – 7
oktas D – 8 oktas Ref: all Ans: C
9661. Compare the following TAF and VOLMET reports for Nice: TAF 240716 VRB02KT
CAVOK = 0920Z 13012KT 8000 SCT040CB BKN100 20/18 Q1015 TEMPO TS= What can be
concluded from the differences between the two reports: A – That the weather in
Nice after 0920 is also likely to be as predicted in the TAF B – That the weather
conditions at 0920 were actually predicted in the TAF C – That the weather at Nice
is clearly more volatile than the TAF could have predicted earlier in the morning D
– That the VOLMET speaker has got his locations mixed up, because there is no way
the latest VOLMET report could be so different from the TAF Ref: all Ans: C 9662.
What units are used to report vertical wind shear? A – kt B – kt/100 ft C – m/100
ft D – m/sec Ref: all Ans: B 9665. (Refer to figure 050-70) In what height range
and at what intensity could you encounter turbulence in CAT area no2? A – From FL
220 to FL 400, moderate B – From FL 240 to FL 370, light C – From below FL 130 to
FL 270, light D – From FL 250 to FL 320, moderate Ref: all Ans: A
9666. What does the abbreviation “NOSIG” mean? A – No significant changes B – No
report received C – No weather related problems D – Not signed by the meteorologist
Ref: all Ans: A 9667. (Refer to figure 050-84) On which of these routes would you
have to worry about turbulence at FL 340? A – Rome – Berlin B – Zurich – Rome C –
Zurich – Athens D – Shannon – Hamburg Ref: all Ans: D 9668. A SPECI is: A – an
aviation routine weather report B – an aviation selected special weather report C –
a warning for special weather phenomena D – a forecast for special weather
phenomena Ref: all Ans: B 9673. What does the term METAR signify? A – A METAR is a
flight forecast, issued by the meteorological station several times daily B – A
METAR is a warning of dangerous meteorological conditions within a FIR C – A METAR
signifies the actual weather report at an aerodrome and is generally issued in
half-hourly intervals D – A METAR is a landing forecast added to the actual weather
report as a brief prognostic report Ref: all Ans: C
9674. Refer to the TAF for Amsterdam airport: FCNL31 281500 EHAM 281601 14010KT
6000 –RA SCT025 BECMG 1618 12015G25KT SCT008 BKN013 TEMPO 1823 3000 RA BKN005
OVC010 BECMG 2301 25020KT 8000 NSW BKN020= Flight from Bordeaux to Amsterdam, ETA
2100 UTC. What is the minimum visibility forecast for ETA Amsterdam? A – 5 km B – 3
km C – 5 NM D – 6 km Ref: all Ans: B 9675. How may the correct wind speed be found,
for a level, which is between two upper air chart levels? (eg. Wind at FL 250, when
the 500 hPa and the 300 hPa chart are available) A – By interpolation of the wind
information available from the two charts, while also considering the maximum wind
information found on the Significant Weather Chart B – By simple interpolation of
wind information available from the two charts C – By reading wind direction and
speed from the next higher chart D – By reading wind direction and speed from the
300 hPa chart Ref: all Ans: A 9676. (Refer to figure 050-86) If you are flying from
Zurich to London at FL 220, what conditions can you expect at cruising altitude? A
– Overcast nimbo layered cumulonimbus B – Individual cumulonimbus C – Scattered
towering cumulus D – Scattered castellanus Ref: all Ans: B
9678. (Refer to figure 050-74) Select from the map the wind for the route Zurich-
London at FL 280: A – 040/80 B – 250/80 C – 220/60 D – 160/90 Ref: all Ans: C 9681.
(Refer to figure 050-46) Which of the following weather conditions would be
expected at Athens Airport (LGAT) at around 1450 UTC? A – 16002KT 0200 R33L/0600N
FG W001 12/12 Q1031 BECMG 0800= B – 26014KT 8000 BKN090 17/12 Q1009 BECMG 4000 RA=
C – 21002KT 5000 HZ SCT040 29/16 Q1026 NOSIG= D – 23018G35KT 9999 SCT035 10/04
Q0988 NOSIG= Ref: all Ans: C 9683. What is the meaning of the abbreviation SCT? A –
1-2 oktas B – 3-4 oktas C – 5-7 oktas D – 1-4 oktas Ref: all Ans: B
9687. Which of the following statements is an interpretation of the SIGMET? SIGMET
VALID 121420/121820 embd t sobs and fcst in w part of athinai fir/mov e/inst nc = A
– Athens Airport is closed due to thunderstorms. The thunderstorm zone should be
east of Athens by 1820 UTC B – Thunderstorms have formed in the eastern part of the
Athens FIR and are slowly moving west C – Thunderstorms must be expected in the
western part of the Athens FIR. The thunderstorm zone is moving east. Intensity is
constant D – The thunderstorms in the Athens FIR are increasing in intensity, but
are stationary above the western part of the Athens FIR Ref: all Ans: C 9688. Given
the following METAR: EDDM 250850Z 33005KT 2000 R26R/P1500N R26L/1500N BR SCT002
OVC003 05/05 Q1025 NOSIG A – Visibility is reduced by water droplets B – There is a
distinct change in RVR observed C – Runway 26R and runway 26L have the same RVR D –
RVR on runway 26R is increasing Ref: all Ans: A 9691. (Refer to figure 050-97) What
is the deviation of the temperatue at FL 140 above Copenhagen compared to ISA? A –
4oC warmer than ISA B – 8oC colder than ISA C – 8oC warmer than ISA D – 29oC colder
than ISA Ref: all Ans: B
9692. ATIS information contains: A – operational information and if necessary
meteorological information B – only meteorological information C – meteorological
and operations information D – only operational information Ref: all Ans: C 9693.
Which of the following phenomena should be described as precipitation at the time
they are observed? A – FZFG B – BCFG C – SN D – HZ Ref: all Ans: C 9694. Which of
the following weather reports could be, in accordance with the regulations,
abbreviated to CAVOK? (MSA minus airport elevation equals: LSZB 10000 FT, LSZH 8000
FT, LSGG 12000 FT, LFSB 6000 FT) A – LFSB 24008KT 9999 SCT050 18/11 Q1017 RERA
NOSIG= B – LSZB 28012KT 9999 OVC100 16/12 Q1012 BECMG 5000 –RA= C – LSZH VRB02KT
9000 BKN080 21/14 Q1022 NOSIG= D – LSGG 22006KT 9999 BKN090 17/15 Q1008 RERA NOSIG=
Ref: all Ans: B 9696. What does the expression Broken (BKN) mean? A – 5-7 eights of
the sky is cloud covered B – 3-4 eights of the sky is cloud covered C – 3-5 eights
of the sky is cloud covered D – Nil significant cloud cover Ref: all Ans: A
9699. In which of the following circumstances is a SIGMET issued? A – Clear ice on
the runways of an aerodrome B – Fog or a thunderstorm at an aerodrome C – Severe
mountain waves D – A sudden change in the weather conditions contained in the METAR
Ref: all Ans: C 9704. Which of the four answers is a correct interpretation of data
from the following METAR? LSZH 050820Z 16003KT 0400 R14/P1500 R16/1000N FZFG VV003
M02/M02 Q1026 BECMG 2000= A – Meteorological visibility 1000m, RVR 400m, freezing
level at 300m, variable winds, temperature 2oC B – RVR for runway 16 1000m,
meteorological visibility increasing in the next two hours to 2000m, vertical
visibility 300m, temperature -2oC C – RVR for runway 14 1500m meteorological
visibility 400m, QNH 1026 hPa, wind 160o at 3 kt D – Meteorological visibility
400m, RVR for runway 16 1000m, dew point – 2oC freezing fog Ref: all Ans: D 9706.
(Refer to figure 050-36) What weather conditions are expected at Paris airport
(LFPO) around 0550 UTC? A – 23014KT 3000 +RA SCT008 OVC025 15/13 Q1004 NOSIG= B –
26012KT 9999 SCT025 SCT040 14/09 Q1018 TEMPO 5000 SHRA= C – 22020G36KT 1500 TSGR
SCT004 BKN007 BKN025CB 18/13 Q1009 BECMG NSW= D – 20004KT 8000 SCT110 SCT250 22/08
Q1016 NOSIG= Ref: all Ans: B
9709. Which of these four METAR reports suggests that a thunderstorm is likely in
the next few hours? A – 1350Z 21005KT 9999 SCT040CB SCT100 26/18 Q1016 TEMPO
24018G30 TS= B – 1350Z 16004KT 8000 SCT110 OVC220 02/M02 Q1008 NOSIG= C – 1350Z
34003KT 0800 SN VV002 M02/M04 Q1014 NOSIG= D – 1350Z 04012KT 3000 OVC012 04/03
Q1022 BECMG 5000= Ref: all Ans: A 9712. (Refer to figure 050-79) Over Amsterdam,
what amount and general type of cloud would you expect at FL 160? A – Mainly 5 to 8
oktas of stratiform cloud in layers B – 4 oktas broken cumulus C – Isolated
cumulonimbus only D – 5 to 7 oktas towering cumuliform cloud and with moderate
turbulence Ref: all Ans: A 9714. (Refer to figure 050-37) Which airport, at 1200
UTC, has the lowest probability of precipitation? A – ESSA B – LSZH C – ENFB D –
EFHK Ref: all Ans: B 9716. (Refer to figure 050-71) Flight Munich to London. What
is the direction and maximum speed of the jet stream affecting the route between
Munich and London? A – 220o / 120 kt B – 220o / 120 km/h C – 050o / 120 kt D – 230o
/ 120 m/sec Ref: all Ans: A
9717. What is a trend forecast? A – An aerodrome forecast valid for 9 hours B – A
landing forecast appended to METAR/SPECI, valid for 2 hours C – A route forecast
valid for 24 hours D – A routine report Ref: all Ans: B 9719. LSZH 061019
20018G30KT 9999 –RA SCT050 BKN080 TEMPO 23012KT 6000 –DZ BKN015 BKN030 BECMG 1518
23020G35KT 4000 RA OVC010= The lowest cloud base forecast at ETA Zurich (1200 UTC)
is: A – 1000 ft B – 1500 ft C – 1500 m D – 5000 ft Ref: all Ans: B 9721. If CAVOK
is reported then: A – no low drifting snow is present B – no clouds are present C –
low level windshear has not been reported D – any CBs have a base above 5000 ft
Ref: all Ans: A 9722. How long from the time of observation is a TREND in a METAR
valid? A – 9 hours B – 2 hours C – 1 hour D – 30 minutes Ref: all Ans: B
10445. In the weather briefing room during the pre-flight phase of a passenger
flight from Zurich to Rome, you examine the following weather reports of pressing
importance at the time: EINN SHANNON 2808 sigmet 2 valid 0800/1100 loc sev turb
fcst einn fir blw ft 050 south of 53n wkn= LIMM MILANO 2809 sigmet 2 valid
0900/1500 mod sev cat btn fl 250 and fl 430 fcst limm fir stnr nc= EGLL LONDON 2808
sigmet nr01 valid 0800/1200 for London fir isol cb embd in lyr cloud fcst tops fl
300 btn 52n and 54n east of 002e sev ice sev turb ts also fcst move wkn= Which
decision is correct? A – You show no further interest in these reports, since they
do nt concern the route to be flown B – Because of the expected turbulence you
select a flight level below FL 250 C – Owing to these reports and taking into
account the presence of heavy thunderstorms at planned FL 310 you select a higher
flight level (FL 370) D – You cancel the flight since the expected dangerous
weather conditions along the route would demand too much of the passengers Ref: all
Ans: B 10446. (Refer to figure 050-100) Assuming a normal vertical temperature
gradient, at what altitude will the freezing level above Tunis be found? A – FL 100
B – FL 20 C – FL 180 D – FL 260 Ref: all Ans: A
10447. Refer to the TAF for Amsterdam airport: FCNL31 281500 EHAM 281601 14010KT
6000 –RA SCT025 BECMG 1618 12015G25KT SCT008 BKN013 TEMPO 1823 3000 RA BKN005
OVC010 BECMG 2301 25020KT 8000 NSW BKN020= Flight from Bordeaux to Amsterdam, ETA
2100 UTC. At ETA Amsterdam what surface wind is forecast? A – 120o / 15 kt gusts 25
kt B – 140o / 10 kt C – 300o / 15 kt maximum wind 25 kt D – 250o / 20 kt Ref: all
Ans: A 10453. Which of the following phenomena should be described as precipitation
at the time they are observed? A - +SHSN B – VA C – BR D – MIFG Ref: all Ans: A
10454. (Refer to figure 050-83) Select from the map the average wind for the route
Zurich – Hamburg at FL 240: A – 230/20 B – 020/20 C – 200/15 D – 260/25 Ref: all
Ans: A
10459. Refer to the TAF for Amsterdam airport: FCNL31 281500 EHAM 281601 14010KT
6000 -RA SCT025 BECMG 1618 12015G25KT SCT008 BKN013 TEMPO 1823 3000 RA BKN005
OVC010 BECMG 2301 25020KT 8000 NSW BKN020= Flight from Bordeaux to Amsterdam, ETA
2100 UTC. What lowest cloud base is forecast for arrival at Amsterdam? A – 500m B –
250 ft C – 500 ft D – 800 ft Ref: all Ans: C 10460. In the TAF for Delhi, during
the summer, for the time of your landing you note: TEMPO TS What is the maximum
time this deterioration in weather can last in anyone instance? A – 120 minutes B –
60 minutes C – 10 minutes D – 20 minutes Ref: all Ans: B 10461. (Refer to figure
050-84) On which of the following routes can you expect icing to occur, on the
basis of the chart? A – Rome – Frankfurt B – Hamburg – Oslo C – Tunis – Rome D –
Copenhagen – Helsinki Ref: all Ans: B
10463. A pilot is warned of severe icing at certain flight levels by information
supplied in: A – TAF and SIGMET B – TAF and METAR C – METAR and SIGMET D – SWC and
SIGMET Ref: all Ans: D 10470. Refer to the TAF for Zurich Airport: TAF LSZH 250716
00000KT 0100 FG VV001 BECMG 0810 0800 VV002 BECMG 1012 23005KT 2500 BKN005 TEMPO
1316 6000 SCT007= Which of these statements best describes the weather that can be
expected at 1200 UTC? A – Meteorological visibility 6 kilometers, cloudbase 500
feet, wind speed 5 knots B – Meteorological visibility 800 metres, wind from 230o,
cloud base 500 feet C – Meteorological visibility 800 metres, vertical visibility
200 feet calm D – Meteorological visibility 2.5 kilometres, cloud base 500 feet,
wind speed 5 knots Ref: all Ans: D 10471. Which of the following phenomena can
product a risk of aqua planning? A – SA B – FG C – BCFG D - +RA Ref: all Ans: D
10473. Refer to the following TAF extract: BECMG 1821 2000 BKN004 PROB30 BECMG 2124
0500 FG VV001 What does the abbreviation VV001 mean? A – RVR greater than 100m B –
RVR less than 100m C – Vertical visibility 100 ft D – Vertical visibility 100m Ref:
all Ans: C 10479. (Refer to figure 050-77) What is the temperature deviation in
degrees Celsius from the International Standard Atmosphere overhead Frankfurt? A –
ISA -2oC B – ISA -13oC C – ISA +13oC D – ISA +2oC Ref: all Ans: B 10480. (Refer to
figure 050-108) Judging by the chart, what windspeeds can you expect at FL 340
above Rome? A – 340 kt B – 145 kt C – 95 kt D – 140 km/h Ref: all Ans: B 10481.
(Refer to figure 050-81) What OAT would you expect at FL 200 over Geneva? A - -24oC
B - -20oC C - -16oC D - -28oC Ref: all Ans: A
10483. Refer to the TAF for Bordeaux airport: FCFR31 281400 LFBD 1524 26015KT 9999
SHRA BKN020 TEMPO 1620 26020G30KT 8000 SHRA BKN015CB PROB30 TSRA= Flight Lisbon to
Bordeaux, ETA 1800 UTC. At ETA Bordeaux what is the lowest quoted visibility
forecast? A – 10 or more km B – 8 NM C – 10 NM D – 8 km Ref: all Ans: D 10485.
SIGMET information is issued as a warning for significant weather to: A – heavy
aircraft only B – light aircraft only C – VFR operations only D – all aircraft Ref:
all Ans: D 10488. Within a short interval, several flight crews report that they
have experienced strong clear air turbulence in a certain airspace. What is the
consequence of these reports? A – The competent aviation weather office will issue
a SPECI B – The airspace in question, will be temporarily closed C – The competent
aviation weather office will issue a SIGMET D – The competent aviation weather
office will issue a storm warning Ref: all Ans: C
10490. In which weather report would you expect to find information about icing
conditions on the runway? A – TAF B – SIGMET C – GAFOR D – METAR Ref: all Ans: D
10491. What does the term TREND signify? A – It is a flight forecast, issued by the
meteorological station several times daily B – It is the actual weather report at
an aerodrome and is generally issued at half-hourly intervals C – It is a warning
of dangerous meteorological conditions D – It is a brief landing forecast added to
the actual weather report Ref: all Ans: D 10492. (Refer to figure 050-109) Select
from the map the average temperature for the route Athens – Geneva at FL 150: A -
-21oC B - -14oC C - -11oC D - -27oC Ref: all Ans: B 10494. The cloud base, reported
in the METAR, is the height above: A – the highest terrain within a radius of 8 km
from the observation station B – mean sea level C – the pressure altitude of the
observation station at the time of observation D – airfield level Ref: all Ans: D
10495. (Refer to figure 050-90) Which of these statements is true? A – Turbulence
is likely to be encountered at FL 410 over Madrid B – Freezing level above Madris
is higher than FL 120 C – The front to the north of London is moving south D –
Scattered thunderstorms can be expected over France Ref: all Ans: D 10499. Refer to
the following TAF extract: BECMG 1821 2000 BKN004 PROB30 BECMG 2124 0500 FG VV001
What does the abbreviation PROB30 mean? A – Probability of 30% B – Conditions will
last for at least 30 minutes C – The cloud ceiling should lift to 3000 ft D –
Change expected in less than 30 minutes Ref: all Ans: A 10500. Refer to TAF below:
EGBB 261812 28015G25KT 9999 SCT025 TEMPO 1822 29018G35KT 5000 SHRASN BKN010CB
PROB30 TEMPO 1821 1500 TSGR BKN008CB BECMG 2124 26010KT From the TAF above you can
assume that visibility at 2055Z in Birmingham (EGBB) will be: A – not less than 1.5
km but could be in excess of 10 km B – a maximum of 5 km C – a minimum of 1.5 km
and a maximum of 5 km D – more than 10 km Ref: all Ans: A
10501. On the European continent METARs of main airports are compiled and
distributed with intervals of: A – 2 hours B – 1 hour C – 0.5 hours D – 3 hours
Ref: all Ans: C 10507. (Refer to figure 050-88) Judging by the chart, on which of
these routes can you expect to encounter moderate CAT at FL 300? A – Zurich –
Athens B – London – Zurich C – Zurich – Stockholm D – Madrid – Bordeaux Ref: all
Ans: A 10510. Which of the following weather reports is a warning of conditions
that could be potentially hazardous to aircraft in flight? A – SPECI B – ATIS C –
SIGMET D – TAF Ref: all Ans: C 10511. (Refer to figure 050-71) If you are flying
from Zurich to Stockholm at FL 280, what conditions can you expect at cruising
altitude? A – Out of cloud throughout the flight B – Cloud most of the way; little
chance of CAT C – Scattered thunderstorms D – Largely free of cloud; moderate
turbulence half way along the route Ref: all Ans: D
10512. (Refer to figure 050-82) Select from the map the average temperature for the
route Zurich – Rome at FL 110: A - -6oC B - -12oC C - -9oC D - +5oC Ref: all Ans: C
10513. (Refer to figure 050-46) Which airport is most likely to have fog in the
coming night? A – ENFB B – LSZH C – EKCH D – ESSA Ref: all Ans: B 10515. In which
of the following 1850 UTC METAR reports, is the probability of fog formation, in
the coming night, the highest? A – VRB02KT 2500 SCT120 14/M08 Q1035 NOSIG= B –
22004KT 6000 –RA SCT012 OVC030 17/14 Q1009 NOSIG= C – VRB01KT 8000 SCT250 11/10
Q1028 BECMG 3000 BR= D – 00000KT 9999 SCT300 21/01 Q1032 NOSIG= Ref: all Ans: C
10517. What does the term SIGMET signify? A – A SIGMET is a warning of dangerous
meteorological conditions B – A SIGMET is a flight forecast, issued by the
meteorological station several times daily C – A SIGMET is a brief landing forecast
added to the actual weather report D – A SIGMET is an actual weather report at an
aerodrome and is generally issued at half-hourly intervals Ref: all Ans: A
10521. Which of the statements is true concerning equal lines? A – Severe squall
lines always move from north west to south east B – Severe squall lines only occur
in the tropics C – For severe squall lines a TAF is issued D – For severe squall
lines a SIGMET is issued Ref: all Ans: D 10525. (Reer to figure 050-109) Select
from the map the average wind for the route Athens – Geneva at FL 160: A – 240/40 B
– 210/25 C – 260/45 D – 050/35 Ref: all Ans: A 10529. Refer to the following TAF
for Zurich: LSZH 261019 20018G30KT 9999 –RA SCT050 BKN080 TEMPO 23012KT 6000 –DZ
BKN015 BKN030 BECMG 1518 23020G35KT 4000 RA OVC010= The lowest visibility forecast
at ETA Zurich 1430 UTC is: A – 6 NM B – 6 km C – 4 km D – 10 km Ref: all Ans: B
10534. Which of these four METAR reports suggests that rain is most likely in the
next few hours? A – 23015KT 8000 BKN030 OVC070 17/14 Q1009 BECMG 4000= B – 34004KT
9999 SCT040 SCT100 m05/m08 Q1014 NOSIG= C – 16002KT 0100 FG SCT300 06/06 Q1022
BECMG 1000 D – 05016G33KT 8000 OVC015 08/06 Q1028 NOSIG= Ref: all Ans: A 10536.
Which of these statements best describes the weather most likely to be experienced
at 1500 UTC? TAF LSZH 211200Z 211322 22018G35KT 9999 SCT012 BKN030 BECMG 1315
25025G45KT TEMPO 1720 4000 +SHRA BKN025TCU BECMG 2022 25015KT TX18/15Z TN14/21Z= A
– Visibility 10 kilometres or more, ceiling 1200 feet, gusts up to 45 knots B –
Visibility 4000 metres, gusts up to 25 knots, temperature 18oC C – Visibility 10
kilometres or more, ceiling 3000 feet, wind 250o, temperature 18oC D – Severe
rainshowers, visibility 4000 metres, temperature 15oC, gusts up to 35 knots Ref:
all Ans: A 10537. Which of the following statements is an interpretation of the
METAR? 25020G38KT 1200 +TSGR BKN006 BKN015CB 23/18 Q1016 BECMG NSW= A – Gusts of 38
knots, thunderstorm with heavy hail, dew point 18oC B – Mean wind speed 20-38
knots, meteorological visibility 1200 metres, temperature 23oC C – Broken, cloud
base 600 feet and 1500 feet, temperature 18oC D – Wind 250o, thunderstorm with
moderate hail, QNH 1016 hPa Ref: all Ans: A
10539. (Refer to figure 050-68) What is the optimum flight level between Rome and
Paris according to the significant weather chart? A – FL 340 B – FL 220 C – FL 160
D – FL 180 Ref: all Ans: B 10545. Which of the following statements is an
interpretation of the METAR? 00000KT 0200 R14/0800U R16/P1500U FZFG VV001 m03/m03
Q1022 BECMG 0800= A – Meteorological visibility for runway 14 800 metres, fog with
hoar front, RVR for runway 16 more than 1500 metres B – Meteorological visibility
200 metres, RVR for runway 16 1500 metres, temperature -3oC, vertical visibility
100 metres C – Meteorological visibility 200 feet, RVR for runway 16 more than 1500
metres, vertical visibility 100 feet, fog with hoar frost D – RVR for runway 14 800
metres, vertical visibility 100 feet, calm, meteorological visibility improving to
800 metres in the next 2 hours Ref: all Ans: D 10546. Does the following report
make sense? LSZH VRB02KT 5000 MIFG 02/02 Q1015 NOSIG A – The report is nonsence,
because it is impossible to observe a meteorological visibility of 5 km if shallow
fog is reported B – The report would never be seen, because shallow fog is not
reported when the meteorological visibility is more than 2 km C – The report is
possible, because shallow fog is defined as a thin layer of fog below eye level D –
The report is not possible, because with a temperature of 2oC and a dew point of
2oC there must be uniform fog Ref: all Ans: C
10547. The wind directin in a METAR is measured relative to: A – the 0-meridian B –
magnetic north C – true north D – grid north Ref: all Ans: C 10551. In METAR
messages, the pressure group represents the: A – QNH rounded up to the nearest hPa
B – QFE rounded to the nearest hPa C – QNH rounded down to the nearest hPa D – QFE
rounded down to the nearest hPa Ref: all Ans: C 10552. (Refer to figue 050-100)
What is the mean temperature deviation from ISA for the Frankfurt – Rome route? A –
10oC colder than ISA B – 4oC warmer than ISA C – 4oC colder than ISA D – 10oC
warmer than ISA Ref: all Ans: C 10553. Appended to a METAR you get the following
runway report: 01650428. What must you consider when making performance
calculations? A – Aquaplaning conditions B – The braking action will be medium to
good C – The runway will be wet D – The friction co-efficient is 0.28 Ref: all Ans:
D
10557. Refer to the following TAF extract: BECMG 1821 2000 BKN004 PROB30 BECMG 2124
0500 FG VV001 What does the BECMG data indicate for the 18 to 21 hour time frame? A
– The new conditions are achieved between 1800 and 2100 UTC B – A quick change to
new conditions between 1800 UTC and 1900 UTC C – Many short term changes in the
original weather D – Many long term changes in the original weather Ref: all Ans: A
10578. (Refer to figure 050-46) On which airport do you expect the development of
the following weather situation? TAF 1322 24014G32KT 4000 +TSRA SCT005 BKN015
BKN020CB BECMG 1416 29012KT 9999 BKN030TCU SCT100 TEMPO 8000 SHRA BKN025TCU BECMG
1922 27012KT 9999 SCT030 OVC220= A – ESSA B – EINN C – LSZH D – EKCH Ref: all Ans:
B 15804. Refer to the TAF for Bordeaux airport. FCFR31 281400 LFBD 1524 26015KT
9999 SHRA BKN020 TEMPO 1620 26020G30KT 8000 +SHRA BKN015CB PROB30 TSRA= Flight
Lisbon to Bordeaux, ETA 1800 UTC. What type of precipitation is forecast on the
approach to Bordeaux? A – Light drizzle and fog B – Moderate snow showers C – Heavy
rain showers D – Continuous moderate rain Ref: all Ans: C
15819. Refr to the following TAF extract: BECMG 1821 2000 BKN004 PROB30 BECMG 2124
0500 FG 2001 What does the abbreviation BKN004 mean? A – 4-8 oktas, ceiling 400m B
– 1-4 oktas, ceiling 400m C – 5-7 oktas, ceiling 400 ft D – 1-4 oktas, ceiling 400
ft Ref: all Ans: C 15855. The validity of a TAF is: A – between 6 and 9 hours B – 9
hours from the time of issue C – stated in the TAF D – 2 hours Ref: all Ans: C
15856. The RVR, as reported in a METAR, is always the: A – value representative of
the touchdown zone B – average value of the A-, B- and C- position C – highest
value of the A-, B- and C- position D – lowest value of the A-, B- and C- position
Ref: all Ans: C 15880. A flight is to depart from an airport with runways 09 and
27. Surface wind is 270/05; an inversion is reported at 300 feet with turbulence
and wind shear. The wind just above the inversion is 090/30. What is the safest
departure procedure? A – Depart runway 27 with as steep an ascent as possible B –
Take off is not possible under these conditions C – Depart on runway 09 with a tail
wind D – Depart runway 27 with maximum throttle, during the passage through the
inversion Ref: all Ans: C
16529. The heights of cloud bases in TAFs are reported as being: A – AGL B – AMSL C
– AAL D – Pressure altitude Ref: all Ans: C 16551. How is the cloud base reported
in a METAR: A – In steps of 100 ft up to 10,000 ft and in steps of 1,000 ft above
10,000 ft B – In steps of 100m up to 10,000 ft and in steps of 1,000 ft above
10,000 ft C – In steps of 10 ft up to 10,000 ft and in steps of 1,000 ft above
10,000 ft D – It steps of 10m up to 10,000 ft and in steps of 1,000 ft above 10,000
ft Ref: all Ans: A 24173. (Refr to figure 050-115) At Lyon (LFLY, N4545 E00500), at
1200 UTC, the sky is overcast with Stratocumulus and Altostratus and it is raining.
Using the SWC in appendix, valid at 1200 UTC, we can estimate a weather improvement
for Lyon: A – 1330 UTC B – 2100 UTC C – 0300 UTC the following day D – 1215 UTC
Ref: all Ans: B
24176. (Refer to figure 050-36) At which airport, is the following weather
development taking place? TAF 060600Z 060716 25006KT 8000 BKN240 BECMG 0710 OVC200
BECMG 1013 23010KT 8000 OVC100 BECMG 1316 23014KT 6000 RA SCT030 OVC050= A – LFPO B
– EDDL C – LOWW D – LEMD Ref: all Ans: B 24187. (Refer to figure 050-04) To which
aerodrome is the following TAF most applicable? TAF 230900Z 231019 24014KT 6000
SCT030 BKN100 TEMPO 1113 5020G38KT 2500 +TSRA SCT008 BKN025CB BECMG 1315 28012KT
9999 SCT025 TEMPO 1617 5000 SHRA BKN020 A – EKCH B – LFPG C – LEMD D – LOWW Ref:
all Ans: D 24252. Below is the forecast for the destination of a flight whose ETA
is 2030 UTC. TAF YUDO 120600Z 121206 17010KT 3000 SCT005 BECMG 1820 RB03KT BCFG SKC
BECMG 2022 0300 FG VV002= What can be said about the destination airport at 2030
UTC if CAT II ILS inimum: A – Open the whole night because 300 m of visibility
corresponds to at least 900m RVR B – Still open until 2200 UTC because the weather
change is not completed before 2200 UTC C – Has to be considered as closed D –
Still open because the decrease of the visibility below the CAT II minima will be a
little bit later than 2030 UTC Ref: all Ans: C
24315. Refer to the following TAF extract: BECMG 1821 2000 BR BKN004 BECMG 2123
0500 FG VV001 What visibility is forecast for 2400 UTC? A – 500 m B – 2000 m C –
Between 5000m and 2000m D – Between 0m and 1000m Ref: all Ans: A 24316. Refer to
the following TAF message LFxx 180800Z 180918 22020KT 6000 SCT015 SCT080 BECMG 1214
24025KT 2000 RA BKN009 OVC070= At 1400 UTC, the lowest cloud base will be: A –
between 900 and 1500 feet AMSL B – at 900 feet AGL C – between 900 and 1500 feet
AGL D – at 1500 feet AGL Ref: all Ans: B 24322. TAF EHAM 142300Z 150009 33005KT
9999 SCT025 BKN100 BECMG 0002 27015KT 4500 –SN SCT008 OVC015 TEMPO 0206 0400 +SN
VV002 BECMG 0406 01008KT 9999 NSW SCT030 TEMPO 0709 03015G25KT 1200 SNSH SCT006
SCT015CB= What is the expected visibility at 0300 UTC? A – 4500m B – Between 400m
and 10 km or more C – Between 4500m and 10 km or moe D – Between 400m and 4500 m
Ref: all Ans: D
24323. TAF LSZH 250600Z 250716 00000KT 0100 FG VV001 BECMG 0810 0800 VV002 BECMG
1012 23005KT 2500 BR BKN005 TEMPO 1316 6000 SCT007= Which of these statements best
describes the weather that can be expected at 1200 UTC? A – Visibility 800 metres,
fog, vertical visibility 200 feet, calm B – Visibility 800 metres, fog, wind from
230o, cloud base 500 feet C – Visibility 2.5 kilometres, mist, cloud base 500 feet,
wind speed 5 knots D – Visibility 6 kilometres, cloud base 500 feet, wind speed 5
knots Ref: all Ans: C 24333. The following weather message: EDDM 241200Z 241322
VRB03KT 1500 BR OVC004 BECMG 1517 00000KT 0500 FG VV002 TEMPO 2022 0400 FG VV001 Is
a: A – METAR B – 24 hour TAF C – SPECI D – 9 hour TAF Ref: all Ans: D 24351. The
TAF weather messages are: A – airport forecasts B – hourly or semi-hourly weather
observations C – special weather observations D – landing forecasts of the “trend”
type Ref: all Ans: A
24354. The term CAVOK is used when weather conditions are: A – 9999, NSC, NOSIG B –
9000, SKC, NOSIG C – 8000, HAZARDOUS WX NIL, NOSIG D – 9999, NSC, NSW Ref: all Ans:
D 24355. The term PROB as used in a TAF message, indicates the probability in
percentage, of phenomena described during a specific period. The numerical values
immediately following the term PROB, are: A – 25 or 25 B – 20 or 30 C – 30 or 40 D
– 35 or 50 Ref: all Ans: C 24363. What does the code TAF AMD mean? A – Revised TAF
B – No change compared with the previous TAF C – Weather conditions expected to
affect the safety of normal operations D – Delayed issue of a TAF Ref: all Ans: A
24376. What is the height of the lowest cloud forecast for Bangkok at ETA 1400 UTC?
TAF VTBD 271800Z 280024 VRB05KT 1200 BR TEMPO 0002 4000 BECMG 0205 9999 SCT015
BKN100 TEMPO 1118 04010G20KT 5000 TSRA SCT009 BKN014 BKN018CB BECMG 2024 6000= A –
1800 feet B – 1200 feet C – 1400 feet D – 900 feet Ref: all Ans: D
24377. What is the lowest cloud base forecast for arrival at Geneva (ETA 1200 UTC)?
TAF LSGG 020900Z 021019 18007KT 9999 SCT020 BKN100 BECMG 1114 8000 RA SCT010 OVC020
PROB30 TEMPO 1018 VRB10G25KT TSRA SCT005 BKN015CB= A – 500 ft B – 500 m C – 1000 ft
D – 1000 m Ref: all Ans: A 24380. What is the lowest probably cloud base forecast
for ETA 1700 UTC at Kingston? TAF MKJP 160000Z 160606 36010KT 9999 FEW025 BECMG
1315 14020G34KT FEW015CB SCT025 PROB30 TEMPO 1520 6000 +SHRA SCT010 BKN015CB BECMG
2301 34010KT FEW025= A – 1500 m B – 1000 m C – 1500 ft D – 1000 ft Ref: all Ans: D
24382. What is the lowest visibility forecast for approach into Geneva (ETA 1200
UTC)? TAF LSGG 020900Z 021019 18007KT 9999 SCT020 BKN100 BECMG 1114 8000 RA SCT010
OVC020 PROB30 TEMPO 1018 VRB10G25KT TSRA SCT005 BKN015CB= A – 8 NM B – 8 km C – 6
NM D – 10 km Ref: all Ans: B
24383. What is the lowest visibility forecast for Bangkok at ETA 1400 UTC? TAF VTBD
271800Z 280024 VRB05KT 1200 BR TEMPO 0002 4000 BECMG 0205 9999 SCT015 BKN100 TEMPO
1118 04010G20KT 5000 TSRA SCT009 BKN014 BKN018CB BECMG 2024 6000= A – 1200 metres B
– 5 km C – 6 km D – 10 km or more Ref: all Ans: B 24384. What is the lowest
visibility that may be expected during an approach into Dhahran at ETA 0600 UTC?
TAF OEDR 280000Z 280110 VRB08KT CABOK BECMG 0103 7000 TEMPO 0410 28014G24KT 4000
SA= A – 4000 yards B – 10 km or more C – 7 km D – 4 km Ref: all Ans: D 24388. What
lowest cloud base is most likely to be experienced during an approach into Madrid
at 2300 UTC? TAF LEMD 281200Z 281812 13005KT CAVOK TEMPO 1821 8000 SCT020 SCT030
BECMG 2123 21005KT 9999 SCT015 BKN080 PROB40 TEMPO 2306 6000 SCT008 A – 2000 feet B
– 1500 feet C – 3000 feet D – 800 feet Ref: all Ans: B
24389. What surface wind is forecast for 2200 UTC? EDDF 272200Z 280624 VRB05KT 4000
BR SCT005 OVC013 BECMG 1314 9000 SHRA OVC 015 PROB40 TEMPO 1416 VRB15G25KT 1600
TSRA OVC010CB BECMG 1618 26010KT BKN030 BECMG 2122 CAVOK A – 260o / 10 kt B –
Variable / 05 kt C – Variable / 15 to 25 kt D – Calm Ref: all Ans: A 24390. What
surface wind is forecast for ETA 1700 UTC at Kingston? MKJP 160430Z 160606 36010KT
9999 FEW025CB BECMG1315 14020G34KT FEW015CB SCT025 PROB30 TEMPO 1720 6000 +SHRA
SCT010 BKN015CB BECMG 2301 34010KT FEW025= A – 360o / 10 kt B – 140o / 20 kt gusts
34 kt C – 340o / 10 kt D – 140o / 27 kt Ref: all Ans: B 24391. What type of
meteorological hazard to safe flight is most likely to be experienced during the
final approach to Geneva (ETA 1200 UTC)? TAF LSGG 020900Z 021019 18007KT 9999
SCT020 BKN100 BECMG 1114 8000 RA SCT010 OVC020 PROB30 TEMPO 1018 VRB10G25KT TSRA A
– Anabatic winds B – Roll cloud associated with standing waves C – Radiation fog D
– Low level wind shear Ref: all Ans: D
24392. What type of precipitation might occur at 1700 UTC? MKJP 160430Z 160606
36010KT 9999 FEW025 BECMG 1315 14020G34KT FEW015CB SCT025 PROB30 TEMPO 1720 6000
+SHRA SCT010 BKN015CB BECMG 2224 34010KT FEW025= A – Heavy rain showers B – Light
drizzle C – Continuous moderate rain D – Intermittent light rain Ref: all Ans: A
24395. What visibility is most likely to be experienced during an approach into
Madrid at 2300 UTC? TAF LEMD 281200Z 281812 13005KT CAVOK TEMPO 1821 8000 SCT020
SCT030 BECMG 2123 21005KT 9999 SCT015 BKN080 PROB40 TEMPO 2306 6000 SCT008 A – 6000
metres B – 8000 metres C – 10 kilometres or more D – Greater than 10 kilometres
Ref: all Ans: C 24416. Which of the following phenomena has to be mentioned in a
SIGMET? A – Heavy duststorm B – Strong inversion C – Thick fog D – Snow and ice on
the runway Ref: all Ans: A
24417. Which of the following phenomena has to be mentioned in a SIGMET? A – Strong
inversion B – Heavy duststorm C – Thick fog D – Snow and ice on the runway Ref: all
Ans: B 25577. (Refer to figure 050-106) The following TAF applies best to which
aerodrome? 19010KT 8000 RA BKN014 TEMPO 1518 4000 RADZ BKN010 A – EBBR B – LOWW C –
MADRID D – PARIS Ref: all Ans: A 25590. (Refer to figure 050-36) Which weather
situation do you expect for EGLL at 1150 UTC? A – 23015KT 8000 SCT100 BKN200 21/07
Q1002 NOSIG= B – 32002KT 3000 OBC006 16/12 Q1024 TEMPO 8000= C – 28006KT 4000 =TSRA
SCT012 BKN030CB 19/14 Q1022 BECMG NSW= D – 17016G28KT 2500 +RA BKN009 OVC 018 12/10
Q0994 BECMG 8000= Ref: all Ans: D
25591. (Refer to figure 050-36) At which airport is the following weather
development most likely to be taking place? TAF 060716 25006KT 8000 BKN240 BECMG
0710 OVC200 BECMG 1013 23010KT 8000 OVC100 BECMG 1316 23014KT 6000 RA SCT030
OVC050= A – LFPO B – EDDL C – LEMD D – EPWA Ref: all Ans: B 25592. (Refer to figure
050-36) On which airport do you expect the development of the following most likely
weather situation? TAF 1019 21010KT 8000 SCT120 OCT 180 BECMG 1013 OVC090 TEMPO –RA
BECMG 1417 22016KT 5000 RA BKN020 OVC030 TEMPO 3000 +RA BKN012 OVC020 A – LFPO B –
EGLL C – EDDL D – LOWW Ref: all Ans: C 25593. (Refer to figure 050-46) At which
airport is the following weather development taking place? TAF 231322 24014G32KT
4000 TSRA SCT005 BKN015 BKN020CB BECMG 1416 29012KT 9999 BKN030TCU SCT100 TEMPO
8000 SHRA BKN025TCU BECMG 1922 27012KT 9999 SCT030 OVC220= A – EINN B – LFPO C –
LFML D – EKCH Ref: all Ans: A
25594. (Refer to figure 050-46) The attached chart shows the weather conditions on
the ground at 1200 UTC on May 23. Which of the following reports reflects weather
development at Zurich Airport? A – TAF LSZH 101601 05020G35KT 8000 BKN015 TEMPO
1720 05018KT 0300 +SHSN VV002= B – TAF LSZH 10161 23012KT 6000 RA BKN012 OVC030
TEMPO 2023 22025G40KT 1600 +SNRA BKN003 OVC015= C – TAF LSZH 101601 32008KT 9999
SCT030TCU TEMPO 2201 32020G32KT 3000 TSRA BKN020CB= D – TAF LSZH 101601 VRB02KT
8000 SCT280 BECMG 1618 00000KT 3500 MIFG BECMG 1820 1500 BCFG BECMG 2022 0100 FG
VV001= Ref: all Ans: D 25599. (Refer to figure 050-44) The attached chart shows
isobars and fronts at 1200 UTC on October 10. Which of the following reports
reflects weather development at Zurich Airport (LSZH)? A – TAF LSZH 101500Z 101601
VRB02KT 8000 SCT280 BECMG 1618 00000KT 3500 MIFG BECMG 1820 1500 BCFG BECMG 2022
0100 FG VV001= B – TAF LSZH 101500Z 101601 05020G35KT 8000 BKN015 TEMPO 1720
05018KT 0300 +SHSN VV002= C – TAF LSZH 101500Z 101601 23012KT 6000 RA BKN012 OVC030
TEMPO 2023 22025G40KT 1600 +SNRA BKN003 OVC015= D – TAF LSZH 101500Z 101601 32008KT
9999 SCT030TCU TEMPO 2201 32020G32KT 3000 TSRA BKN020CB= Ref: all Ans: A

Vous aimerez peut-être aussi